You are on page 1of 738

Chapter 1 Solutions

Engineering and Chemical Thermodynamics

Wyatt Tenhaeff
Milo Koretsky
Department of Chemical Engineering
Oregon State University
koretsm@engr.orst.edu

1.2
An approximate solution can be found if we combine Equations 1.4 and 1.5:

1 2
mV ekmolecular
2
3
kT ekmolecular
2

3kT
V
m

Assume the temperature is 22 C. The mass of a single oxygen molecule is m 5.14 10 26 kg .


Substitute and solve:

V 487.6 m/s

The molecules are traveling really, fast (around the length of five football fields every second).
Comment:
We can get a better solution by using the Maxwell-Boltzmann distribution of speeds that is
sketched in Figure 1.4. Looking up the quantitative expression for this expression, we have:
m
f (v)dv 4

2kT

3/ 2

m 2 2
v v dv
exp
2kT

where f(v) is the fraction of molecules within dv of the speed v. We can find the average speed
by integrating the expression above

f (v)vdv
0

f (v)dv

8kT
449 m/s
m

1.3
Derive the following expressions by combining Equations 1.4 and 1.5:

3kT
Va2
ma

3kT
Vb2
mb

Therefore,

Va2 mb

Vb2 ma

Since mb is larger than ma, the molecules of species A move faster on average.

1.4
We have the following two points that relate the Reamur temperature scale to the Celsius scale:

0 C, 0 Reamur and 100 C, 80 Reamur


Create an equation using the two points:
T Reamur 0.8 T Celsius

At 22 C,
T 17.6 Reamur

1.5
(a)
After a short time, the temperature gradient in the copper block is changing (unsteady state), so
the system is not in equilibrium.
(b)
After a long time, the temperature gradient in the copper block will become constant (steady
state), but because the temperature is not uniform everywhere, the system is not in equilibrium.
(c)
After a very long time, the temperature of the reservoirs will equilibrate; The system is then
homogenous in temperature. The system is in thermal equilibrium.

1.6
We assume the temperature is constant at 0 C. The molecular weight of air is

MW 29 g/mol 0.029 kg/mol


Find the pressure at the top of Mount Everest:

0.029 kg/mol9.81 m/s8848 m


P 1 atm exp

2
73.15
K

8.314

mol K

P 0.330 atm 33.4 kPa

Interpolate steam table data:

T sat 71.4 C

for P sat 33.4 kPa

Therefore, the liquid boils at 71.4 C. Note: the barometric relationship given assumes that the
temperature remains constant. In reality the temperature decreases with height as we go up the
mountain. However, a solution in which T and P vary with height is not as straight-forward.

1.7
To solve these problems, the steam tables were used. The values given for each part constrain
the water to a certain state. In most cases we can look at the saturated table, to determine the
state.
(a)

(b)

(c)

(d)

Subcooled liquid
Explanation: the saturation pressure at T = 170 [oC] is 0.79 [MPa] (see page 508);
Since the pressure of this state, 10 [bar],
is greater than the saturation
pressure, water is a liquid.
Saturated vapor-liquid mixture
Explanation: the specific volume of the saturated vapor at T = 70 [oC] is 5.04
[m3/kg] and the saturated liquid is 0.001 [m3/kg] (see page 508); Since the volume
of this state, 3 [m3/kg], is in between these values we have a saturated vaporliquid mixture.
Superheated vapor
Explanation: the specific volume of the saturated vapor at P = 60 [bar] = 6 [MPa],
is 0.03244 [m3/kg] and the saturated liquid is 0.001 [m3/kg] (see page 511); Since
the volume of this state, 0.05 [m3/kg], is greater than this value, it is a vapor.
Superheated vapor
Explanation: the specific entropy of the saturated vapor at P = 5 [bar] = 0.5
[MPa], is 6.8212 [kJ/(kg K)] (see page 510); Since the entropy of this state,
7.0592 [kJ/(kg K)], is greater than this value, it is a vapor. In fact, if we go to the
superheated water vapor tables for P = 500 [kPa], we see the state is constrained
to T = 200 [oC].

1.8
From the steam tables in Appendix B.1:

m3
v critical 0.003155

kg

T 374.15 C, P 22.089 MPa

At 10 bar, we find in the steam tables


vlsat

m3
0.001127

kg

m3
vvsat 0.19444

kg
Because the total mass and volume of the closed, rigid system remain constant as the water
condenses, we can develop the following expression:
v critical 1 x vlsat xvvsat

where x is the quality of the water. Substituting values and solving for the quality, we obtain
x 0.0105 or 1.05 %

A very small percentage of mass in the final state is vapor.

1.9
The calculation methods will be shown for part (a), but not parts (b) and (c)
(a)
Use the following equation to estimate the specific volume:
v1.9 MPa, 250 C v1.8 MPa, 250 C 0.5v2.0 MPa, 250 C v1.8 MPa, 250 C
Substituting data from the steam tables,

m3
v1.9 MPa, 250 C 0.11821

kg
From the NIST website:
m3
v NIST 1.9 MPa, 250 C 0.11791

kg
Therefore, assuming the result from NIST is more accurate
v v NIST

% Difference
100 % 0.254 %
v NIST

(b)
Linear interpolation:

m3
v1.9 MPa, 300 C 0.13284

kg
NIST website:
m3
v NIST 1.9 MPa, 300 C 0.13249

kg
Therefore,
% Difference 0.264 %

(c)
Linear interpolation:

m3
v1.9 MPa, 270 C 0.12406

kg

Note: Double interpolation is required to determine this value. First, find the molar volumes at
270 C and 1.8 MPa and 2.0 MPa using interpolation. Then, interpolate between the results from
the previous step to find the molar volume at 270 C and 1.9 MPa.
NIST website:
m3
v NIST 1.9 MPa, 270 C 0.12389

kg
Therefore,
% Difference 0.137 %

With regards to parts (a), (b), and (c), the values found using interpolation and the NIST website
agree very well. The discrepancies will not significantly affect the accuracy of any subsequent
calculations.

10

1.10
For saturated temperature data at 25 C in the steam tables,
m3
L
vlsat 0.001003 1.003
kg
kg

Determine the mass:


m

vlsat

1 L
0.997 kg
L
1.003
kg

Because molar volumes of liquids do not depend strongly on pressure, the mass of water at 25 C
and atmospheric pressure in a one liter should approximately be equal to the mass calculated
above unless the pressure is very, very large.

11

1.11
First, find the overall specific volume of the water in the container:

m3
1 m3
0.2

5 kg
kg

Examining the data in the saturated steam tables, we find


at P sat 2 bar

vlsat v vvsat

Therefore, the system contains saturated water and the temperature is


T T sat 120.23 C
Let ml represent the mass of the water in the container that is liquid, and mv represent the mass
of the water in the container that is gas. These two masses are constrained as follows:
ml mv 5 kg
We also have the extensive volume of the system equal the extensive volume of each phase
ml vlsat mv vvsat V

ml 0.001061 m 3 / kg mv 0.8857 m 3 /kg 1 m 3


Solving these two equations simultaneously, we obtain
ml 3.88 kg
mv 1.12 kg
Thus, the quality is
x

mv 1.12 kg

0.224
m
5 kg

The internal energy relative to the reference state in the saturated steam table is
U ml ulsat mv uvsat

From the steam tables:

12

ulsat 504.47 kJ/kg

uvsat 2529.5 kJ/kg


Therefore,
U 3.88 kg 504.47 kJ/kg 1.12 kg 2529.5 kJ/kg
U 4790.4 kJ

13

1.12
First, determine the total mass of water in the container. Since we know that 10 % of the mass is
vapor, we can write the following expression

V m 0.9 vlsat 0.1vvsat

From the saturated steam tables for P sat 1 MPa


m3
sat
vl 0.001127

kg

m3
vvsat 0.1944

kg
Therefore,
m

0.9vlsat 0.1vvsat

m 48.9 kg

1 m3

m3
m3
0.9 0.001127

0.1 0.1944

kg
kg

and

Vv 0.1mvvsat 0.148.9 kg 0.1944 m 3 / kg


Vv 0.950 m 3

14

1.13
In a spreadsheet, make two columns: one for the specific volume of water and another for the
pressure. First, copy the specific volumes of liquid water from the steam tables and the
corresponding saturation pressures. After you have finished tabulating pressure/volume data for
liquid water, list the specific volumes and saturation pressures of water vapor. Every data point
is not required, but be sure to include extra points near the critical values. The data when plotted
on a logarithmic scale should look like the following plot.
The Vapor-Liquid Dome for H2O
100.0000
Critical Point

Pressure, P (MPa)

10.0000
Liquid
1.0000
0.0001

Vapor-Liquid

0.001

0.01

Vapor

0.1

10

0.1000

0.0100

0.0010

Specific Volume, v (m 3/kg)

15

100

1000

1.14
The ideal gas law can be rewritten as

RT
P

For each part in the problem, the appropriate values are substituted into this equation where
L bar
R 0.08314
mol K
is used. The values are then converted to the units used in the steam table.
(a)

L
v 30.7
mol
L
30.7
3
3
v
mol 10 3 m 1.70 m
v

kg
L
MW
kg


0.018

mol
For part (a), the calculation of the percent error will be demonstrated. The percent error will be
based on percent error from steam table data, which should be more accurate than using the ideal
gas law. The following equation is used:
v vST

100 %
%Error IG
vST

where v IG is the value calculated using the ideal gas law and vST is the value from the steam
table.
3
3

1.70 m 1.6729 m

kg
kg
%Error

m3

1.6729

kg

% 1.62%

This result suggests that you would introduce an error of around 1.6% if you characterize boiling
water at atmospheric pressure as an ideal gas.

16

(b)

L
v 64.3
mol
m3
v 3.57

kg
% Error 0.126%
For a given pressure, when the temperature is raised the gas behaves more like an ideal gas.
(c)

L
v 0.643
mol
m3
v 0.0357

kg
%Error = 8.87%
(d)

L
v 1.06
mol
m3
v 0.0588

kg
%Error = 0.823%

The largest error occurs at high P and low T.

17

1.15
The room I am sitting in right now is approximately 16 ft long by 12 feet wide by 10 feet tall
your answer should vary. The volume of the room is

V 12 ft 16 ft 10 ft 1920 ft 3 54.4 m 3
The room is at atmospheric pressure and a temperature of 22 C. Calculate the number of moles
using the ideal gas law:

1.01325 10 5 Pa 54.4 m 3
PV

RT
J
295.15 K
8.314
mol K

n 2246 mol

Use the molecular weight of air to obtain the mass:


m nMW 2246.27 mol0.029 kg/mol 65.2 kg

Thats pretty heavy.

18

1.16
First, find the total volume that one mole of gas occupies. Use the ideal gas law:

RT

v
P

J
8.314
293.15 K
mol K

1105 Pa

m3
0.0244

mol

Now calculate the volume occupied by the molecules:


Number of molecules Volume of
4

N A r 3

v occupied
per mole
3

one molcule

6.022 10 23 molecules 4
3
1.5 10 10 m
v occupied

1 mol

m3
v occupied 8.5110 6

mol

Determine the percentage of the total volume occupied by the molecules:


v occupied

Percentage
100 % 0.0349 %
v

A very small amount of space, indeed.

19

1.17
Water condenses on your walls when the water is in liquid-vapor equilibrium. To find the
maximum allowable density at 70 F, we need to find the smallest density of water vapor at 40 F
which satisfies equilibrium conditions. In other words, we must find the saturation density of
water vapor at 40 F. From the steam tables,

m3
vvsat 153.74

kg

(sat. water vapor at 40 F = 4.44 C)

We must correct the specific volume for the day-time temperature of 70 F (21.1 C ) with the
ideal gas law.

vvsat
,277.6 K
T277.6

v
294.3 K
T294.3

m3
m3
T
294.3 K

v294.3 K 294.3 vvsat

153
.
74
163

, 277.6 K
T277.6
277.6 K
kg
kg

Therefore,

1
vvsat

kg
6.13 10 3

m3

If the density at 70 F is greater than or equal to 6.13 10 3 kg/m 3 , the density at night when the
temperature is 40 F will be greater than the saturation density, so water will condense onto the

wall. However, if the density is less than 6.13 10 3 kg/m 3 , then saturation conditions will not
be obtained, and water will not condense onto the walls.

20

1.18
We consider the air inside the soccer ball as the system. We can answer this question by looking
at the ideal gas law:
Pv RT

If we assume it is a closed system, it will have the same number of moles of air in the winter as
the summer. However, it is colder in the winter (T is lower), so the ideal gas law tells us that Pv
will be lower and the balls will be under inflated.
Alternatively, we can argue that the higher pressure inside the ball causes air to leak out over
time. Thus we have an open system and the number of moles decrease with time leading to the
under inflation.

21

1.19
First, write an equation for the volume of the system in its initial state:
ml vlsat mv vvsat V

Substitute ml 1 x m and mv xm :

m 1 x vlsat xvvsat V

At the critical point

mv critical V
Since the mass and volume dont change
v critical 1 x vlsat xvvsat

From the steam tables in Appendix B.1:


m3
v critical 0.003155

kg

T 374.15 C, P 22.089 MPa

At 0.1 MPa, we find in the steam tables


m3
vlsat 0.001043

kg
m3
sat
vv 1.6940

kg
Therefore, solving for the quality, we get
x 0.00125 or 0.125 %

99.875% of the water is liquid.

22

1.20
As defined in the problem statement, the relative humidity can be calculated as follows

Relative Humidity

mass of water
mass of water at saturation

We can obtain the saturation pressure at each temperature from the steam tables. At 10 [oC], the
saturation pressure of water is 1.22 [kPa]. This value is proportional to the mass of water in the
vapor at saturation. For 90% relative humidity, the partial pressure of water in the vapor is:

pWater 0.9 1.22 1.10 [kPa]


This value represents the vapor pressure of water in the air. At 30 [oC], the saturation pressure of
water is 4.25 [kPa]. For 590% relative humidity, the partial pressure of water in the vapor is:

pWater 0.5 5.63 2.12 [kPa]


Since the total pressure in each case is the same (atmospheric), the partial pressure is
proportional to the mass of water in the vapor. We conclude there is about twice the amount of
water in the air in the latter case.

23

1.21
(a)
When you have extensive variables, you do not need to know how many moles of each substance
are present. The volumes can simply be summed.

V1 Va Vb
(b)
The molar volume, v1, is equal to the total volume divided by the total number of moles.
v1

V1 Va Vb

ntot na nb

We can rewrite the above equation to include molar volumes for species a and b.
nb
n v nb vb
na
vb
va
v1 a a

na nb
na nb
na nb
v1 xa va xb vb

(c)
The relationship developed in Part (a) holds true for all extensive variables.

K1 K a K b
(d)

k1 xa k a xb kb

24

Chapter 2 Solutions
Engineering and Chemical Thermodynamics

Wyatt Tenhaeff
Milo Koretsky
Department of Chemical Engineering
Oregon State University
koretsm@engr.orst.edu

2.1
There are many possible solutions to this problem. Assumptions must be made to solve the
problem. One solution is as follows. First, assume that half of a kilogram is absorbed by the
towel when you dry yourself. In other words, let
m H 2 O 0.5 kg

Assume that the pressure is constant at 1.01 bar during the drying process. Performing an energy
balance and neglecting potential and kinetic energy effects reveals

q h
Refer to the development of Equation 2.57 in the text to see how this result is achieved. To find
the minimum energy required for drying the towel, assume that the temperature of the towel
remains constant at T 25 C 298.15 K . In the drying process, the absorbed water is
vaporized into steam. Therefore, the expression for heat is
v
l
q hH
hH
2O
2O

v
is the specific enthalpy of water vapor at P 1.01 bar and T 298.15 K and
where is hH
O
2
h l
is the specific enthalpy of liquid water at P 1.01 bar and T 298.15 K . A hypothetical
H 2O

path must be used to calculate the change in enthalpy. Refer to the diagram below
P

liquid
1 atm

vapor

P = 1 [atm]
h3

h1
P

3.17 kPa

sat

h2

liquid

vapor

By adding up each step of the hypothetical path, the expression for heat is
q h1 h2 h3
h l ,sat 25 C h l

25 C, 1.01 bar hHv,satO 25 C hHl ,satO 25 C


hHv O 25 C, 1.01 bar hHv ,sat
O 25 C

H 2O

H 2O

However, the calculation of heat can be simplified by treating the water vapor as an ideal gas,
which is a reasonable assumption at low pressure. The enthalpies of ideal gases depend on
temperature only. Therefore, the enthalpy of the vapor change due to the pressure change is
zero. Furthermore, enthalpy is weakly dependent on pressure in liquids. The leg of the
hypothetical path containing the pressure change of the liquid can be neglected. This leaves
v
25 C, 3.17 kPa hHl 2 O 25 C, 3.17 kPa
q hH
O
2

From the steam tables:


kJ
v, sat

hH
2547
.
2
kg
O
2

kJ
l , sat
hH

104
.
87
kg
O
2

(sat. H2O vapor at 25 C)


(sat. H2O liquid at 25 C)

which upon substitution gives


kJ
q 2442.3
kg

Therefore,

kJ
Q 0.5 kg 2442.3 1221.2 kJ
kg

To find the efficiency of the drying process, assume the dryer draws 30 A at 208 V and takes 20
minutes (1200 s) to dry the towel. From the definition of electrical work,
W IVt 30 A 208 V 1200 s 7488 kJ

Therefore, the efficiency is


1221.2 kJ

100 %
100 % 16.3%
W

7488 kJ

There are a number of ways to improve the drying process. A few are listed below.
Dry the towel outside in the sun.
Use a smaller volume dryer so that less air needs to be heated.
Dry more than one towel at a time since one towel cant absorb all of the available
heat. With more towels, more of the heat will be utilized.

2.3
In answering this question, we must distinguish between potential energy and internal energy.
The potential energy of a system is the energy the macroscopic system, as a whole, contains
relative to position. The internal energy represents the energy of the individual atoms and
molecules in the system, which can have contributions from both molecular kinetic energy and
molecular potential energy. Consider the compression of a spring from an initial uncompressed
state as shown below.

uncompressed

compressed
x

State 1

State 2

Since it requires energy to compress the spring, we know that some kind of energy must be
stored within the spring. Since this change in energy can be attributed to a change of the
macroscopic position of the system and is not related to changes on the molecular scale, we
determine the form of energy to be potential energy. In this case, the springs tendency to restore
its original shape is the driving force that is analogous to the gravity for gravitational potential
energy.
This argument can be enhanced by the form of the expression that the increased energy takes. If
we consider the spring as the system, the energy it acquires in a reversible, compression from its
initial uncompressed state may be obtained from an energy balance. Assuming the process is
adiabatic, we obtain:

E Q W W
We have left the energy in terms of the total energy, E. The work can be obtained by integrating
the force over the distance of the compression:

W F dx kxdx

1 2
kx
2

Hence:
E

1 2
kx
2

We see that the increase in energy depends on macroscopic position through the term x.
It should be noted that there is a school of thought that assigns this increased energy to internal
energy. This approach is all right as long as it is consistently done throughout the energy
balances on systems containing springs.

2.4
For the first situation, let the rubber band represent the system. In the second situation, the gas is
the system. If heat transfer, potential and kinetic energy effects are assumed negligible, the
energy balance becomes
U W

Since work must be done on the rubber band to stretch it, the value of the work is positive. From
the energy balance, the change in internal energy is positive, which means that the temperature
of the system rises.
When a gas expands in a piston-cylinder assembly, the system must do work to expand against
the piston and atmosphere. Therefore, the value of work is negative, so the change in internal
energy is negative. Hence, the temperature decreases.
In analogy to the spring in Problem 2.3, it can be argued that some of the work imparted into the
rubber band goes to increase its potential energy; however, a part of it goes into stretching the
polymer molecules which make up the rubber band, and the qualitative argument given above
still is valid.

2.5
To explain this phenomenon, you must realize that the water droplet is heated from the bottom.
At sufficiently high temperatures, a portion of the water droplet is instantly vaporized. The
water vapor forms an insulation layer between the skillet and the water droplet. At low
temperatures, the insulating layer of water vapor does not form. The transfer of heat is slower
through a gas than a liquid, so it takes longer for the water to evaporate at higher temperatures.

2.6

Apartment

Surr.
HOT
System

Fridge

+
W -

If the entire apartment is treated as the system, then only the energy flowing across the apartment
boundaries (apartment walls) is of concern. In other words, the energy flowing into or out of the
refrigerator is not explicitly accounted for in the energy balance because it is within the system.
By neglecting kinetic and potential energy effects, the energy balance becomes

U Q W
The Q term represents the heat from outside passing through the apartments walls. The W term
represents the electrical energy that must be supplied to operate the refrigerator.
To determine whether opening the refrigerator door is a good idea, the energy balance with the
door open should be compared to the energy balance with the door closed. In both situations, Q
is approximately the same. However, the values of W will be different. With the door open,
more electrical energy must be supplied to the refrigerator to compensate for heat loss to the
apartment interior. Therefore,
Wajar Wshut

where the subscript ajar refers the situation where the door is open and the subscript shut
refers to the situation where the door is closed. Since,
Qajar Qshut
U ajar Qajar Wajar U shut Qshut Wshut
Tajar Tshut

The refrigerator door should remain closed.

2.7
The two cases are depicted below.
Heater off

Heater on

Surr.

Surr.

System

System

Heater

Heater

Lets consider the property changes in your house between the following states. State 1, when
you leave in the morning, and state, the state of your home after you have returned home and
heated it to the same temperature as when you left. Since P and T are identical for states 1 and 2,
the state of the system is the same and U must be zero, so

U Q W 0
or

Q W
where -Q is the total heat that escaped between state 1 and state 2 and W is the total work that
must be delivered to the heater. The case where more heat escapes will require more work and
result in higher energy bills. When the heater is on during the day, the temperature in the system
is greater than when it is left off. Since heat transfer is driven by difference in temperature, the
heat transfer rate is greater, and W will be greater. Hence, it is cheaper to leave the heater off
when you are gone.

2.8
The amount of work done at constant pressure can be calculated by applying Equation 2.57

H Q
Hence,

H Q mh
where the specific internal energy is used in anticipation of obtaining data from the steam tables.
The mass can be found from the known volume, as follows:
3

1.0 kg

m3
0.0010

kg

1L 0.001 m

As in Example 2.2, we use values from the saturated steam tables at the same temperature for
subcooled water at 1 atm. The specific enthalpy is found from values in Appendix B.1:
kJ
kJ
kJ
u ul ,2 at 100 o C ul ,1 at 25 o C 419.02 104.87 314.15
kg
kg
kg

Solve for heat:

kJ
Q mu 1.0 kg 314.05 314.15 kJ
kg

and heat rate:

Q
Q
t

314.15 kJ
0.52 kW
60 s
10 min.

min

This value is the equivalent of five strong light bulbs.

2.9
(a)
From Steam Tables:

kJ
u1 2967.8
kg
kJ
u 2 2659.8
kg

(100 kPa, 400 C)


(50 kPa, 200 C)

kJ
u u 2 u1 308.0
kg
(b)
From Equations 2.53 and 2.63
T2

T2

T1

T1

u u 2 u1 cv dT

c P R dT

From Appendix A.2


c P R( A BT CT 2 DT 2 ET 3 )
T2

u R A BT CT 2 DT 2 ET 3 1 dT
T1

Integrating

E
1 1
B
C
u R ( A 1)(T2 T1 ) (T2 2 T12 ) (T23 T13 ) D( ) (T2 4 T14 )
2
3
4
T2 T1

The following values were found in Table A.2.1


A 3.470
B 1.45 10 3
C 0
D 1.21 10 4
E 0
Substituting these values and using

10

J
R 8.314
mol K
T1 (400 273.15 K) 673.15 K
T2 (200 273.15 K) 473.15 K

provides
J
u 5551
mol
kJ

J 1 [mol H 2 O] 1000 g 1 kJ

u 5551
308
.
1

kg

mol 18.0148 [g H 2 O] 1 kg 1000 J

The values in parts (a) and (b) agree very well. The answer from part (a) will serve as the basis
for calculating the percent difference since steam table data should be more accurate.
% Difference

308 308.1
100 % 0.03%
308.0

11

2.10
(a)
Referring to the energy balance for closed systems where kinetic and potential energy are
neglected, Equation 2.30 states

U Q W
(b)
Since internal energy is a function of temperature only for an ideal gas (Equation 2.4) and the
process is isothermal
U 0

According to Equation 2.77


P
W nRT ln 2
P1

P
n1 RT1 ln 2

P1

From the ideal gas law:


n1RT1 P1V1
P
W P1V1 ln 2
P1

Substitution of the values from the problem statement yields

5 bar
W 8 105 Pa 2.5 10 3 m 3 ln

8 bar
W 940 J
The energy balance is
0 J Q W

Q 940 J
(c)
Since the process is adiabatic

Q0
The energy balance reduces to
U W

12

The system must do work on the surroundings to expand. Therefore, the work will be negative
and
U 0
T2

U n cv T 0
T1

T2 T1
T2 will be less than 30 C

13

2.11
(a)
(i).
1

Path B

Pa
A
th

P [bar]

2
1

0.01

0.02

0.03
3
v [m /mol]

(ii).
Since internal energy is a function of temperature only for an ideal gas (Equation 2.4) and the
process is isothermal
u 0

Equation 2.48 states that enthalpy is a function of temperature only for an ideal gas. Therefore,
h 0

Performing an energy balance and neglecting potential and kinetic energy produces
u q w 0

For an isothermal, adiabatic process, Equation 2.77 states


P
W nRT ln 2
P1

or
w

P
W
RT ln 2
n
P1

Substituting the values from the problem statement gives

14


1 bar
J
w 8.314
(88 273.15) K ln

3 bar
mol K

J
w 3299
mol

Using the energy balance above

J
q w 3299
mol
(b)
(i).

See path on diagram in part (a)

(ii).
Since the overall process is isothermal and u and h are state functions
u 0
h 0

The definition of work is

w PE dv
During the constant volume part of the process, no work is done. The work must be solved for
the constant pressure step. Since it is constant pressure, the above equation simplifies to
w PE dv PE (v2 v1 )
The ideal gas law can be used to solve for v 2 and v1

J mol
8.314
(88 273.15) K
m3
RT2
K

v2
0
.
030

P2
1 10 5 Pa
mol

J mol
8.314
(88 273.15) K
m3
RT1
K

0
.
010
v1

P1
3 10 5 Pa
mol

Substituting in these values and realizing that PE P1 since the process is isobaric produces

15


m3
m3
w (3 10 5 Pa) 0.030
0.010

mol
mol

J
w 6000
mol

Performing an energy balance and neglecting potential and kinetic energy results in

u q w 0
J
q w 6000
mol

16

2.12
First, perform an energy balance. No work is done, and the kinetic and potential energies can be
neglected. The energy balance reduces to

U Q
We can use Equation 2.53 to get
T2

Q n cv dT
T1

which can be rewritten as


T2

Q n c P dT
T1

since the aluminum is a solid. Using the atomic mass of aluminum we find
n

5 kg
185.3 mol
kg
0.02698
mol

Upon substitution of known values and heat capacity data from Table A.2.3, we get

J
Q 185.3 mol 8.314
2.486 1.49 10 3 T dT

mol
K

294.15 K

Q 131.61 kJ

323.15 K

17

2.13
First, start with the energy balance. Potential and kinetic energy effects can be neglected.
Therefore, the energy balance becomes

U Q W
The value of the work will be used to obtain the final temperature. The definition of work
(Equation 2.7) is
V2

W PE dV
V1

Since the piston expands at constant pressure, the above relationship becomes
W PE V2 V1

From the steam tables


m3
v1 0.02641

kg

(10 MPa, 400 C)

m3
3

V1 m1v1 (3 kg) 0.02641


0.07923 m

kg

Now V2 and v2 are found as follows


V2 V1
V
v2 2
m2

748740 J
W
0.4536 m3
0.07923 m3
6
PE
2.0 10 Pa
m3
0.4536 m 3

0.1512

3 kg
kg

Since v2 and P2 are known, state 2 is constrained. From the steam tables:
T2 400 C

3
20 bar, 0.1512 m

kg

Now U will be evaluated, which is necessary for calculating Q . From the steam tables:

18

kJ
u 2 2945.2
kg

3
20 bar, 0.1512 m

kg

kJ
u1 2832.4
kg

100 bar, 400 C

kJ
kJ
U m1 u 2 u1 3 kg 2945.2 2832.4 338.4 kJ
kg
kg

Substituting the values of U and W into the energy equation allows calculation of Q

Q U W
Q 338400 [J] 748740 J 1.09 10 6 J

19

2.14
In a reversible process, the system is never out of equilibrium by more than an infinitesimal
amount. In this process the gas is initially at 2 bar, and it expands against a constant pressure of
1 bar. Therefore, a finite mechanical driving force exists, and the process is irreversible.

To solve for the final temperature of the system, the energy balance will be written. The pistoncylinder assembly is well-insulated, so the process can be assumed adiabatic. Furthermore,
potential and kinetic energy effects can be neglected. The energy balance simplifies to
U W

Conservation of mass requires


n1 n2
Let n n1 n2

The above energy balance can be rewritten as


T2

V2

T1

V1

n cv dT PE dV

Since cv and PE are constant:


ncv T2 T1 PE V2 V1
V2 and T1 can be rewritten using the ideal gas law

nRT2
P2
PV
T1 1 1
nR

V2

Substituting these expressions into the energy balance, realizing that PE P2 , and simplifying
the equation gives
5

P2 P1 V1
2
T2
7
nR
2
Using the following values

20

P1 2 bar
P2 1 bar

V1 10 L

n 1.0 mol
L bar
R 0.08314
mol K

results in
T2 206 K

To find the value for work, the energy balance can be used
W U ncv T2 T1
Before the work can be calculated, T1 must be calculated
PV
T1 1 1
nR

2 bar 10 L
1 mol 0.08314 L bar
mol K

241 K

Using the values shown above


W 727 J

21

2.15
The maximum work can be obtained through a reversible expansion of the gas in the piston.
Refer to Section 2.3 for a discussion of reversible processes. The problem states that the piston
assembly is well-insulated, so the heat transfer contribution to the energy balance can be
neglected, in addition to potential and kinetic energy effects. The energy balance reduces to
U W

In this problem, the process is a reversible, adiabatic expansion. For this type of process,
Equation 2.90 states
W

1
P2V2 P1V1
k 1

From the problem statement (refer to problem 2.13),


P1 2 bar

V1 10 L

P2 1 bar

To calculate W, V2 must be found. For adiabatic, reversible processes, the following


relationship (Equation 2.89) holds:
PV k const
where k is defined in the text. Therefore,
1
P
k
V2 1 V1k
P2

c
7
Noting that k P and substituting the proper values provides
cv 5
V2 16.4 L

Now all of the needed values are available for calculating the work.
W 9 L bar 900 J

From the above energy balance,


U 900 J

22

The change in internal energy can also be written according to Equation 2.53:
T2

U n cv dT
T1

Since cv is constant, the integrated form of the above expression is


5
U n R T2 T1
2
Using the ideal gas law and knowledge of P1 and V1 ,
T1 240.6 K

and
T2 197.3 K

The temperature is lower because more work is performed during the reversible expansion.
Review the energy balance. As more work is performed, the cooler the gas will become.

23

2.16
Since the vessel is insulated, the rate of heat transfer can be assumed to be negligible.
Furthermore, no work is done on the system and potential and kinetic energy effects can be
neglected. Therefore, the energy balance becomes

u 0
or
u 2 u1

From the steam tables


kJ
u1 2619.2
kg
kJ
u 2 2619.2
kg

(200 bar, 400 C)

The values of u 2 and P2 constrain the system. The temperature can be found from the steam
tables using linear interpolation:
T2 327.5 C

kJ
100 bar , u 2 2619.2

kg

Also at this state,


m3
v2 0.02012

kg
Therefore,

m3
3
Vvessel mv2 1.0 kg 0.02012
0.020 m

kg

24

2.17

Let the entire tank represent the system. Since no heat or work crosses the system boundaries,
and potential and kinetic energies effects are neglected, the energy balance is
u 0

Since the tank contains an ideal gas


T2 T1 0
T2 T1 300 K
The final pressure can be found using a combination of the ideal gas law and conservation of
mass.
T1
T
2
P1V1 P2V2
We also know
V2 2V1

Therefore,
P
P2 1 5 bar
2

25

2.18
(a)
First, as always, simplify the energy balance. Potential and kinetic energy effects can be
neglected. Therefore, the energy balance is

U Q W
Since, this system contains water, we can the use the steam tables. Enough thermodynamic
properties are known to constrain the initial state, but only one thermodynamic property is
known for the final state: the pressure. Therefore, the pressure-volume relationship will be used
to find the specific volume of the final state. Since the specific volume is equal to the molar
volume multiplied by the molecular weight and the molecular weight is constant, the given
expression can be written
Pv1.5 const
This equation can be used to solve for v2 .
1

P
1.5
v2 1 v11.5
P2

Using
P1 20 bar
v1

m3
1.0 m 3
0.1
10 kg
kg

P2 100 bar

gives
m3
v2 0.0342

kg
Now that the final state is constrained, the steam tables can be used to find the specific internal
energy and temperature.
T2 524.7 K
kJ
u 2 3094.6
kg

To solve for the work, refer to the definition (Equation 2.7).


26

V2

W PE dV
V1

or
v 2

w PE dv
v1

Since the process is reversible, the external pressure must never differ from the internal pressure
by more than an infinitesimal amount. Therefore, an expression for the pressure must be
developed. From the relationship in the problem statement,
Pv1.5 P1v11.5 const
Therefore, the expression for work becomes
v 2

v1

P1v11.5
v1.5

dv P1v11.5

v 2

v1.5 dv

v1

Integration and substitution of proper values provides


bar m 3
kJ
w 2.840
284
kg
kg

kJ
W 10 kg 284 2840 kJ
kg

A graphical solution is given below:

P [bar]

2
100

60

20

Work
0.02

0.06

27

0.10
v [m 3/kg]

To solve for Q , U must first be found, then the energy balance can be used.

kJ
kJ
U mu 2 u1 10 kg 3094.6 2602.8 4918 kJ
kg
kg

Now Q can be found,


Q U W 4918 kJ 2840 kJ 2078 kJ

(b)
Since the final state is the same as in Part (a), U remains the same because it is a state function.
The energy balance is also the same, but the calculation of work changes. The pressure from the
weight of the large block and the piston must equal the final pressure of the system since
mechanical equilibrium is reached. The calculation of work becomes:
v 2

W mPE dv
v1

All of the values are known since they are the same as in Part (a), but the following relationship
should be noted
PE P2

Substituting the appropriate values results in


W 6580 kJ

Again we can represent this process graphically:

P [bar]

2
100

Work
60

20

0.02

0.06

28

0.10
v [m 3/kg]

Now Q can be solved.


Q U W 4918 kJ 6580 kJ 1662 kJ

(c)
This part asks us to design a process based on what we learned in Parts (a) and (b). Indeed, as is
characteristic of design problems there are many possible alternative solutions. We first refer to
the energy balance. The value of heat transfer will be zero when
U W

For the same initial and final states as in Parts (a) and (b),
W U 4918 kJ

There are many processed we can construct that give this value of work. We show two
alternatives which we could use:
Design 1:
If the answers to Part (a) and Part (b) are referred to, one can see that two steps can be used: a
reversible compression followed by an irreversible compression. Let the subscript i" represent
the intermediate state where the process switches from a reversible process to an irreversible
process. The equation for the work then becomes
vi

1
1
.
5

W m P2 (v2 vi ) P1v1
dv 4918000 J
1
.
5

v1 v

Substituting in known values (be sure to use consistent units) allows calculation of vi :
m3
vi 0.0781

kg
The pressure can be calculated for this state using the expression from part (a) and substituting
the necessary values.
1.5

v
Pi P1 1
vi
Pi 29.0 bar

29

Now that both Pi and vi are known, the process can be plotted on a P-v graph, as follows:

P [bar]

2
100

Work

60

20

0.02

0.06

0.10
v [m 3/kg]

Design 2:
In an alternative design, we can use two irreversible processes. First we drop an intermediate
weight on the piston to compress it to an intermediate state. This step is followed by a step
similar to Part (b) where we drop the remaining mass to lead to 100 bar external pressure. In this
case, we again must find the intermediate state. Writing the equation for work:
W m Pi (vi v1 ) P2 (v2 vi ) 4918000 J
However, we again have the relationship:
v
Pi P1 1
vi

1.5

Substitution gives one equation with one unknown vi:


1.5

v1
W m P1 (vi v1 ) P2 (v2 vi ) 4918000 J

vi

There are two possible values vi to the above equation.


Solution A:
m3
vi 0.043
kg

30

which gives
Pi 70.8 bar

This solution is graphically shown below:

P [bar]

2
100

i
60

20

Work

0.06

0.10
v [m 3/kg]

0.02

Solution B
m3
vi 0.0762
kg

which gives
Pi 30.0 bar

This solution is graphically shown below:

P [bar]

2
100

60
i
20

Work

0.06

0.10
v [m 3/kg]

0.02

31

2.19
(a)
Force balance to find k:

Fspring=kx

Fatm=PatmA

Piston

Fmass=mg

Fgas=PgasA

mg

Pgas Patm A kx
A

since V=Ax

Pgas Patm A k2V


A
mg

since V is negative. Now solve:


2 10 5 Pa 1 10 5 Pa

2040 kg 9.81 m/s 2 k 0.02 m 3

0.1 m 2 2

0 .1 m 2

N
k 5.01 10 4
m

Work can be found graphically (see P-V plot) or analytically as follows: Substituting the
expression in the force balance above:
P
k V
W A
atm A A 2 dV
mg

V f

Vf

mg
kV

W A Patm
d V
dV
2
A

A
Vi
Vi

5.01 10 4 N/m 0.02 m 3 0 2


W A 3.00 10 Pa 0.03 0.05 m
2

2
0 .1 m 2

32

W A 5 103 J
-W

0.5kJ
= 10squares square
= 5 kJ

3
P [bar]

k V
2
A
2

mg
A

Work

1
1
Patm

0.01

0.03

0.05
V [m3 ]

(b)
You need to find how far the spring extends in the intermediate (int) position. Assume
PVn=const (other assumptions are o.k, such as an isothermal process, and will change the answer
slightly). Since you know P and V for each state in Part (a), you can calculate n.
Pi
Pf

V n
V i
or
f

10 5 Pa
2 10 5 Pa

0.03 m 3

3

0.05 m

n 1.35

Now using the force balance


mg

Pgas,int Patm A kV
A2
with the above equation yields:

P
V

Pint Pi V i
int

mg k Vint V i
atm A
A2

This last equality represents 1 equation. and 1 unknown (we know k), which gives
Vint 0.0385 m 3

Work can be found graphically (see P-V plot) or analytically using:

33

V int

Vf

Vi

Vint

W B Pgas dV PgasdV

expanding as in Part (a)


V

int
5 N
W B
2 10 m2 dV

Vi

Vint

V if

V i

Vint

Vint

f
6 N

5
V

10
d
V

3 10 mN2
5
dV
m

6
5 10 N5 V d V
m

Therefore,
WB 3.85 kJ

just like we got graphically.

P [bar]

2
2

mg
A
mg
A
1

Work
Patm

0.01

0.03

0.05
V [m3 ]

(c)
The least amount of work is required by adding differential amounts of mass to the piston. This
is a reversible compression. For our assumption that PVn = const, we have the following
expression:

WC , rev

Vf

Vf

Vi

Vi

const

Pgas dV V 1.35 dV

Calculate the constant from the initial state

34

const. 1 105 Pa 0.05 m 3

1.35 1.75 103

Therefore,
.03

WC,rev 1.75 103 dV


1.35
V

.05

10 V
1.750.35
3

0 .35 .03
.05

2800 J

P [bar]

2
2

1
1

Workrev
Patm

0.01

0.03

0.05
V [m3 ]

35

2.20
Before this problem is solved, a few words must be said about the notation used. The system
was initially broken up into two parts: the constant volume container and the constant pressure
piston-cylinder assembly. The subscript 1 refers to the constant volume container, 2 refers
the piston-cylinder assembly. i" denotes the initial state before the valve is opened, and f
denotes the final state.

To begin the solution, the mass of water present in each part of the system will be calculated.
The mass will be conserved during the expansion process. Since the water in the rigid tank is
saturated and is in equilibrium with the constant temperature surroundings (200 C), the water is
constrained to a specific state. From the steam tables,
kJ
v1l, i 0.001156
kg
kJ
v1v, i 0.12736
kg
kJ
u1l, i 850.64
kg
kJ
u1v, i 2595.3
kg

(Sat. water at 200 C)

P sat 1553.8 kPa


Knowledge of the quality of the water and the overall volume of the rigid container can be used
to calculate the mass present in the container.

V1 0.05m1 v1l, i 0.95m1 v1v,i

Using the values from the steam table and V1 0.5 m 3 provides
m1 4.13 kg

Using the water quality specification,


m1v 0.95m1 3.92 kg
m1l 0.05m1 0.207 kg

For the piston-cylinder assembly, both P and T are known. From the steam tables

36

m3
v2, i 0.35202

kg
kJ
u 2, i 2638.9
kg

(600 kPa, 200 C)

Enough information is available to calculate the mass of water in the piston assembly.
V
m2 2 0.284 kg
v2
Now that the initial state has been characterized, the final state of the system must be determined.
It helps to consider what physically happens when the valve is opened. The initial pressure of
the rigid tank is 1553.8 kPa. When the valve is opened, the water will rush out of the rigid tank
and into the cylinder until equilibrium is reached. Since the pressure of the surroundings is
constant at 600 kPa and the surroundings represent a large temperature bath at 200 C, the final
temperature and pressure of the entire system will match the surroundings. In other words,
kJ
u f u 2, i 2638.9
kg

(600 kPa, 200 C)

Thus, the change in internal energy is given by

U (m2 m1v, i m1l, i )u f m2 u 2, i m1v, i u1v, i m1l, i u1l, i


Substituting the appropriate values reveals
U 541.0 kJ

To calculate the work, we realize the gas is expanding against a constant pressure of 600 kPa
(weight of the piston was assumed negligible). From Equation 2.7,
Vf

W PE dV PE (V f Vi )
Vi

where
PE 600000 Pa

V f (m2 m1v, i m1l,i )v2, i 1.55 m 3

Vi 0.1 m 3 0.5 m 3 0.6 m 3

37

Note: v2, i was used to calculate V f because the temperature and pressure are the same
for the final state of the entire system and the initial state of the piston-cylinder assembly.
The value of W can now be evaluated.
W 570 kJ

The energy balance is used to obtain Q.


Q U W 541.0 kJ 570 kJ 1111 kJ

38

2.21

A sketch of the process follows:


system

boundary

system
Process

B
p 1,B = 20 bar
T1,B = 250 o C

A
p 1,A = 10 bar
T1,A = 700 o C
H2 O

boundary

H2 O

50 cm

A
p2
T2
H2 O

B
p2
T2
H2 O

10 cm

The initial states are constrained. Using the steam tables, we get the following:

p
T
v
u
V
m

V
v

State 1,A
10 [bar]
700 [oC]
0.44779 [m3/kg]
3475.35 [kJ/kg]
0.01 m3

State 1,B
20 [bar]
250 [oC]
0.11144 [m3/kg]
2679.58 [kJ/kg]
0.05 m3

0.11 [kg]

0.090 [kg]

All the properties in the final state are equal. We need two properties to constrain the system:
We can find the specific volume since we know the total volume and the mass:
v2

V1, A V1, B
m1, A m1, B

m3
0.06 m 3
0.30

0.20 kg
kg

We can also find the internal energy of state 2. Since the tank is well insulated, Q=0. Since it is
rigid, W=0. An energy balance gives:
U Q W 0

Thus,
U 2 U1 m1,A u1, A m1,Bu1,B
or
u2

kJ
U 2 m1, Au1, A m1, B u1, B

3121
m2
m1, A m1, B
kg

We have constrained the system with u2 and v2, and can find the other properties from the steam
Tables. Very close to
T2 = 500 [oC] and

P2 = 1200 [kPa]

Thus,

39


m3
V2, A v2, A m2, A 0.30
0.09 kg 0.267 m and

kg

x 0.267 0.1 0.167 m

40

2.22
We start by defining the system as a bubble of vapor rising through the can. We assume the
initial temperature of the soda is 5 oC. Soda is usually consumed cold; did you use a reasonable
estimate for T1? A schematic of the process gives:
State 2:
T 2= ?
P2= 1.01 bar

State 1:
T 1 = 278 K
P1 = 3 bar

where the initial state is labeled state 1, and the final state is labeled state 2. To find the final
temperature, we perform an energy balance on the system, where the mass of the system (CO2 in
the bubble) remains constant. Assuming the process is adiabatic and potential and kinetic energy
effects are negligible, the energy balance is
u w

Expressions for work and internal energy can be substituted to provide


cv T2 T1 PE dv PE v 2 v1
where cv = cP R. Since CO2 is assumed an ideal gas, the expression can be rewritten as
T

PT
T
cv T2 T1 PE R 2 1 R T2 2 1
P1
P2 P1

where the equation was simplified since the final pressure, P2, is equal to the external pressure,
PE. Simplifying, we get:

RP
R
T2 1 T1 1 2
c v P1
cv

or

RP c
T2 T1 1 2 v 237 K
cv P1 c P

41

2.23
The required amount of work is calculated as follows:
W P V

The initial volume is zero, and the final volume is calculated as follows:

4
4
V r 3 0.5 ft 3 1.54 ft 3 0.0436 m 3
3
3
Assuming that the pressure is 1 atm, we calculate that

W 1.01325 105 Pa 0.0436 m 3 0 m 3 4417 J


This doesnt account for all of the work because work is required to stretch the rubber that the
balloon is made of.

42

2.24
(a)
Since the water is at its critical point, the system is constrained to a specific temperature,
pressure, and molar volume. From Appendix B.1

m3
vc 0.003155

kg
Therefore,
m

vc

0.01 m 3
3.17 kg
m3
0.003155

kg

(b)
The quality of the water is defined as the percentage of the water that is vapor. The total volume
of the vessel can be found using specific volumes as follows

V m l v l m v v v 1 x mv l xm v v
where x is the quality of the water. To solve for the quality, realize that starting with saturated
water at a pressure of 1 bar constrains the water. From the steam tables,
m3
v v 1.6940

kg
m3
v l 0.001043

kg

(sat. H2O at P = 1 bar)

Now the quality can be found


x 0.00125

Thus, the quality of the water is 0.125%.


(c)
To determine the required heat input, perform an energy balance. Potential and kinetic energy
effects can be neglected, and no work is done. Therefore,

U Q

43

where

U mu 2 1 x mu1l xm u1v

From the steam tables


kJ
u 2 2029.58
kg
m3
l

u1 417.33

kg
m3
u1v 2506.1

kg

(H2O at its critical point)

(sat. H2O at P=1 bar)

Evaluation of the expression reveals


U 5102.6 kJ 5.10 10 6 J

44

2.25
(a)
Consider the air in ChE Hall to be the system. The system is constant volume, and potential and
kinetic energy effects can be neglected. Furthermore, disregard the work. The energy balance is

du
q
dt
since the temperature of the system changes over time. Using the given expression for heat
transfer and the definition of dU , the expression becomes
cv dT
hT Tsurr
dt
We used a negative sign since heat transfer occurs from the system to the surroundings. If cv is
assumed constant, integration provides
cv ln T Tsurr ht C
where C is the integration constant. Therefore,
T Tsurr C1e

h
t
cv

where C1 is a constant. Examining this equation reveals that the temperature is an exponential
function of time. Since the temperature is decreasing, we know that the plot of temperature vs.
time shows exponential decay.

Temperature

T0

Tsurr
time

45

(b)
Let time equal zero at 6 PM, when the steam is shut off. At 6 PM, the temperature of the hall is
22 C. Therefore,
T Tsurr C1e

h
t
c
v

22 C 2 C C1e (0)
C1 20 C
After 10 PM, ( t 4 hr ), the temperature is 12 C.
12 C 2 C 20 C e
h
0.173 hr -1
cv

( 4 hr)
c

At 6 AM, t 12 hr . Substitution of this value into the expression for temperature results in
T 4 .5 C

46

2.26
The gas leaving the tank does flow work as it exits the valve. This work decreases the internal
energy of the gas lowering the temperature. During this process, water from the atmosphere
will become supersaturated and condense. When the temperature drops below the freezing point
of water, the water forms a solid.

Attractive interactions between the compressed gas molecules can also contribute to this
phenomena, i.e., it takes energy to pull the molecules apart as they escape; we will learn more of
these interactions in Chapter 4.

47

2.27
Mass balance

dm
m in m out m in
dt
Separating variables and integrating:
m2

m1

dm m in dt

or
t

m2 m1 m in dt
0

Energy balance
Since the potential and kinetic energy effects can be neglected, the open system, unsteady state
energy balance is
dU

m out hout m in hin Q W s


dt sys out
in

The process is adiabatic and no shaft work is done. Furthermore, there is only one inlet stream
and not outlet stream. Therefore, the energy balance simplifies to
dU

dt

m in hin
sys

The following math is performed


U2

U1

dU m in hin dt hin m in dt

U 2 U 1 m2 u 2 m1u1 m2 m1 hin
where the results of the mass balance were used. Both m2 and m1 can be calculated by dividing
the tank volume by the specific volume

48

m2

V
v2

m1

V
v1

Substitution of these relationships and simplification results in

u 2 hin u1 hin
v2

v1

From the steam tables:


kJ
u1 2583.6
kg
m3
v1 0.19444

kg

kJ
hin 3177.2
kg

(sat. H2O vapor at 1 MPa)

(6 MPa, 400 C)

There are still two unknowns for this one equation, but the specific volume and internal energy
are coupled to each other. To solve this problem, guess a temperature and then find the
corresponding volume and internal energy values in the steam tables at 6 MPa. The correct
temperature is the one where the above relationship holds.
T 600 C : Expression = 4427.6
T 500 C : Expression = 1375.9
T 450 C : Expression = -558.6

Interpolation between 500 C and 450 C reveals that the final temperature is
T2 464.4 C

49

2.28
We can pick room temperature to be 295 K

Tin T1 295 K
Mass balance
dn
n in n out n in
dt
Separating variables and integrating:
n2

n1

dn nin dt

or
t

n2 n1 nin dt
0

Energy balance
Neglecting ke and pe, he unsteady energy balance, written in molar units is written as:
dU
nin hin nout hout Q W

dt
sys

The terms associated with flow out, heat and work are zero.
dU
nin hin

dt sys

Integrating both sides with respect to time from the initial state where the pressure is 10 bar to
the final state when the tank is at a pressure of 50 bar gives:
U2

U1

dU nin hin dt hin nin dt

since the enthalpy of the inlet stream remains constant throughout the process. Integrating and
using the mass balance above:
n2u2 n1u1 n2 n1 hin

50

Now we do some math:


n2u 2 n1u1 n2 n1 hin
n2 u 2 hin n1 u1 hin
By the definition of h
hin uin Pin vin uin RTin u1 RT1
so
n2 u 2 u1 n2 RT1 n1 u1 u1 n1R T1
n2 cv T2 T1 n2 RT1 n1RT1
Since cv c P R

n2

3
R
2

3
T2 T1 n2T1 n1T1
2

or
3n2T2 5n2T1 2n1T1

dividing by n1:
n
n
3 2 T2 5 2 T1 2T1
n1
n1
Using the ideal gas law:
n2 P2T1

n1 P1T2
so
P T
P T
3 2 1 T2 5 2 1 T1 2T1
P1T2
P1T2

or

51

P T
5 2 1
P1 434 [K]
T2
P2
3 2
P1
(b) Closed system

u q w q
u

cv
T2 T1 5R T2 T1 28.9 kJ
MW
2MW
kg

kJ
q 28.9
kg
(c)

P2T2 P3T3
PT
P3 2 2 34 bar
T3

52

2.29
Mass balance

dn
n in n out n in
dt
Separating variables and integrating:
n2

n1 0

dn nin dt

or
t

n2 nin dt
0

Energy balance
Neglecting ke and pe, the unsteady energy balance, in molar units, is written as:
dU

nin hin nout hout Q W


dt

sys
The terms associated with flow out and heat are zero.

dU

nin hin W
dt sys
Integrating both sides with respect to time from the empty initial state to the final state gives:
U2

U1

dU nin hin dt W dt hin nin dt W hin n2 W

since the enthalpy of the inlet stream remains constant throughout the process. The work is
given by:

W n2 Pext (v2 v1 ) n2 Pext v2

53

n2u2 n2 hin Pext v2


Rearranging,

u 2 hin Pext v2 uin Pin vin Pext v2


u 2 uin Pin vin Pext v2
P T
cv T2 Tin RTin R ext 2
P2

so
T2

cv R

P
cv ext R
P2

Tin 333 K

54

2.30
valve maintains
pressure in system
constant
v
T1 = 200 oC
x1 = 0.4
V = 0.01 m3
l

Mass balance
dm
m in m out m out
dt
Separating variables and integrating:
m2

m1

dm m out dt

or
t

m2 m1 m out dt
0

Energy balance
dU
m out hout Q

dt

sys
Integrating
m 2 u 2

m1u1

dU m out hout Q dt hout m out dt Q dt

Substituting in the mass balance and solving for Q


Q m2u2 m1u1 m2 m1 hout

55

We can look up property data for state 1 and state 2 from the steam tables:
m3
v1 (1 x)v f xvg 0.6 .001 0.4 0.1274 0.051
kg
m3
v2 0.1274
kg

So the mass in each state is:

m1

0.01 m 3
V1
0.196 kg

v1
m3
0.051
kg

m2

V2

v2

0.01 m 3
0.0785 kg
m3
0.1274
kg

m2 m1 0.1175 kg

And for energy and enthalpy

kJ
u1 (1 x)u f xu g 0.6 850.64 0.4 2597.5 1549
kg
kJ
u 2 2595.3
kg
kJ
hout 2793.2
kg
Solving for heat, we get
Q m2u2 m1u1 m2 m1 hout 228 kJ

56

2.31
Consider the tank as the system.

Mass balance
dm
m in m out m in
dt
Separating variables and integrating:
m2

m1

dm m in dt

or
t

m2 m1 m in dt
0

Energy balance
Since the potential and kinetic energy effects can be neglected, the open system, unsteady state
energy balance is
dU

m out hout m in hin Q W s


dt sys out
in

The process is adiabatic and no shaft work is done. Furthermore, there is only one inlet stream
and not outlet stream. Therefore, the energy balance simplifies to
dU

m in hin
dt sys

The following math is performed


U2

U1 0

dU m

h dt hin m in dt

in in

U 2 m2 u 2 m2 hin
where the results of the mass balance were used. Thus,

u 2 hin
From the steam tables

57

kJ
u 2 3632.5
kg

(9 MPa, 800 C)

so

kJ
hin 3632.5
kg
We can use the value of hin and the fact that the steam in the pipe is at 9 MPa to find the
temperature.

Tin 600 C

58

2.32
(a)
First, the energy balance must be developed. Since the problem asks how much energy is stored
in the battery after 10 hours of operation, the process is not steady-state. Let the battery be the
system. Potential and kinetic energy effects can be neglected. Furthermore, heating of the
battery as it is charged can be ignored. The energy balance is
dU

Q W s
dt

sys

No shaft work is performed, but electrical is supplied to the battery, which must be accounted for
in W s . The value of Q is given explicitly in the problem statement. Both of these values remain
constant over time, so integration provides

U Q W s t
From the problem statement
W s 5 kW
Q 1 kW
t 36000 s

Substituting these values allows the calculation of the amount of energy stored:
U 144,000 kJ 144 MJ

(b)
To calculate the velocity of the falling water, an energy balance must be developed with the
water passing through the electricity generator (probably a turbine) as the system, where the

water enters with a velocity V1 and leaves with a negligible velocity, which will be approximated
as 0. Assume that potential energy changes can be neglected. Furthermore, assume that the
temperature of the water does not change in the process, so the change in internal energy is zero.
Also, view the process as adiabatic. The energy balance reduces to

E K W river
where W river is the power of the flowing water. The actual power being provided by the stream
can be calculated using the efficiency information. Let represent the efficiency.

59

s
Wriver

W
5 kW
W river s
10 kW
0.5

The value of W river should be negative since the water is supplying work that is stored electrical
energy. Therefore, the energy balance becomes

E K 10000 W
This expression can be rewritten as

1 2 2
m V2 V1 10000 W
2
From the problem statement and the assumptions made,

kg
m 200
s

m
V2 0
s
Therefore,

m
V1 10
s
There are a number of reasons for the low conversion efficiency. A possible potential energy
loss inherent in the design of the energy conversion apparatus decreases the efficiency. Heat is
lost to the surroundings during conversion. Some of the energy is also lost due to friction (drag)
effects.

60

2.33
Considering the turbine to be the system, rearrangement of the steady-state, open system energy
balance provides

nout (h ek e p )out nin (h ek e p )in Q W s


out

in

Performing a mass balance reveals


n in n1 n out n 2
Assuming the rate of heat transfer and potential energy effects are negligible and realizing that
there is one inlet and one outlet allows the simplification of the above equation to
W s n 2 h2 h1 e K , 2 e K ,1

h2 h1 can be rewritten using Equations 2.58 and Appendix A.2

h2 h1 c p dT R A BT CT 2 DT 2 ET 3 dT
T2
T1

Since the quantity ek ,2 ek ,1 is multiplied by n , it is rewritten as follows for dimensional


homogeneity

k ,2

2 2
1
ek ,1 ( MW ) air V2 V1
2

To solve for n , the ideal gas law is used


P2V2 n 2 RT21
P V
n 2 2 2
RT2

To solve for the volumetric flow rate, the fluid velocity must be multiplied by the cross-sectional
area

D2 2V2

V2

4
The energy balance is now

61

D 2V T2
2 2
P
1
2
3
2
2
2
2

Ws
R A BT CT DT ET dT ( MW ) air V2 V1

RT2
4
2
T1

Substituting values from Table A.2.1 and the problem statement results in

W 4.84 10 6 [W] -4.84 [MW]

62

2.34
First, a sketch of the process is useful:

30 bar

20 bar

100 oC

150 oC

To find the heat in we will apply the 1st law. Assuming steady state, the open system energy
balance with one stream in and one stream out can be written:
0 n h1 h2 Q
which upon rearranging is:
Q
h2 h1
n
Thus this problem reduces to finding the change in the thermodynamic property, enthalpy from
the inlet to the outlet. We know 2 intensive properties at both the inlet and outlet so the values
for the other properties (like enthalpy!) are already constrained. From Table A.2.1, we have an
expression for the ideal gas heat capacity:
cp
R

1.424 14.394 10 3 T 4.392 10 6 T 2

with T in (K). Since this expression is limited to ideal gases any change in temperature must be
under ideal conditions. From the definition of heat capacity:

T2
Q
h2 h1 1.424 14.394 10 3 T 4.392 10 6 T 2 dT
n
T1

By integrating and substituting the temperatures, we obtain:


Q
J
5590
n
mol

63

2.35
A schematic of the process follows:
Ws
T1 = 350 oC

T1 = 308 oC
P2= 1 atm

v1 = 600 cm3/mol

To solve for W s / n we need a first law balance. With negligible eK and eP, the 1st law for a
steady state process becomes:
0 n h1 h2 Q W s
If heat transfer is negligible,
W s
h
n
We can calculate the change in enthalpy from ideal gas heat capacity data provided in the
Appendix.

T2
T2
W s
h c p dT R 1.213 28.785 10 3 T 8.824 10 6 T 2 dT
n
T1

T1

Integrate and evaluate:


W s
J
5358
n
mol

64

2.36
(a)
First start with the energy balance. Nothing is mentioned about shaft work, so the term can be
eliminated from the energy balance. The potential and kinetic energy effects can also be
neglected. Since there is one inlet and one outlet, the energy balance reduces to

Q n 2 h2 n1h1
A mass balance shows
n 2 n1

so the energy balance reduces to


Q n1 h2 h1
Using the expressions from Appendix A.1, the energy balance becomes
T2

Q n1R A BT CT 2 DT 2 ET 3 dT
T1

Using
A 3.376
B 0.557 10 3
C 0
D 0.031 105
E 0
J
R 8.314
mol K
mol
n1 20
s
T1 373.15 K
T2 773.15 K
gives
Q 245063 W 245.1 kW

65

(b)
To answer this question, think about the structure of n-hexane and carbon monoxide. N-hexane
is composed of 20 atoms, but carbon monoxide has two. One would expect the heat capacity to
be greater for n-hexane since there are more modes for molecular kinetic energy (translational,
kinetic, and vibrational). Because the heat capacity is greater and the rate of heat transfer is the
same, the final temperature will be less.

66

2.37
First start with the energy balance around the nozzle. Assume that heat transfer and potential
energy effects are negligible. The shaft work term is also zero. Therefore, the energy balance
reduces to
n 2 (h e K ) 2 n1 (h e K )1 0

A mass balance shows


n1 n 2

On a mass basis, the energy balance is

1
h2 h1 eK ,1 eK ,2 V12 V2 2
2

Since the steam outlet velocity is much greater than the velocity of the inlet, the above
expression is approximately equal to

1
h2 h1 V2 2
2

The change in enthalpy can be calculated using the steam tables.


J
h1 2827.9 103
kg
J
h2 2675.5 103
kg

(10 bar, 200 C)


(sat. H2O(v) at 100 kPa)

Therefore,

m
V2 552
s
To solve for the area, the following relationship is used

AV2
m
v2

From the steam tables

67

m3
v2 1.6940

kg
Now all but one variable is known.

A 3.07 10 3 m 2

68

2.38
First start with the energy balance around the nozzle. Assume that heat transfer and potential
energy effects are negligible. The shaft work term is also zero. Therefore, the energy balance
reduces to

n 2 (h ek ) 2 n1 (h ek )1 0
The molar flow rates can be eliminated from the expression since they are equal. Realizing that
e K ,2 e K ,1 since the velocity of the exit stream is much larger than the velocity of the inlet
stream simplifies the energy balance to
h2 h1 ek ,2
Using Appendix A.2 and the definition of kinetic energy
T2

1
h2 h1 R A BT CT 2 DT 2 ET 3 dT ( MW ) C 3 H 8 V22
2
T1

From Table A.2.1


A 1.213
B 28.785 10 3
C 8.824 10 6
D0
E 0
It is also important that the units for the molecular weight and universal gas constant are
consistent. The following values were used
J
R 8.314
mol K
kg
( MW ) C3 H 8 0.0441
mol
Integration of the above expression and then solving for T2 provides
T2 419.2 K

69

2.39
First start an energy balance around the diffuser. Assume that heat transfer and potential energy
effects are negligible. The shaft work term is also zero. The energy balance reduces to

n 2 (h ek ) 2 n1 (h ek )1 0
A mass balance reveals
n1 n 2

The molar flow rates can be eliminated from the expression. Using the definitions of enthalpy
and the kinetic energy, the equation can be rewritten as

T2

2 2
1
c
dT

(
MW
)
V
2 V1
P
air

T1

The temperature and velocity of the outlet stream are unknown, so another equation is needed to
solve this problem. From the conservation of mass,

P1V1 P1 A1V1
P2 A2V2

T1
T1
T2

where A2, the cross-sectional area of the diffuser outlet, is twice the area of the inlet. Therefore,

1 P T
V2 1 2 V1
2 P2 T1

Using Appendix A.2 and the above expression, the energy balance becomes
T2

R A BT CT DT
T1


1 P1 T2
1

ET dT ( MW ) air V1 V12
2 P2 T1

Substituting values from the problem statement provides an equation with one unknown:
T2 381 K

Therefore,
1 1 bar 381 K
m
300 m/s 111
V2

2 1.5 bar 343.15 K


s

70

2.40
To find the minimum power required for the compressor, one must look at a situation where all
of the power is used to raise the internal energy of the air. None of the power is lost to the
surroundings and the potential and kinetic energy effects must be neglected. Therefore, the
energy balance becomes

0 n1h1 n 2 h2 W s
Performing a mass balance reveals
n1 n 2

The energy balance reduces to

W s n1 h2 h1
Using Equation 2.58 and Appendix A.2, the equation becomes
T2

W s n1R A BT CT 2 DT 2 ET 3 dT
T1

Table A.2.1 and the problem statement provide the following values

A 3.355
B 0.575 10 3
C 0
D 1600
E 0
mol
n1 50
s
T1 300 K

To find the work, we still need T2. We need to pick a reasonable process to estimate T2. Since
the heat flow is zero for this open system problem, we choose an adiabatic, reversible piston
situation. For this situation,
PV k const.

Since we are assuming the air behaves ideally, we can rewrite the equation as

71

n RT
n RT
P1 1 1 P2 2 2
P1
P2

P11 k T1k P21 k T2k

Substituting values from the problem statement, we obtain

1 7 / 5

7 / 5 1 bar
T2 300 K

10 bar 1 7 / 5

5/7

579 K

Substitute this value into the expression for the work and evaluate:
W s 417.4 kW

72

2.41
(a)
Perform a mass balance:

n1 n 2 n out
Apply the ideal gas law:
P1V1 P2V2

n out
RT1 RT2
Substitute values from the problem statement:

1105 5 10 3 2 105 2.5 10 3 nout

8.314373.15
8.314293.15
n out 0.366 mol/s

(b)
No work is done on the system, and we can neglect potential and kinetic energy effects. We will
assume the process is also adiabatic. The energy balance reduces to

0 nin hin n out hout


in

out

n out hout n1h1 n 2 h2 n1 hout h1 n 2 hout h2 0


We can calculate the enthalpy difference from the given ideal heat capacity:

3.267 5.324 10

Tout

n1 R

373.15

3.267 5.324 10

Tout

T dT n 2 R

T dT 0

293.15

Again, we must calculate the molar flow rates from the ideal gas law. Upon substitution and
evaluation, we obtain
Tout 329 K

73

2.42
(a)
Since the temperature, pressure, and volumetric flow rate are given, the molar flow rate is
constrained by the ideal gas law.

Note: 1.67 10 8 m 3 /s 1 cm 3 / min


n

PV 1.0135 10 5 Pa 1.67 10 8 m 3 /s

7.45 10 7 mol/s
8.314 J/mol K 273.15 K
RT

To recap, we have shown


1 SCCM 7.45 10 7 mol/s
(b)
Assumptions:

N2 is an ideal gas
All power supplied by the power supply is transferred to the N2
Uniform temperature radially throughout sensor tube
Kinetic and potential energy effects negligible in energy balance

Let x represent the fraction of N2 diverted to the sensor tube, and n s represent the molar flow
rate through the sensor tube. Therefore, the total molar flow rate, ntotal , is
n
ntotal s
x
We can use temperature and heat load information from the sensor tube to find the molar flow
rate through the sensor tube. First, perform an energy balance for the sensor tube:
H n s hout hin Q
The enthalpy can be calculated with heat capacity data. Therefore,
n s

Q
T2

cP dT

T1

Now, we can calculate the total molar flow rate.

74

ntotal

T2

x c P dT
T1

To find the flow rate in standard cubic centimeters per minute, apply the conversion factor found
in Part (a)
Q

vtotal (SCCM )

T2

x c P dT

1 SCCM

7.45 10 7 mol/s

T1

(c)
To find the correction factor for SiH4, re-derive the expression for flow rate for SiH4 and then
divide it by the expression for N2 for the same power input, temperatures, and fraction of gas
diverted to the sensor tube.

Q
T2

Factor

vtotal , SiH 4
vtotal , N 2

x c P, SiH 4 dT

T1

Q
T2

x c P, N 2 dT

1 SCCM

7.45 10 7 mol/s

1 SCCM

7.45 10 7 mol/s

T2

c P, N

dT

T1
T2

c P, SiH

dT

T1

T1

If we assume that heat capacities are constant, the conversion factor simplifies:
Factor

c P, N 2
c P, SiH 4

Using the values in Appendix A.2.2 at 298 K, we get


Factor 0.67

75

2.43
(a)
It takes more energy to raise the temperature of a gas in a constant pressure cylinder. In both
cases the internal energy of the gas must be increased. In the constant pressure cylinder work,
Pv work must also be supplied to expand the volume against the surroundings pressure. This is
not required with a constant volume.
(b)
As you perspire, sweat evaporates from your body. This process requires latent heat which cools
you. When the water content of the environment is greater, there is less evaporation; therefore,
this effect is diminished and you do not feel as comfortable.

76

2.44
From the steam tables at 10 kPa:

T(K)

323.15

2592.6

373.15

2687.5

423.15

2783

473.15

2879.5

h vs. T
6000
y = 0.0003x

5000

523.15

2977.3

573.15

3076.5

4000

673.15

3279.5

3000

773.15

3489

873.15

3705.4

973.15

3928.7

1073.15

4159.1

1173.15

4396.4

1273.15

4640.6

1373.15

4891.2

1473.15

5147.8

1573.15

5409.7

+ 1.6241x + 2035.7
2

R =1

Series1

2000

Poly. (Series1)
1000
0
0

dh
cP
1.6241 0.0006624T
dT P

500

1000
T

kJ
J
kg K 3.516 0.001434T R mol K

Now compare the above values to those in Appendix A.2.


A

Steam Tables

3.516

0.001434

Appendix A

3.470

0.001450

% difference

1.3

1.4

77

1500

2000

2.45
For throttling devices, potential and kinetic energy effects can be neglected. Furthermore, the
process is adiabatic and no shaft work is performed. Therefore, the energy balance for one inlet
and one outlet is simplified to
n1 h1 n 2 h2

which is equivalent to
m 1 h1 m 2 h2
Since mass is conserved
h1 m 2 h2
From the steam tables:
kJ
h1 3398.3
kg
kJ
h2 3398.3
kg

(8 MPa, 500 C)

Now that we know u 2 and P2 , T2 is constrained. Linear interpolation of steam table data gives
T2 457 C

78

2.46
(a)
An expression for work in a reversible, isothermal process was developed in Section 2.7.
Equation 2.77 is
P
W nRT ln 2
P1

Therefore,
P
w RT ln 2
P1

Evaluating the expression with


J
R 8.314
mol K
T 300 K
P2 100 kPa
P1 500 kPa

gives
J
w 4014
kg
(b)
Equation 2.90 states

R
T2 T1
k 1

Since the gas is monatomic


5
cP R
2
3
cv R
2
and

79

5
3

T2 can be calculated by applying the polytropic relation derived for adiabatic expansions. From
Equation 2.89

PV k const
Pv k const
By application of the ideal gas law
RT
P
Since R is a constant, substitution of the expression for P into the polytropic relation results in
v

P 1 k T k const
P11 k T 1 k P2 1 k T2 k
This relation can be used to solve for T2 .
T2 157.6 K

Now that T2 is known, value of work can be solved.


J
w 1775.9
kg

80

2.47
(a)
The change in internal energy and enthalpy can be calculated using

h h l 1 atm, 100 C h l 1 atm, 0 C


u u l 1 atm, 100 C u l 1 atm, 0 C
We would like to calculate these values using the steam tables; however, the appendices dont
contain steam table data for liquid water at 0.0 C and 1 atm. However, information is provided
for water at 0.01 C and 0.6113 kPa. Since the enthalpy and internal energy of liquid water is
essentially independent of pressure in this pressure and temperature range, we use the steam
table in the following way
kJ
h l 1 atm, 100 C 419.02
kg
kJ
h l 1 atm, 0 C h l 0.6113 kPa , 0.01 C 0
kg
kJ
u l 1 atm, 100 C 418.91
kg
kJ
u l 1 atm, 0 C u l 0.6113 kPa , 0.01 C 0
kg
Therefore,
kJ
h 419.02
kg
kJ
u 418.91
kg

(b)
The change in internal energy and enthalpy can be calculated using

h h v 1 atm, 100 C h l 1 atm, 100 C


u u v 1 atm, 100 C u l 1 atm, 100 C
From the steam tables

81

kJ
h v 2676.0
kg
kJ
h l 419.02
kg
kJ
u v 2506.5
kg
kJ
u l 418.91
kg
Therefore,
kJ
h 2256.99
kg
kJ
u 2087.59
kg

The change in internal energy for the process in Part (b) is 5.11 times greater than the change in
internal energy calculated in Part (a). The change in enthalpy in Part (b) is 5.39 times greater
than the change in enthalpy calculated in Part (a).

82

2.48
To calculate the heat capacity of Ar, O2, and NH3 the following expression, with tabulated values
in Table A.2.1, will be used,

cP
A BT CT 2 DT 2 ET 3
R

where T is in Kelvin. From the problem statement


T 300 K

and from Table 1.1


J
R 8.314
mol K
To find the A-E values, Table A.2.1 must be referred to.
Formula
Ar
O2
NH3

A
3.639
3.5778

B 103
0.506
3.02

C 10 6
0
0

D 10 5
-0.227
-0.186

E 109
0
0

The values are not listed for Ar since argon can be treated as a monatomic ideal gas with a heat
capacity independent of temperature. The expression for the heat capacity is
5
c P, Ar R
2
Now that expressions exist for each heat capacity, evaluate the expressions for T 300 K .
J
c P, Ar 20.785
mol K
J
c P, O2 29.420
mol K
J
c P, NH 3 35.560
mol K
By examining the heat capacity for each molecule, it should be clear that the magnitude of the
heat capacity is directly related to the structure of the molecule.

83

Ar

O2

Since argon is monatomic, translation is the only mode through which the atoms can
exhibit kinetic energy.

Translation, rotation, and vibration modes are present. Since oxygen molecules are
linear, the rotational mode of kinetic energy contributes RT per mol to the heat capacity.

NH3

Translation, rotation, and vibration modes are present. Ammonia molecules are nonlinear, so the rotation mode contributes 3RT/2 per mole to the heat capacity.

The vibration contributions can also be analyzed for oxygen and ammonia, which reveals that the
vibration contribution is greatest for ammonia. This is due to ammonias non-linearity.

84

2.49
For a constant pressure process where potential and kinetic energy effects are neglected, the
energy balance is given by Equation 2.57:

Q H

The change in enthalpy can be written as follows


H mh2 h1

From the steam tables:


kJ
h2 hsat. vapor at 10 kPa 2584.6
kg
kJ
h1 hsat. liquid at 10 kPa 191.81
kg
Therefore,

kJ
kJ
Q 2 kg 2584.6 191.81
kg
kg

Q 4785.6 kJ

We can find the work from its definition:


Vf

W PE dV
Vi

The pressure is constant, and the above equation can be rewritten as follows
W PE mv2 v1

From the steam tables:


m3
v2 vsat. vapor at 10 kPa 14.674

kg
m3
v1 vsat. liquid at 10 kPa 0.00101

kg
Therefore,

85


m3

W 10000 Pa 2 kg 14.674 0.00101
293.5 kJ

kg
kg

86

2.50
First, perform an energy balance on the system. Potential and kinetic energy effects can be
neglected. Since nothing is mentioned about work in the problem statement, W can be set to
zero. Therefore, the energy balance is

Q U

Performing a mass balance reveals


m2 m1

where
m1 m1l m1v

Now the energy balance can be written as

Q m1 (u 2 u1 ) m1u 2 m1l u1l m1v u1v

Since two phases coexist initially (water is saturated) and P1 is known, state 1 is constrained.
From the saturated steam tables
kJ
u1l 191.79
kg
kJ
u1v 2437.9

(sat. H2O at 10 kPa)

kg

As heat is added to the system, the pressure does not remain constant, but saturation still exists.
One thermodynamic property is required to constrain the system. Enough information is known
about the initial state to find the volume of the container, which remains constant during heating,
and this can be used to calculate the specific volume of state 2.
l l
v v
V2 V1 m1 v1 m1 v1
v2

m2 m1
m1l m1v

From the saturated steam tables

87

m3
v1l 0.001010

kg
m3
v1l 14.674

kg

(sat. H2O at 10 kPa)

Therefore,
m3
v2 1.335

kg
The water vapor is now constrained. Interpolation of steam table data reveals
kJ
u 2 2514.6
kg
Now that all of the required variables are known, evaluation of the expression for Q is possible.
Q 11652 kJ

88

2.51
Let the mixture of ice and water immediately after the ice has been added represent the system.
Since the glass is adiabatic, no work is performed, and the potential and kinetic energies are
neglected, the energy balance reduces
H 0

We can split the system into two subsystems: the ice (subscript i) and the water (subscript w).
Therefore,
H mi hi mw hw 0
and
mi hi mw hw

We can get the moles of water and ice.

Vw
0.0004 m 3

0.399 kg
vw
m3
0.001003

kg

mw
0.399 kg 22.15 mol
nw

MW H 2O 0.0180148 kg
mol
mi
ni
5.55 mol
MW H 2O
mw

Now, lets assume that all of the ice melts in the process. (If the final answer is greater than 0
C, the assumption is correct.) The following expression mathematically represents the change
in enthalpy.

ni c P, i 0 10 C h fus c P, w T f 0 C nwc P, w T f 25 C
Note: Assumed the heat capacities are independent of temperature to obtain this
expression.

From Appendix A.2.3


c P ,i 4.196 R
c P ,w 9.069 R

and
kJ
h fus 6.0
mol

89

Substitution of values into the above energy balance allows calculation of Tf.
T f 3.12 C

(Our assumption that all the ice melts is correct.)


(b)
To obtain the percentage of cooling achieved by latent heat, perform the following calculation

Fractionlatent

ni h fus

nw c P, w T f Tw, i

5.55 mol 6000

mol

Fractionlatent
0.911
J
3.12 C 25 C
22.15 mol9.069 8.314
mol K
Percentlatent 91.1%

90

2.52
A mass balance shows
n2 n1l n1v

To develop the energy balance, neglect kinetic and potential energy. Also, no shaft work is
performed, so the energy balance becomes

U Q
The energy balance can be expanded to

Q n1l n1v u 2 n1l u1l n1v u1v

If the reference state is set to be liquid propane at 0 C and 4.68 bar, the internal energies become
u1l 0
u1v u vap 0 C
u 2 u vap 0 C

T2

c P R dT

273 K

Once the change in internal energy for vaporization and temperature of state 2 is determined, Q
can be solved. As the liquid evaporates, the pressure increases. At state 2, where saturated
propane vapor is present, the ideal gas law states
P2

RT2
v2

To find v2 , assume that v1v v1l . The volume of the rigid container is
V n1v v1v n1v

m3
RT1
0.00485

P1
mol

Therefore,
v2

m3
0.00243

n1l n1v
mol

91

Also, since the propane is saturated, P2 and T2 are not independent of each other. They are
related through the Antoine Equation,

ln P sat A
T

sat

where
P sat P2 and T sat T2

Substitution provides,
RT
B
ln 2 A
T2 C
v2

bar m 3
Using values from Table A.1.1 and R 8.314 10 5

mol K
T2 301.7 K

To find u vap , refer to the definition of enthalpy.


hvap h v h l (u Pv) v u Pv l
Since v v v l , the above the change in internal energy of vaporization can be written as
u vap h vap Pv v h vap RT
Therefore,
kJ
u vap 0 C 14.39
mol
Evaluation of the following equation after the proper values have been substituted from Table
A.2.1

301.7 K

Q n1l n1v u vap 0 C c P R dT n1l 0 n1v uvap 0 C

273 K

92

gives
Q 18.1 kJ

93

2.53
The equation used for calculating the heat of reaction is given in Equation 2.72. It states

hrxn
vi h f

This equation will be used for parts (a)-(e). Since the heat of reaction at 298 K is desired, values
from Appendix A.3 can be used.
(a)
First the stoichiometric coefficient must be determined for each species in the reaction.

vCH 4 ( g ) 1
vO2 ( g ) 1
vCO2 ( g ) 1
v H 2 O( g ) 1

From Tables A.3.1 and A.3.2


kJ
hf ,298 CH ( g ) 74.81 mol

kJ
hf ,298 O ( g ) 0 mol

kJ
hf ,298 CO ( g ) 393.51 mol

kJ
hf ,298 H O( g ) 241.82 mol

From Equation 2.72, the equation for the heat of reaction is

hrxn
,298 vCH 4 ( g ) h f ,298 CH ( g ) vO2 ( g ) h f ,298 O ( g ) vCO2 ( g ) h f ,298 CO ( g ) v H 2 O ( g ) h f ,298 H O ( g )
4
2
2
2

kJ
kJ
kJ
kJ

hrxn
0
393.51
241.82

, 298 74.81

mol mol
mol
mol

kJ

hrxn
, 298 560.52
mol
Now that a sample calculation has been performed, only the answers will be given for the remaining
parts since the calculation process is the same.

94

(b)

kJ

hrxn
, 298 604.53
mol
(c)

kJ

hrxn
, 298 206.12
mol
(d)

kJ

hrxn
, 298 41.15
mol
(e)

kJ

hrxn
, 298 905.38
mol

95

2.54
The acetylene reacts according to the following equation

C2H2(g) + (5/2)O2(g) 2CO2(g) + H2O(g)


(a)
First, choose a basis for the calculations.
nC 2 H 2 1 mol

Calculate the heat of reaction at 298 K using Equation 2.72 and Appendix A.3

hrxn
h f
C H

hrxn
vi h f
2

O 2hf CO hf H O

2.5 h f

hrxn
1.255 10 6
mol

H rxn,298 nC 2 H 2 hrxn
1.255 10 6 J

The required amount of oxygen is calculated as follows

nO 1 2.5(nC H
2

)1 2.5 mol

The compositions for both streams are


Streams
1 (Inlet)
2 (Outlet)

nC 2 H 2

nO2

nN 2

nCO2

nH 2O

1
0

2.5
0

0
0

0
2

0
1

D 10 5
-1.299
-0.227
-1.157
0.121

E 109
0
0
0
0

From Table A.2.2


Species
C2H2
O2
CO2
H2O

A
6.132
3.639
5.457
3.470

B 103
1.952
0.506
1.045
1.45

C 10 6
0
0
0
0

Integration of the following equation provides an algebraic expression where only T2 is


unknown.

96

T2

H rxn,298

ni 2 c P i dT 0

298 i

Substituting the proper values into the expression gives


T2 6169 K

(b)
The calculations follow the procedure used in Part (a), but now nitrogen is present. The basis is
nC 2 H 2 1 mol

The heat of reaction is the same as in Part (a), but the gas composition is different. Since
stoichiometric amount of air is used,

nO 1 2.5 mol
2

yN
n N 2 nO2 2 9.40 mol
1
1 y

O2 air

The composition of the streams are summarized below


Streams

nC 2 H 2

nO2

nN 2

nCO2

nH 2O

1
2

1
0

2.5
0

9.40
9.40

0
2

0
1

D 10 5
-1.299
-0.227
-1.157
0.121
0.04

E 109
0
0
0
0
0

From Appendix A.2


Species

C2H2
O2
CO2
H2O
N2

6.132
3.639
5.457
3.470
3.280

B 103
1.952
0.506
1.045
1.45
0.593

C 10 6
0
0
0
0
0

Therefore,
T2 2792 K

97

(c)
Now excess air is present, so not all of the oxygen reacts. The heat of reaction remains the same
because only 1 mole of acetylene reacts. Since the amount of air is twice the stoichiometric
amount

nO 1 5 mol
2

yN
n N 2 nO2 2 18.80 mol
1
1 y

O2 air

The compositions of the streams are summarized below


Streams

nC 2 H 2

nO2

nN 2

nCO2

nH 2O

1
2

1
0

5
2.5

18.8
18.8

0
2

0
1

The table of heat capacity data in Part (b) will be used for this calculation. Using the expression
shown in Part (a)
T2 1787 K

98

2.55
(a)
The combustion reaction for propane is

C3H8(g) + 5O2(g) 3CO2(g) + 4H2O(g)


For all subsequent calculations, the basis is one mole of propane. The heat of reaction is
calculated as follows

C H 5hf O 3hf CO 4hf H O

hrxn
h f

hrxn
2.044 10 6
mol

H rxn,298 nC 3 H 8 hrxn
2.044 10 6 J

The required amount of oxygen for complete combustion of propane is

nO 1 5(nC H
2

)1 5 mol

yN
n N 2 nO2 2 18.81 mol
1
1 y

O2 air
The stream compositions are listed below


Streams
1 (Inlet)
2 (Outlet)

nC 3 H 8

nO2

nN 2

nCO2

nH 2O

1
0

5
0

18.8
18.8

0
3

0
4

D 10 5
0.04
-1.157
0.121

E 109
0
0
0

From Table (a)2.2


Species
N2
CO2
H2O

A
3.280
5.457
3.470

B 103
0.593
1.045
1.45

C 10 6
0
0
0

Now all of the necessary variables for the following equation are known, except T2 .
T2

H rxn,298

ni 2 c P i dT 0

298 i

Solving the resulting expression provides


T2 2374 K

99

(b)
The combustion reaction for butane is

C4H10(g) + (13/2)O2(g) 4CO2(g) + 5H2O(g)


For all subsequent calculations, the basis is one mole of butane. The heat of reaction is
calculated as shown in Part (a)
H rxn,298 2.657 10 6 J

The moles of nitrogen and oxygen in the feed stream are calculated according to the method in
Part (a). The compositions are
Streams
1 (Inlet)
2 (Outlet)

nC 4 H 10

nO2

nN 2

nCO2

nH 2O

1
0

6.5
0

24.5
24.5

0
4

0
5

The c P data listed in Part (a) can also be used for this reaction since there is no remaining
butane.
T2 2376 K

(c)
The combustion reaction for pentane is

C5H12(g) + 8O2(g) 5CO2(g) + 6H2O(g)


The basis is one mole of pentane. The heat of reaction is calculated as shown in Part (a).
H rxn,298 3.272 10 6 J

The moles of nitrogen and oxygen in the feed stream are calculated according to the method in
Part (a). The compositions are listed below
Streams
1
2

nC 5 H 12

nO2

nN 2

nCO2

nH 2O

1
0

8
0

30.1
30.1

0
5

0
6

Substitution of the values into the expression used to find T2 and subsequent evaluation results
in
T2 2382 K

The adiabatic flame temperatures are nearly identical in all three cases.
100

2.56
The equation for the combustion of methane is

CH4(g) + 2O2(g) CO2(g) + 2H2O(g)


Using Equation 2.72
J

hrxn
8.02 105
mol
The basis for this problem is

nCH 1 1 mol
4

Also, let represent the fractional conversion of methane. Therefore, the composition of the
product gas leaving the reactor is

nCH 2 11 mol
nO 2 21 mol
nN 2 7.52 mol
nCO 2 mol
nH O 2 2 mol
4

Furthermore, the heat of reaction is calculated as follows

H rxn,298 8.02 105 J

From Table A.2.2


Species

CH4
O2
CO2
H2O
N2

1.702
3.639
5.457
3.470
3.280

B 103
9.081
0.506
1.045
1.45
0.593

C 10 6
-2.164
0
0
0
0

D 10 5
0
-0.227
-1.157
0.121
0.04

E 109
0
0
0
0
0

After substitution of the outlet composition values, heat capacity data, and the heat of reaction
into the following equation

101

1273

H rxn, 298

n c dT 0
i 2

P i

298 i

integration provides an equation with one unknown: . Solving the equation gives

=0.42
Since the fractional conversion is 0.42, 58% of the methane passed through the reactor unburned.

102

2.57
For the entire cycle,

U11 U12 U 23 U 31 0

U12 50 kJ
From state 1 to state 2
U12 Q12 W12
Q12 400 kJ
From state 2 to state 3

U 23 Q23 W23
Q23 0 kJ
From state 3 to state 1
U 31 Q31 W31

W31 250 kJ

Hence, the completed table is


Process
State 1 to 2
State 2 to 3
State 3 to 1

U kJ
-50
800
-750

W kJ
-400
800
-250

Q kJ
350
0
-500

To determine if this is a power cycle or refrigeration cycle, look at the overall heat and work,
W11 and Q11 .
W11 W12 W23 W31 150 kJ

Q11 Q12 Q23 Q31 150 kJ

Since work is done on the system to obtain a negative value of heat, which means that heat is
leaving the system, this is a refrigeration cycle.

103

2.58
Refer to the graph of the Carnot cycle in Figure E2.20. From this graph and the description of
Carnot cycles in Section 2.9, it should be clear that state 3 has the lowest pressure of all 4 states,
and state 1 has the highest pressure. States 1 and 2 are at the higher temperature. States 3 and 4
have the lower temperature. Since both the temperature and pressure are known for states 1 and
3, the molar volume can be calculated using

RT
P

The table below summarizes the known thermodynamic properties.

State

T K

P bar

1
2
3
4

1073
1073
298
298

60

v m 3 /mol
0.00149

0.2

0.124

For each step of the process, potential and kinetic energy effects can be neglected. The step from
state 1 to state 2 is a reversible, isothermal expansion. Since it is isothermal, the change in
internal energy is 0, and the energy balance becomes
Q12 W12

From Equation 2.77,


P
W12 nRT1 ln 2
P1

where W12 is the work done from state 1 to state 2. The value of P2 is not known, but
recognizing that the process from state 2 to 3 is an adiabatic expansion provides an additional
equation. The polytropic relationship can be employed to find P2 . A slight modification of
Equation 2.89 provides
Pv k const
From the ideal gas law
P

RT
v

Combining this result with the polytropic expression and noting that R is constant, allows the
expression to be written as

104

Tv k 1 const
Therefore,
1

T
k 1
v2 3 v3k 1
T2

Substituting the appropriate values (k=1.4) gives


m3
v2 0.00504

mol
Applying the ideal gas law
P2

RT2
17.7 bar
v2

Now, W12 can be calculated.


W12 10.89 kJ

Calculation of W34 follows a completely analogous routine as calculation for W12 . The
following equations were used to find the necessary properties
1
m3
T
k 1
0.0367
v4 1 v1k 1

T4

mol

P4

RT4
0.675 bar
v4

Now the following equation can be used


P
W34 nRT3 ln 4
P3

which gives
W34 3.01 kJ

105

For an adiabatic, reversible process, Equation 2.90 states


W

nR
T2 T1
k 1

This equation will be used to calculate the work for the remaining processes.
nR
T3 T2 16.11 kJ
k 1
nR
T1 T4 16.11 kJ
W41
k 1

W23

To find the work produced for the overall process, the following equation is used
Wnet W12 W23 W34 W41
Evaluating this expression with the values found above reveals
Wnet 7.88 kJ
Therefore, 7.88 kilojoules of work is obtained from the cycle.
The efficiency of the process can be calculated using Equation 2.98:

Wnet
7.88

0.72
QH 10.89

since QH W12 10.89 kJ . Alternatively, if we use Equation E2.20D.

TC
TH

where TC T3 T4 and TH T1 T2 . Upon substitution of the appropriate values

0.72

106

2.59
Since this is a refrigeration cycle, the direction of the cycle described in Figure 2.17 reverses.
Such a process is illustrated below:
Constant T H
QH

Isothermal
compression
Adiabatic
compression
State 2
Well - insulated

T2=T1
P2

T1
P1

State 3
State 1
T1
P1

Adiabatic
expansion

State 1

T3
P3

State 4
T4
P4

State 4
T4=T3
P4

State 2 State 3
T3
T2
P3
P2

Isothermal
expansion
QC
Constant T C

States 1 and 2 are at the higher temperature. States 3 and 4 have the lower temperature. Since
the both the temperature and pressure are known for states 2 and 4, the molar volume can be
calculated using
v

RT
P

The following table can be made


State

T K

1
2
3
4

1073
1073
298
298

P bar

v m 3 /mol

60

0.00149

0.2

0.124

For each step of the process, potential and kinetic energy effects can be neglected. The process
from state 1 to state 2 is a reversible, isothermal expansion. Since it is isothermal, the change in
internal energy is 0, and the energy balance becomes
Q12 W12

107

From Equation 2.77,


P
W12 nRTH ln 2
P1

where W12 is the work done from state 1 to state 2. The value of P1 is not known, but
recognizing that the process from state 4 to 1 is an adiabatic compression provides an additional
relation. The polytropic relationship can be employed to find P1 . A slight modification of
Equation 2.89 provides
Pv k const
From the ideal gas law
P

RT
v

Combining this result with the polytropic expression and noting that R is constant allows the
expression to be written as
Tv k 1 const
Therefore,
1

T
k 1
v1 4 v 4k 1
T1

Substituting the appropriate values (k=1.4) gives


1
k 1

m3
v1 4 v 4k 1 0.00504

mol
T1

RT
P1 1 17.7 bar
v1

Now, W12 can be calculated.


W12 10.9 kJ

Calculation of W34 follows a completely analogous routine as the calculation for W12 . The
following equations were used to find the necessary properties:
108

m3
v3 0.0367

mol

Applying the ideal gas law


P3

RT3
0.675 bar
v3

Now the following equation can be used


P
W34 nRTC ln 4
P3

which gives
W41 3.0 kJ

Equation 2.90 can be used to determine the work for adiabatic, reversible processes. This
equation will be used to calculate the work for the remaining processes.
nR
TC TH 16.11 kJ
k 1
nR
TH TC 16.11 kJ
W41
k 1
W23

To find the work produced for the overall process, the following equation is used
Wnet W12 W23 W34 W41
Evaluating this expression with the values found above reveals
Wnet 7.88 kJ
Therefore, 7.88 kJ of work is obtained from the cycle. The coefficient of performance is defined
in Equation 2.99 as follows
COP

QC
Wnet

where QC is the equal to Q34 . From the energy balance developed for the process from state 3
to state 4

109

Q34 W34 3.01 kJ


Therefore,
COP

3.01 kJ
0.382
7.88 kJ

110

2.60
(a)
The Pv path is plotted on log scale so that the wide range of values fits (see Problem 1.13)

logP
100

0.075

3
2

log v
v
(b)
The work required to compress the liquid is the area under the Pv curve from state 3 to state 4.
Its sign is positive. The power obtained from the turbine is the area under the curve from state 1
to 2. Its sign is negative. The area under the latter curve is much larger (remember the log
scale); thus the net power is negative.
(c)
First, perform a mass balance for the entire system:

m 1 m 2 m 3 m 4 m
Since no work is done by or on the boiler, the energy balance for the boiler is
m h1 m h4 Q H

Similarly, the energy balance for the condenser is


m h3 m h2 Q C
To find the necessary enthalpies for the above energy balances, we can use the steam tables:
kJ
h1 3424.5
kg

(520 C, 100 bar)

111

kJ
kJ
kJ
h2 0.10168.77 0.90 2574.8 2334.2
kg
kg
kg

(sat. liq at 7.5 kPa) (sat. vap. at 7.5 kPa)

kJ
h3 168.77
kg

(sat. liquid at 0.075 bar)

kJ
h4 342.81
kg

(subcooled liquid at 80 C, 100 bar)

Now, we can calculate the heat loads:

kJ
kJ
Q H 100 kg/s 3424.5 342.81 308169 kW
kg
kg

kJ
kJ
Q C 100 kg/s 168.77 2334.2 216543 kW
kg
kg

(d)
Use Equation 2.96:

W net Q net 0
From Part (c), we know
Q net 308169 kW 216543 kW 91626 kW
Therefore,
W net 91626 kW
(e)
Using the results from Parts (c) and (d):

91626 kW
0.297
308169 kW

112

Chapter 3 Solutions
Engineering and Chemical Thermodynamics

Wyatt Tenhaeff
Milo Koretsky
Department of Chemical Engineering
Oregon State University
koretsm@engr.orst.edu

3.1
Since entropy is a state function
s sys s sys, step1 s sys, step 2

Step 1 is a constant volume process. Therefore, no work is done. After neglecting potential and
kinetic energy effects, the energy balance for a reversible process becomes

u q
du q
cv dT q
Using the definition of entropy,
final

s sys ,step1

initial

q rev
T

T2

cv dT
T
T1

Step 2 is an isothermal process. For and ideal gas u is zero and the energy balance is (PE and
KE neglected)

q w
dq w
For an ideal gas, the following can be shown

wrev Pdv

RT
dv
v

Therefore,
final

s sys ,step 2

initial

qrev
T

v2

v2

v dv R ln v

v1

Combination of both steps yields


T2

s sys

T1

v
cv dT
R ln 2
T
v1

3.2
Equation 3.62 states
T2

s sys

cP

T1

P
dT R ln 2
P1

Substituting the equation for c P yields


T2

s sys

T1

P
A BT CT 2
dT R ln 2
T
P1

P
T
C
s sys A ln 2 BT2 T1 T22 T12 R ln 2
2
P1
T1

3.3
Equation 3.3 states
S univ S sys S surr

where
S sys m( s2 s1 )

S surr

Qsurr
Tsurr

(the temperature of the surroundings is constant)

First, we will calculate S sys . From the steam tables:


kJ
s1 7.1228

kg K

(300 C, 10 bar)

Since the container is rigid and mass is conserved in the process,


v2 v1

m3
v1 0.25794

kg

(300 C, 10 bar)

m3
v2 0.25794

kg
To find the number of phases present in state 2, compare the specific volume of state 2 to the
specific volume for saturated water and saturated water vapor at 1 bar. From the steam tables,

m3
v2l , sat 0.001043

kg

(sat. H2O(l) at 1 bar)

m3
v2v, sat 1.6940

kg

(sat. H2O(v) at 1 bar)

Since v2l , sat v2 v2v, sat , two phases are present. The quality of the water can be calculated as
follows
v2 1 x v2l , sat xv2v, sat

Therefore,

x 0.152

From the steam tables:

kJ
s2l , sat 1.3025

kg K
kJ
s2v, sat 7.3593

kg K

(sat. H2O(l) at 1 bar)


(sat. H2O(v) at 1 bar)

The entropy of state 2 can be calculated using the following equation:


s2 1 x s2l , sat xs2v, sat
kJ
s2 2.2231

kg K

Now the entropy change of the system can be calculated.

kJ
kJ
kJ
49.0
S sys 10 kg 2.2231
7.1228

K
kg K
kg K

To find the change in entropy of the surroundings, an energy balance will be useful. Since no
work is done, the energy balance is

U Q
We also know

Qsurr Q
To following expression is used to solve for the internal energy:

U m 1 x u 2l , sat xu 2v, sat u1

From the steam tables

kJ
u1 2793.2
kg
kJ
u 2l , sat 417.33
kg

(300 C, 10 bar)
(sat. H2O(l) at 1 bar)

kJ
u 2v, sat 2506.1
kg

(sat. H2O(v) at 1 bar)

The expression for internal energy yields


U 20583.77 kJ

Therefore,
Q 20583.77 kJ

and
S surr

Qsurr 20583.77 kJ
kJ

70.25
Tsurr
293 K
K

Now the change in entropy of the universe can be calculated

kJ
kJ
kJ
S univ S sys S surr 49.0 70.25 21.25
K
K
K
Since Suniv > 0, this process is possible.

3.4
Entropy Balance:
S univ S sys S surr

Considering the copper block to be the system, no work is done on the system; thus, the energy
balance is

du q
cv dT q

(neglecting PE and KE)

This can be used in the following expression for entropy


T2

s sys

q rev

T1
T2

s sys

cv

dT

T1

From Table A.2.3


c P 2.723R
cv c P 2.723R

(for liquids and solids)

Therefore,
T2

s sys

T1

2.723R
J 280 K
ln
dT 2.723R ln 2 2.723 8.314

T
mol K 373.15 K

T1

J
s sys 6.50
mol K

10 kg
6.50 J 1024 J

S sys
mol K
K

kg

0
.
063465

mol

Since the temperature of the lake remains constant, the change in entropy of the surroundings can
be calculated as follows
S surr

Qsurr
nCu c P T2 T1 2.723nCu RT2 T1
Q

Tsurr Tsurr
Tsurr
Tsurr

280 K 373.15 K
2.723 8.314

10 kg

mol K

S surr

kg
280 K

0.063546

mol

J
S surr 1184
K
From the definition of entropy:
J
J
J
S univ S sys S surr 1024 1184 160
K
K
K

3.5
(a)
Since the process is adiabatic and reversible,
S sys 0

(b)
The change in entropy is calculated by

liquid

S sys n

qrev
T

vapor

nh
Tb

vap

kJ
1 mol 8.2

mol
J

73.9
111 K
K

The sign is negative because there is less randomness in the liquid phase.
(c)
For this situation
q c P dT

Therefore,
Tf

S sys m
Ti

q rev
T

J 273.15 K
cP

dT
18
.
0148
g
4
.
2

g K ln 373.15 K


373.15 K

273.15 K

J
S sys 23.6
K
The sign is negative because as the water cools, less translational energy states are occupied by
the molecules. Therefore, the randomness decreases.
(d)
First, calculate the temperature at which the blocks (block A and block B) equilibrate. The
energy balance for the process is
n Ac P T2 373.15 K n B c P T2 473.15 K 0

Since the heat capacities number of moles of A and B are equal, we find that
T2 423.15 K

Now, calculate the change in entropy:

S sys S sys, A S sys, B

T
T
S sys n Ac P ln 2 n B c P ln 2
T1, A
T1, B

Substituting values, we obtain


J
S sys 0.337
K
The sign is positive because two objects at different temperatures will spontaneously equilibrate
to the same temperature when placed together.

10

3.6

The solution below compares problems 2.14 and 2.15, the calculation of 2.13 was erroneously
included in the problem statement of the first printing and is shown at the end of this problem.
Problem 2.14
Since the system is well-insulated no heat is transferred with the surroundings. Therefore, the
entropy change of the surroundings is zero and
S univ S sys

The gas in the piston-cylinder system is ideal and c P is constant, so

T
P
S sys n c P ln 2 R ln 2
T1
P1

From the problem statement, we know


P1 2 bar
P2 1 bar

The ideal gas law can be used to solve T1 .


PV
2 bar 10 L
T1 1 1
240.6 K
nR
L bar
0.08314
1 mol
mol K

Solving for T2 is slightly more involved. The energy balance for this system where potential and
kinetic energy effects are neglected is
U W

Conservation of mass requires


n1 n2
Let n n1 n2

The energy balance can be rewritten as


T2

V2

T1

V1

n cv dT PE dV

11

Since cv and PE are constant


ncv T2 T1 PE V2 V1
V2 and T1 can be rewritten using the ideal gas law

nRT2
P2
PV
T1 1 1
nR

V2

Substituting these expressions into the energy balance, realizing that PE P2 , and simplifying
the equation gives
5

P2 P1 V1
2
T2
7
nR
2
Using the following values
P1 2 bar
P2 1 bar

V1 10 L

n 1.0 mol
L bar
R 0.08314
mol K

results in
T2 206 K

Since both states are constrained, the entropy can be calculated from Equation 3.63:

J 206 K
J 1 bar
S univ S sys 1.0 mol 3.5 8.314
ln
ln
8.314

mol K 240.6 K
mol K 2 bar

J
Suniv S sys 1.24
K

12

Problem 2.15
Since the initial conditions in the piston-cylinder assembly are equal to the initial conditions of
Problem 2.14, T1 and P1 are known. Moreover, P2 is known, so we only need to find T2 in
order to calculate the entropy change. For adiabatic, reversible processes, the following
relationship (Equation 2.89) holds:

PV k const
This can be used to find V2 .
1
k
P
V2 1 V1k

P2

c
7
Noting that k P and substituting the proper values results in
cv 5
V2 16.4 L

Th polytropic expression can also be manipulated to yield

TV k 1 const.
Therefore,
T2

T1V1k 1
V2k 1

Substitution of the appropriate variables provides


T2 197.4 K

Now the entropy can be calculated,

J 1 bar
J 197.4 K
S univ S sys 1.0 mol 3.5 8.314
ln

8.314
ln

mol K 2 bar
mol K 240.6 K

J
S univ S sys 0.004
K
The value of the entropy shown represents round-off error. Since the process is reversible and
adiabatic, we know from Table 3.1 and the related discussion in Section 3.3 that the entropy
changes of the system, surroundings, and universe will be zero.
13

Problem 2.13

Equation 3.3 states


S univ S sys S surr m( s2 s1 )

Qsurr
Tsurr

where the temperature of the surroundings is constant. First, we will determine s 2 . The first law
can be applied to constrain state 2. With potential and kinetic energy effects neglected, the
energy balance becomes
U Q W
The value of the work will be used to obtain the final temperature. The definition of work
(Equation 2.7) is
V2

W PE dV
V1

Since the piston expands at constant pressure, the above relationship becomes
W PE V2 V1

From the steam tables


kJ
s1 6.2119
(10 MPa, 400 C)
kg K
m3
v1 0.02641

kg

(10 MPa, 400 C)

m3
3
V1 m1v1 (3 kg) 0.02641
0.07923 m

kg

Now V2 and v2 are found as follows

748740 J
W
0.4536 m3
0.07923 m3
6
PE
2.0 10 Pa
3
m3
V
0.4536 m
v2 2
0.1512

m2
3 kg
kg

V2 V1

14

Since v2 and P2 are known, state 2 is constrained. From the steam tables:
kJ
s2 7.1270

kg K

3
20 bar, 0.1512 m

kg

Now U will be evaluated, which is necessary for calculating Qsurr . From the steam tables:
kJ
u 2 2945.2
kg

3
20 bar, 0.1512 m

kg

kJ
u1 2832.4
kg

100 bar, 400 C

kJ
kJ
U m1 u 2 u1 3 kg 2945.2 2832.4 338.4 kJ
kg
kg

Substituting the values of U and W into the energy equation allows calculation of Q
Q U W
Q 338400 [J] 748740 J 1.09 10 6 J Qsurr

so
S univ m( s2 s1 )

Qsurr
Tsurr

kJ 1.09 103 kJ
kJ
kJ

1.126

3 kg 7.1270
6
.
2119

673.15 K
K
kg K
kg K

Therefore, this process is irreversible.

15

3.7
(a)

From Equation 3.63:


T
P
s sys c P ln 2 R ln 2
T1
P1

0.5 bar
J 7 500 K
J
s sys 8.314
ln
20.63
ln

mol K 2 300 K
mol K
1 bar

(b)
From Equation 3.65:
T
v
s sys cv ln 2 R ln 2
T1
v1

3
0.025 m

J 5 500 K
mol 4.85 J

ln
s sys 8.314

ln

mol K
m 3

mol K 2 300 K


0.05

mol

(c)
First, we can find the molar volume of state 1 using the ideal gas law.

v1

m3
RT1
0.025

P1
mol

Now, we can use Equation 3.65


T
v
s sys cv ln 2 R ln 2
T1
v1

3
0.025 m

mol

J
J 5 500 K

10.62

ln
s sys 8.314
ln

m 3

mol K 2 300 K
mol K


0.025

mol

16

3.8
(i). We wish to use the steam tables to calculate the entropy change of liquid water as it goes
from its freezing point to its boiling point. The steam tables in Appendices B.1 B.5 do not
have data for subcooled water at 1 atm. However, there is data for saturated water at 0.01 C and
a pressure of 0.6113 kPa. If we believe that the entropy of water is weakly affected by pressure,
then we can say that the entropy of water at 0.01 C and 0.6113 kPa is approximately equal to the
entropy at 0 C and 1 atm. The molar volumes of most liquids do not change much with pressure
at constant temperature. Thus, the molecular configurations over space available to the water
molecules do not change, and the entropy essentially remains constant. We do not need to
consider the molecular configurations over energy since the temperature difference is so slight.
So, from the steam tables:

kJ
s0 C , 1 atm s0.01 C , 0.6113 kPa 0

kg K
kJ
s100 C , 1 atm 1.3068

kg K
Therefore,
kJ
kJ
s s100 C , 1 atm s0 C , 1 atm 1.3068
0

kg K
kg K
kJ
s 1.3068

kg K
(ii). From the steam tables:
kJ
sl 100 C , 1 atm 1.3068

kg K
kJ
s v 100 C , 1 atm 7.3548

kg K
Therefore,
kJ
kJ
s s v 100 C , 1 atm s l 100 C , 1 atm 7.3548
1.3068

kg K
kg K
kJ
s 6.048

kg K
The change in entropy for process (ii) is 4.63 times the change in entropy for process (i). There
are many ways to reconcile this difference, but think about it from a molecular point of view. In
process (i), the available molecular configurations over energy are increased as the temperature
increases. As the temperature increases, the molar volume also increases slightly, so the
17

available molecular configurations over space also increase. Now consider process (ii), where
the molecules are being vaporized and entering the vapor phase. The molecular configuration
over space contribution to entropy is drastically increased in this process. In the liquid state, the
molecules are linked to each other through intermolecular interactions and their motion is
limited. In the vapor state, the molecules can move freely. Refer to Section 3.10 for a discussion
of entropy from a molecular view.

18

3.9
Before calculating the change in entropy, we need to determine the final state of the system. Let
the mixture of ice and water immediately after the ice has been added represent the system.
Since the glass is adiabatic, no work is performed, and the potential and kinetic energies are
neglected, the energy balance reduces
H 0

We can split the system into two subsystems: the ice (subscript i) and the water (subscript w).
Therefore,
H mi hi mw hw 0
and
mi hi mw hw

We can get the moles of water and ice.

Vw
0.0004 m 3

0.399 kg
vw
m3
0.001003

kg

mw
0.399 kg 22.15 mol

nw
MW H 2O 0.0180148 kg
mol
mi
5.55 mol
ni
MW H 2O

mw

Now, lets assume that all of the ice melts in the process. (If the final answer is greater than 0
C, the assumption is correct.) The following expression mathematically represents the change
in internal energies (cP=cv).

ni c P ,i 0 10 C h fus c P ,w T f 0 C nw c P ,w T f 25 C
Note: Assumed the heat capacities are independent of temperature to obtain this
expression.
From Appendix A.2.3
c P ,i 4.196 R
c P ,w 9.069 R
and
kJ
h fus 6.0
mol

19

Substitution of values into the above energy balance allows calculation of Tf.
T f 3.12 C
(Our assumption that all the ice melts is correct.)
Now, we can calculate the change in entropy. From Equation 3.3
suniv s sys s surr
Since the glass is considered adiabatic,
s surr 0
suniv s sys
We will again break the system into two subsystems: the ice and the water. The change in
entropy of the universe can be calculated as follows
S univ ni si nw s w
The definition of change in entropy is
final

initial

qrev
T

Assuming the heat capacities of ice and water are independent of temperature, the expressions
for the change in entropy of the subsystems are
h fus
276.27 K
273.15 K
c P, w ln
si c P, i ln

273.15 K
263.15 K 273.15 K
276.27 K
s w c P, w ln

298.15 K
Therefore
h fus

273.15 K
276.27 K
276.27 K
S univ ni c P, i ln
c P, w ln

n w c P, w ln

263.15 K 273.15 K
273.15 K
298.15 K

Substituting the values used before, we obtain

20

J
Suniv 6.59
mol

21

3.10
(a)
The maximum amount of work is obtained in a reversible process. We also know the entropy
change for the universe is zero for reversible processes. From the steam tables

kJ
s1 6.2119

kg K
kJ
s2 8.5434

kg K

(400 C, 100 bar)


(400 C, 1 bar)

Using the entropy criterion,


Suniv 0 ms2 s1 S surr

kJ
kJ
S surr ms2 s1 0.5 kg 8.5434
6.2119


kg K
kg K

kJ
S surr 1.1658
K

Since the process is isothermal, we know that the temperature of the surroundings is constant at
400 C. Therefore,
Qsurr
kJ
S surr 1.1658
Tsurr
K
Qsurr 784.76 kJ

To find the work obtained, perform an energy balance. An energy balance where potential and
kinetic energy effects are neglected is

U Q W
We can calculate the change in internal energy from the steam tables, and we also know that
Q Qsurr . From the steam tables:
kJ
u1 2832.4
kg
kJ
u 2 2967.8
kg

(400 C, 100 bar)


(400 C, 1 bar)

Therefore,

22

W mu 2 u1 Q

kJ
kJ
2832 . 4
W 0 . 5 kg 2967 . 8
784 . 76 kJ

kg
kg

W 717.06 kJ

(b)
If steam is modeled as an ideal gas, the change in internal energy is zero. Therefore, the energy
balance is

Q W
The work can be found using Equation 2.77 which is developed for reversible, isothermal
processes.
P
W nRT ln 2
P1

0.5 kg
J
1 bar
W
8.3145
673.15 K ln

mol K
100 bar
kg
0.0180148

mol

W 715332 J 715.3 kJ

Therefore,
Q 715.3 kJ

and

Qsurr 715.3 kJ

Since the temperature of the surroundings is constant,


kJ
S surr 1.06
K

23

3.11
(a)
The initial pressure is calculated as follows

P1 Psurr

25000 kg 9.81 m/s 2


0.05 m

24.6 105 Pa

Similarly, the final pressure is


P2 Psurr

35000 kg 9.81 m/s 2


0.05 m 2

34.4 105 Pa

(b)
The temperature should rise, which can be understood by considering an energy balance.
Because the system is insulated, the work done by the mass being added to the piston is
transformed into molecular kinetic energy.
(c)
The final temperature can be calculated with an energy balance:
U W
ncv T2 T1 W

Since the pressure of the surroundings is constant after the block is added to the piston, the work
is calculated as follows:
W P2 V2 V1

Assume ideal gas behavior:


V2

nRT2
P2

V1

nRT1
0.00169 m 3
P1

Now, we can create one equation with one unknown:


nRT2

n5 / 2 R T2 T1 P2
V1
P2

Substitute values and solve for T2:


T2 557 K

24

(d)
Since the system is well-insulated

s surr 0
Use Equation 3.65 to calculate the change in entropy:
T
V
T
T P
suniv s sys cv ln 2 R ln 2 5 / 2 R ln 2 R ln 2 1
T1
V1
T1
T1P2

Substituting values, we obtain,


J
s sys 0.354
mol K
(e)
Because the change in entropy of the universe is equal to change in entropy of the system, which
is positive in this situation, the second law is not violated.

25

3.12
(a)
To calculate ssys, we need to take the gas from state 1 to state 2 using a reversible process. The
process in part a can be drawn as follows:
24.6 bar
500 K

State 1

34.4 bar
557 K
rev.
adiabatic

State 2
rev.
isothermal

PI
T I = 557 K
State I

We need to find the intermediate pressure, PI where we end up at the temperature in state 2, T2.
To find it, we can use the results from pages 78-79 of the text:
T1P11 k / k TI PI1 k / k T2 PI1 k / k
Therefore,
k

T 1k
P1 35.9 [bar]
PI 1
T2

(I)

If we draw the process on a PT diagram, we get:


40
P
I
2
Adiabatic compression

Isothermal
expansion

1
20
490

560
T

26

The change in entropy can be represented as follows:


s sys s adiabatic sisothermal

For the reversible adiabatic process, the change in entropy is zero. Therefore,
s sys sisothermal

For an isothermal process,

q rev wrev

RT
dP
P

Therefore,
final

s sys

initial

P2


PI

P
R
J
dP R ln 2 0.354
P
mol K
PI

If we substitute relation (I) in the expression above, we get the same expression in the book:
P
s sys R ln 2
P1

T
T
P
k

R ln 2 c P ln 2 R ln 2
1 k
T1
T1
P1

(b)
For this construction, we use the path diagrammed on the following PT diagram:
40
P

2
Isothermal
compression

Isobaric heating
1
20
490

560
T

We write

27

s sys sisobar sisothermal

First, find an expression for the isobaric heating

q rev dh c P dT
Therefore,
T2

sisobar

TP dT c P ln T12

T1

Now, we need an expression for the isothermal, reversible expansion

q rev wrev Pdv


Therefore,
v 2 RT2 / P2

sisothermal

v I RT2 / P1

P
dv
T

v 2 RT2 / P2

v1 RT2 / P1

P
P
R
dv R ln 1 R ln 2
v
P2
P1

Combine the two steps:


T
P
J
s sys c P ln 2 R ln 2 0.354
mol K
T1
P1

(c)
For this construction, we use the path diagrammed on the following PT diagram:

28

40
P

2
Isothermal
compression
I

Isochoric heating

1
20
490

560
T

For isochoric heating followed by an isothermal expansion, the entropy can be expressed as
follows:
s sys sisochoric sisothermal

For the reversible, isothermal expansion, we obtain (refer to Parts (a) and (b) to see how this is
derived)
P
sisothermal R ln 2
PI
However, we can relate the intermediate pressure to the initial pressure through the ideal gas law:

PI P1

T2 T1
or
PI 27.4 [bar]

P2
T
P
P2

sisothermal R ln R ln
R ln 2 R ln 2
T2
PI
T1
P1
P1

T1

For the isochoric process:

q rev cv dT
Therefore,

29

T2

sisochoric

cv

T dT cv ln T12

T1

Combining these results:


T
s sys cv R ln 2
T1

P
R ln 2

P1

T
P
J
c P ln 2 R ln 2 0.354
mol K

T1
P1

Using any of these three reversible paths, we get the same answer!

30

3.13
(a)
Since the process is reversible and adiabatic, the entropy change for the process is zero.

s sys 0
Furthermore, since the process is adiabatic, the changes in entropy of the surroundings and the
universe are also zero.
(b)
Since this process is isentropic (s=0), we can apply an expression for the entropy change of an
ideal gas.
T2
c
P
s 0 P dT Rln 2
P1
T T
1

Insert the expression for cP from Appendix A

P
3.639 0.506 10 3 T 0.227 10 6 T 2
dT R ln 2
T
P1
T1

T2

s 0 R

which upon substituting

T1 250 K
P1 1 bar

P2 12.06 bar
yields
T2 482 K

(c)
An energy balance gives
T2

T2

T1

T1

w u cv dT

1dT

31

J
w R 2.639 0.506 10 3 T 0.227 10 6 T 2 dT 5390
mol
T1
T2

(d)
Reversible processes represent the situation where the minimum amount of energy is required for
compression. If the process were irreversible, more work is required, and since the process is
adiabatic, the change in internal energy is greater. Since the change in internal energy is greater,
so too is the change in temperature. Therefore, the final temperature would be higher than the
temperature calculated in Part (b).

32

3.14
The problem statement states that the vessel is insulated, so we can assume that heat transfer to
the surroundings is negligible. Therefore, the expression for the entropy change of the universe
is
suniv s sys

An energy balance will help us solve for the entropy change. Neglecting potential and kinetic
energy effects, the energy balance is

U Q W
Since the vessel is insulated, the heat term is zero. Furthermore, no work is done, so the energy
balance is
U mu 2 u1 0
u1 u 2

The values for the initial pressure and temperature constrain the value of specific energy at state
1. From the steam tables,

kJ
u1 2619.2
kg
kJ
u 2 2619.2
kg

(400 C, 200 bar)

The problem statement provides the pressure of state 2 (100 bar). Since we know the pressure
and internal energy at state 2, the entropy is constrained. Information given in the problem
statement also constrains state 1. From the steam tables,

kJ
s1 5.5539

kg K
kJ
s2 5.7754

kg K

(400 C, 200 bar)

kJ
100 bar, u 2 2619.2

kg

Therefore,

kJ
suniv ssys s2 s1 0.2215

kg K
kJ
S univ S sys 0.2215
K

33

3.15
The subscript 2 refers to the final state of the system. 1 refers to the gas initially on one side
of the partition. If you take the system to be the entire tank (both sides of the partition), then no
net work is performed as the gas leaks through the hole. Furthermore, the tank is well insulated,
and kinetic and potential energy effects can be neglected. Thus, the energy balance is
u 0

and
T2 T1

(ideal gas)

Initially, the partition is divided into two equal parts. The gas fills the entire volume for the final
state ( V2 2V1 ). The ideal gas law can be used to calculate the moles of gas present.

P1V1
10 105 Pa 0.5 m 3 200.5 mol

RT1
J
300 K
8.314
mol K

Now, the change in entropy can be solved using a slight modification of Equation 3.65:

nv
T
v
T
S sys n cv ln 2 R ln 2 n R ln 2 R ln 2
nv1
T1
v1
T1
2

J 3

S sys 200.5 mol 8.314


ln1 ln2

mol K 2

J
S sys 1155
K

34

3.16
(a) A schematic of the process is shown below:
1 bar, 298 K

N2

N2
N2

N2

P2, T2

1 bar, 298 K

O2

Mixing
Process

O2

N2

N2

N2
N2

N2

N2

N2

N2

O2

O2

N2
N2

N2
N2

The tank is insulated. The change in entropy of the universe can be rewritten as
S univ S sys, N 2 S sys, O2

since the tank is well-insulated. After the partition ruptures, the pressure and temperature will
remain constant at 1 bar and 298 K, respectively. This can be shown by employing mass and
energy balances. Therefore, the partial pressures in the system are

p2, N 2 0.79 bar


p2,O2 0.21 bar
Use Equation 3.62 to determine the entropies:
298 K

s sys, N 2

P2
cP

J 0.79 bar
8.314

ln
ln
dT
R

T
P1 N
mol K 1 bar

298 K
2

J
s sys, N 2 1.96
mol K
298 K

s sys, O2

cP
J 0.21 bar
dT R ln 2 8.314
ln

T
P1 O
mol K 1 bar

298 K
2

J
s sys ,O2 12.98
mol K
Therefore,

J
J
S univ 0.79 mol1.96
0.21 mol12.98

mol K
mol K

35

J
S univ 4.27
K
(b) A schematic of the process is shown below:
2 bar, 298 K

N2

N2

N2

N2

N2

N2

O2

N2

N2
N2

N2

N2

N2
N2

P2, T 2

1 bar, 298 K

N2

N2

Mixing
Process

N2

N2

N2

N2
N2

N2
N2

O2

N2

N2

O2
N2

N2

N2
N2

N2
O2
N2

N2

N2

Before calculating the change in entropy, we need to find how the temperature and pressure
change during the process. The energy balance simplifies to
U 0

which can be rewritten as

nO

n N 2 cvT2 n N 2 cv 298 K nO2 cv 298 K 0

Assuming the heat capacities are equal, we can show that


T2 298 K

We can find the pressure after the rupture by recognizing that the tank is rigid. Therefore,
VO2 V N 2 Vtot

By employing the ideal gas law, we get the following equation (it has been simplified):

nN 2
P1, N 2

nO2
P1, O2

nO

nN 2

P2

Substitute values and solve to obtain


P2 1.65 bar

Now, use Equation 3.62 to calculate the entropies as was done in Part (a):

36

298 K

s sys, N 2

cP
J 1.30 bar
dT R ln 2 8.314
ln

T
P1 N
mol K 2 bar

298 K
2

J
s sys, N 2 3.58
mol K
P

cP
J 0.347 bar
dT R ln 2 8.314
ln

T
mol K 1 bar

P1 O2
298 K

298 K

ssys ,O2

J
s sys ,O2 8.8
mol K
Therefore,

J
J
S univ 0.79 mol 3.58
0.21mol 8.8

mol K
mol K

J
Suniv 4.68
K

37

3.17
First start with the energy balance for the throttle. Potential and kinetic energy effects can be
neglected. During the throttling process, no shaft work is performed and the rate of heat transfer
is negligible. Therefore,
m 2 h2 m 1h1

A mass balance allows the energy balance to be simplified to


h2 h1
From the steam tables:
kJ
h1 3398.3
kg
kJ
h2 3398.3
kg

(500 C, 8 MPa)

Now calculate entropy:


suniv ssys ssurr

Since the process is adiabatic,


kJ
ssurr 0

kg K
The steam tables can be used to calculate the change in entropy of the system.
kJ
s2 8.7098

kg K
kJ
s1 6.7239

kg K

kJ
100 kPa, h2 3398.3

kg

(8 MPa, 500 C)

Thus,
kJ
suniv ssys s2 s1 1.9859

kg K

38

3.18
For this process to work, conservation of mass and the first and second laws of thermodynamics
must hold. The subscript 1 refers to the inlet stream, 2 refers to the cold outlet, and 3
refers to the hot outlet. To test the conservation of mass, perform a mass balance

m 1 m 2 m 3
kg
kg
kg
2 0.5 1.5
s
s
s
Clearly, the conservation of mass holds. Now test the first law of thermodynamics by writing an
energy balance. Since there is no heat transfer or work, the energy balance becomes
m 2 h2 m 3h3 m 1h1 0

(PE and KE effects neglected)

Using the conservation of mass we can rewrite the mass flow rate of stream 1 in terms of streams
2 and 3:

m 2 h2 h1 m 3 h3 h1 0

If we assume that the heat capacity of the ideal gas is constant, the equation can be written as
follows:
m 2 cP T2 T1 m 3cP T3 T1 0
Therefore,
kg
kg
0.5 cP (60 K) 1.5 cP (20 K) 0
s
s
This proves that the first law holds for this system. For the second law to be valid, the rate of
change in entropy of the universe must greater than or equal to 0, i.e.,
dS
0
dt univ
Assuming the process is adiabatic, we can write the rate of entropy change of the universe for
this steady-state process using Equations 3.48-3.50:
dS
m 2 s2 m 3 s3 m 1s1 m 2 s2 s1 m 3 s3 s1
dt univ

39

where a mass balance was used. For constant heat capacity, we can calculate the entropy
differences using Equation 3.63:
5 T
P
P
T
s2 s1 cP ln 2 R ln 2 R ln 2 ln 2
P1
P1
T1
2 T1
5 T
P
P
T
s3 s1 cP ln 3 R ln 3 R ln 3 ln 3
P1
P1
T1
2 T1
Applying these relations, we get:
1
dS
2.77 R 0.057cP

dt univ MW

Therefore, the second law holds if cP 48.6 R ; this value is far in excess of heat capacity for
gases, so this process is possible.

40

3.19
(a)
T1 = 640 oC
P2 = 100 kPa

P1 = 4 MPa
V1= 20 m/s

Nozzle

V2=?

(b)
Since the process is reversible and adiabatic,

J
S sys 0
K
(c)
The steam tables can be used to determine the final temperature. From the interpolation of steam
table data

kJ
s2 s1 640 C, 4 MPa 7.4692

kg K
Now two thermodynamic properties are known for state 2. From the steam tables:

kJ
P2 0.1 MPa, s2 7.4692

kg
K

T2 121.4 C

(d)
An energy balance is required to calculate the exit velocity. The energy balance for the nozzle is

0 m 2 h eK 2 m 1 h e K 1
Realizing that m 1 m 2 allows the energy balance to be written as

eK ,2 h e K 1 h2
which is equivalent to

V2 2 h 0.5V 2 1 h2

Substituting the following values

41


m
V1 20
s

J
h2 2719100
kg
J
h1 3767000
kg

(0.1 MPa, 121.4 C)


(4 MPa, 640 C)

yields

m
V2 1448
s
Note: the velocity obtained is supersonic; however, the solution does not account for this type of
flow.

42

3.20
A schematic of the process follows:

Propane in
v1 = 600 cm3/mol
T1= 350 oC

Turbine

ws
Propane P = 1 atm
2
out

Since this process is isentropic (s=0), we can apply an expression for the entropy change of an
ideal gas. We must be careful, however, to select an expression which does not assume a
constant heat capacity.
T2
c
P
s 0 P dT Rln 2
P1
T T
1

Insert the expression for cP from Appendix A

P
1.213 28.785 10 3 T 8.824 10 6 T 2
s 0 R
dT R ln 2
T
P1
T1
T2

We can also relate P1 to T1 and v1 (which are known) through the ideal gas law:
P1

RT1
v1

This leaves us with 1 equation and 1 unknown (T2). Integrating:


T
Pv
s 0 1.213 ln 2 28.785 10 3 (T2 T1 ) 4.412 10 6 T22 T12 ln 2 1
T1
RT1

Using
T1 = 623 K,
v1 = 600 cm3/mol,
P2 = 1 atm

43

R=82.06 cm3 atm/mol K,


we get:
T2 = 454 K

To solve for

W S
we need a first law balance, with negligible ke and pe, the 1st law for a steady
n

state process becomes:


0 n h1 h2 Q W S
If heat transfer is negligible (isentropic),
T2
T2
2
W S
h2 h1 c p dT R 1.213 28.785 10 3 T 8.824 10 6 T dT
n
T1
T1

integrating:

W S
R 1.213T2 T1 14.393 10 3 T22 T12 2.94110 6 T23 T13
n
Substituting
J
R 8.314
mol K
T2 454 K
T1 623 K

results in
W S
J
19860
n
mol

44

3.21
A reversible process will require the minimum amount of work. For a reversible process,

suniv 0
s sys s surr
For this process, the change in entropy of the system can be calculated as follows
s sys sO2 s N 2

where

sO2

T2 c

P O2

T1

s N 2

T2 c

T1

P
dT R ln 2
P1

P N 2

O2

P
dT R ln 2
P1

N2

Since the temperature of the streams do not change, the expressions reduce to

P
sO2 R ln 2
P1 O2
P
s N 2 R ln 2
P1 N 2
Therefore,

s surr R ln

P2
P1

ln 2
O2
P1 N 2

Since we are assuming the temperature of the surroundings remain constant at 20 C,


q
s surr surr
Tsurr
where

qsurr q

45

Therefore,

P
P2
2

q surr Tsurr s surr RTsurr ln ln


P1 O
P1 N 2
2

1 bar

J
1 bar

q surr 8.314
293
.
15
K
ln
ln

mol K
0.79 bar N 2
0.21 bar O2

J
q surr 4378.2
mol
and

J
q 4378.2
mol
Energy Balance:

h nout hout Q W S 0

in in

in

out

Because the temperature of the oxygen and nitrogen doesnt change in the process, the energy
balance per mole of feed becomes
wS q

J
wS 4378.2
mol

46

3.22
(a)
Take the entire container to be the system and assume no heat or work crosses the system
boundary.

Energy Balance:
U 0

After the oscillation cease, the temperature and pressure on both sides of the piston will be the
same assuming the metallic piston is very thin and the thermal conductivity coefficient is large.
Now, lets rewrite the energy balance.

U n1, A ugas A n1,B ugas B 0


or
n1, A
ugas A ugas B 0
n1,B
n1, A PA,1VA,1TB ,1
P T

A,1 B ,1 2.41
n1,B PB ,1VB ,1TA,1 2 PB ,1TA,1
since
2V A,1 VB,1
so
T

2
3

2.41 RT2 773.15 K R 5/2 1.5 10-3 T dT 0


2

373.15 K

T2 585 K

Mass Balance:
n A,1 n B,1 n A,2 n B,2
Using the ideal gas law, we get

47

PA,1VA,1 PB ,1VB ,1 PA, 2VA, 2 PB , 2VB , 2 P2Vtot

RTA,1
RTB ,1
RTA, 2
RTB , 2
RT2
since the pressure and temperature of state 2 are equal on both sides. But
Vtot V A,1 VB,1 3V A,1
Using the volume relationships and simplifying, we get
P2

T2 PA,1 2 PB ,1 585 K 10 bar


1 bar

3 TA,1 TB ,1
3 773.15 K
373.15 K

P2 3.56 bar

(b)
The container is well-insulated, so
suniv s sys

The entropy change of the system can be split into two subsystems:

ssys

n1, A
n1,B
s gas A
s gas B
n1, A n1,B
n1, A n1,B

Using Equation 3.63 and realizing that c P cv R ,

sgas A
sgas B

P
T
2.41
J
2.5 R ln 2 R ln 2 1.96
3.41
mol K
PA,1
TA,1

T
P
1 2 3.5 1.5 10 3 T
J
R

dT R ln 2 1.51
3.41 TB ,1
T
mol K
PB ,1

Therefore,

J
J
J
suniv ssys 1.96
1.51
3.47

mol K
mol K
mol K
The process is possible.

48

3.23

Equation 3.3 can be modified to show


S univ S sys S surr

Lets first calculate S sys . Before this problem is solved, a few words must be said about the
notation used. The system was initially broken up into two parts: the constant volume container
and the constant pressure piston-cylinder assembly. The subscript 1 refers to the constant
volume container; 2 refers the piston-cylinder assembly. i" denotes the initial state before the
valve is opened, and f denotes the final state.
First, the mass of water present in each part of the system will be calculated. The mass will be
conserved during the expansion process. Since the water in the rigid tank is saturated and is in
equilibrium with the constant temperature surroundings (200 C), the waters entropy is
constrained. From the steam tables,

kJ
v1l, i 0.001156
kg
kJ
v1v, i 0.12736
kg
kJ
s1l, i 2.3308
kg
kJ
s1v, i 6.4322
kg

(Sat. water at 200 C)

P sat 1553.8 kPa


Knowledge of the quality of the water and the overall volume of the rigid container can be used
to calculate the mass present in the container.

V1 0.05m1 v1l, i 0.95m1 v1v,i

Using the values from the steam table and V1 0.5 m 3 provides
m1 4.13 kg

Using the water quality specification,

49

m1v 0.95m1 3.92 kg


m1l 0.05m1 0.207 kg

For the piston-cylinder assembly, both P and T are known. From the steam tables
m3
v2, i 0.35202

kg
kJ
s2, i 6.9665

kg K

(600 kPa, 200 C)

Now enough information is available to calculate the mass of water in the piston assembly.
V
m2 2 0.284 kg
v2
Now the final state of the system must be determined. It helps to consider what physically
happens when the valve is opened. The initial pressure of the rigid tank is 1553.8 kPa. When
the valve is opened, the water will rush out of the rigid tank and into the cylinder until
equilibrium is reached. Since the pressure of the surroundings is constant at 600 kPa and the
surroundings represent a large temperature bath at 200 C, the final temperature and pressure of
the entire system will match the surroundings. In other words,
kJ
s f s2, i 6.9665
kg

(600 kPa, 200 C)

Thus, the change in entropy is given by


S sys (m2 m1v, i m1l, i ) s f m2 s2, i m1v, i s1v, i m1l, i s1l, i

Substituting the appropriate values reveals


kJ
S sys 3.05
K
Now we calculate the change in entropy of the surroundings. Since the temperature of the
surroundings is constant,
S surr

Qsurr
Q

Tsurr Tsurr

After neglecting potential and kinetic energy effects, the energy balance becomes
50

U Q W
The change in internal energy and work will be calculated in order to solve for Q. The following
equation shows how the change in internal energy can be calculated.
U (m2 m1v, i m1l, i )u f m2 u 2, i m1v, i u1v, i m1l, i u1l, i

From the steam tables


kJ
u1l, i 850.64
kg
kJ
u1v, i 2595.3
kg

kJ
u f u 2, i 2638.9
kg

(sat. H2O at 200 C)

(600 kPa, 200 C)

Using these values and the values of mass calculated above,


U 541.0 kJ

Calculating the work is relatively easy since the gas is expanding against a constant pressure of
600 kPa (weight of the piston was assumed negligible). From Equation 2.7,
Vf

W PE dV PE (V f Vi )
Vi

where
PE 600000 Pa

V f (m2 m1v, i m1l,i )v2, i 1.55 m 3

Vi 0.1 m 3 0.5 m 3 0.6 m 3

Note: v2, i was used to calculate V f because the temperature and pressure are the same
for the final state of the entire system and the initial state of the piston-cylinder assembly.
The value of W can now be evaluated.
W 570 kJ

51

The energy balance can be used to obtain Q.


Q U W 541.0 kJ 570 kJ 1111 kJ

Therefore,
S surr

1111 kJ
kJ
2.35
473.15 K
K

and
kJ
kJ
kJ
Suniv S sys S surr 3.05 2.35 0.70
K
K
K

52

3.24
Let the subscript 1 represent the state of the system before the partition is removed. It has two
components: component a and component b. Subscript 2 represents the system after the
partition is removed.

Mass balance:
n1, a n1, b n2
Energy balance for the adiabatic process:
U sys W

n1, a cv, a T2 T1, a n1, b cv, b T2 T1, b P V2 V1, a V1, b

The external pressure, P, for the above energy balance is equal to P1,a. To find the final
temperature, first find the volumes using the ideal gas law.

V1, a

V2

2 mol 8.314

J
300 K
mol K

0.05 m 3
1000 kg 9.8 m s - 2

0.098 m 2

4 mol 8.314

J
T2
mol K

3.3256 10 4 T2 m 3
2
1000 kg 9.8 m s

0.098 m 2

Substitute the volumes into the energy balance and solve for T2 :

2 mol 3 R T2 300 K 2 mol 3 R T2 300 K 1 105 Pa 3.3256 10 4 T2 0.15 m 3

2
T2 360.4 K

To calculate the change in entropy, we can use the following relationship

T
P
T
P
S sys na c P ln 2 R ln 2 nb c P ln 2 R ln 2
T1
P1 b
T1
P1 a

The only unknown in the above equation is P1,b, so we can calculate it with the ideal gas law:

53

P1, b

J
300 K
mol K

2 mol 8.314

0 .1 m

0.5 10 5 Pa

Substitute values into the expression for entropy and solve:


5 360.4 K
5 360.4 K
1 bar
1 bar
ln
S sys 2 molR ln

2 molR ln
ln
0.5 bar b
1 bar a
2 300 K
2 300 K

J
S sys 3.72
K

54

3.25
First perform an energy balance on the process:
U sys W

The change in internal energy can be written:


U n A, f u A, f nB , f u B , f n A,i u
A mass balance gives:
n A,i n A , f nB , f
These two expressions can be substituted to give:
U n A, f cv TA, f Ti nB , f cv TB , f Ti W

5
R , we obtain
2
5
5
5
PB, f VB, f PA, f V A, f PA, iV A, i W
2
2
2

Using the ideal gas law and cv

Now, find an expression for the work


Vf

W PE dV
Vi

The pressure is not constant; its value is given by


k VB VB ,i mg
kx mg

Patm

A2
A
A
A
5
5
PE 1.06 10 8.284 10 VB Pa
PE Patm

Integrating
W 1.06 105VB, f 4.142 105VB2, f

Substituting this expression into the energy balance gives:


5
5
5
PB, f VB, f PA, f V A, f PA,iV A, i 1.06 105VB, f 4.142 105VB2, f
2
2
2

55

This expression can be simplified by recognizing that PB, f PA, f and V A, f V A, i :

5
5
PB, f VB, f V A, i PA, iV A, i 1.06 105VB, f 4.142 105VB2, f
2
2
We still have two unknowns: PB, f and VB, f . We can eliminate PB, f by writing a force
balance for the final state:
PB, f 1.06 105 8.284 105VB, f Pa

Substituting this into the energy balance, we obtain

5
5
1.06 105 8.284 105VB, f VB, f V A, i PA, iV A, i 1.06 105VB, f 4.142 105VB2, f
2
2
Solving for the final volume:
VB, f 0.268 m 3

and for the final pressure.


PB, f PA, f Pf 1.06 105 8.284 105 0.268 3.28 105 Pa

Since the gas in A has undergone an adiabatic, reversible expansion


S sys, A 0

Therefore,
T
P
0 c P ln 2 R ln 2
T1
P1
3.28 10 5 Pa

J 7 T2

0 8.314
ln
ln
7.0 10 5 Pa
mol K 2 313.15 K

Solve for T2:


T2 252 K

56

3.26
A schematic of the process is drawn:
Q surr=69.86 kW
Steam
in
P1 = 60 bar
T1 = 500 oC

Turbine
Steam
out

P2 = 10 bar

Tsurr=300 K

(a) and (b)


The maximum work occurs for a reversible process. Applying the second law, we get:

dS
dS
dS
0
dt univ dt sys dt surr

or
m s2 s1

Q surr
0
Tsurr

Calculate the change in entropy of the surroundings:


kJ
We can look up property values of state 1 from the steam tables s1 6.8802
and
kgK
kJ
h1 3,422.13 . Converting the units of mass flow rate gives:
kg

kg 1 hr
kg
m 4,500
1.25
hr 3,600 s
s

so
s2 s1

kJ
Q surr
6.6939

m Tsurr
kgK

We now know two properties of steam ( s2 and P2) From the steam tables:
T2 200 oC

Using the steam tables for superheated steam at 1 MPa, we find that when water has this value of
entropy

57

kJ
h2 2827.9
kg

Energy balance:

m h2 h1 Q W s

Calculate Ws:

kJ
kJ
W s m h2 h1 Q 1.25 kg/s 2827.9 3422.1 69.86 kW
kg
kg

W s 673 kW

(c)
The isentropic efficiency is defined as follows:

W S actual
W S reversible

For this situation,

W S actual 0.665 673 kW 447.5 kW


(d)
The real temperature should be higher since not as much energy is converted into work.
(e)
Use the energy balance:

m h2 h1 Q W s
kJ
kJ
Q W s 69.86 kW -447.5 kW
h1
3422.1 3008.2
h2
1.25 kg/s
m
kg
kg

At 1 MPa, water has this value of enthalpy when

T2 actual 280.2 C

58

3.27
Since the compressor is adiabatic, the energy balance after neglecting potential and kinetic
energy becomes

n h2 h1 W S

Using the ideal gas law and Appendix A.2, the above equation becomes

T
P1V1 2

WS
A BT CT 2 DT 2 ET 3 dT
T1
T1

Substituting the following values


P1 1 bar 1 105 Pa
m3
V1 1

s
T1 293.15 K
T2 473.15 K
A 3.355; B 0.575 10 3 ; C 0; D 0.016 105 ; E 0 (Table A.2.2)

gives
W S 218.8 kW

This is the work of our real turbine (with 80% isentropic efficiency). We can use the isentropic
efficiency to calculate the work of an equivalent 100% efficient process.

compressor S reversible
WS

compressor

W S reversible W S compressor compressor 0.8218.8 kW


W S reversible 175.4 kW

Since the process is adiabatic, we can use the following equation again to calculate what the final
temperature would be in a reversible process.

T
P1V1 2

WS
A BT CT 2 DT 2 ET 3 dT

T1
T1

59

P V
175400 W 1 1
T1

A BT CT

T2

DT 2 ET 3 dT

T1

Substituting the values from above and solving for T2 , we obtain


T2, rev 164.8 K

To obtain the pressure the final pressure for the real compressor, we can calculate the final
pressure for the reversible process because the final pressure is the same in both cases. For the
isentropic expansion
T2

s 0

P
dT R ln 2
T
P1
T1

cp

A BT CT 2 DT 2 ET 3
P
s 0 R
dT R ln 2
T
P1

T1
T2

T2 438 K

P2
3.355 0.575 10 3 T 1600T 2
dT
ln
s 0 R

1 bar

293
.
15
K
T

Solve for P2:


P2 4.16 bar

60

3.28
Isentropic efficiency for a turbine is defined as

ws actual
ws rev

If the rate of heat transfer is assumed negligible, the energy balance for this process is

0 m h1 h2 W s

For a reversible, adiabatic process,


s sys 0

s2,rev s1 500 C, 10 MPa

From the steam tables


kJ
s2,rev s1 6.5965

kg K

Since s2l , sat s2 s2v, sat , a mixture of liquid and vapor exists. The quality of the water can be
calculated as follows
s2,rev 1 x s2l ,sat xs2v ,sat

kJ
kJ
kJ
x 7.3593
6.5965
1 x 1.3025


kg
K
kg
K

kg K


x 0.874

Therefore,
h2,rev 1 x h2l ,sat xh2v ,sat

kJ
kJ
h2,rev 1 0.874 417.44 0.874 2675.5
kg
kg

kJ
h2,rev 2391.0
kg

Also, from the steam tables,


kJ
h1 3373.6
kg

(500 C, 10 MPa)

61

The reversible work is calculated as:

W S rev h
m 1

kJ
kJ
kJ
h1 2391.0 3373.6 982.6
kg
kg
kg

and the actual work as

W S actual
m 1

W S rev 835 kJ
m 1

kg

The exit temperature is calculated by determining the enthalpy of the actual exit state:

W S actual
kJ
h

2538.4
2, actual h1
m 1
kg
This state is still saturated (although the quality is higher), so the temperature is
T2 = 99.6 oC

62

3.29
The subscripts 2 and 3 represent the two outline streams, and 1 represents the inlet stream.
First, perform a mass balance:

n1 n 2 n3

2
1
n1 n1
3
3

Now write the energy balance:


0 n1h1 n2 h2 n3 h3 Q W S
Since the system is insulated and there is no shaft work, the energy balance can be rewritten as:
2
1
n1h2 n1h3 n1h1 0
3
3
2
h2 h1 1 h3 h1 0
3
3
Substituting expressions for heat capacity A.2.2, we obtain the following expression:
400 K

4000
1
4000
3
3
3.280 0.593 10 T T 2 dT 3 R 3.280 0.593 10 T T 2 dT 0
300 K
300 K
Solving, we find

2
R
3

T3 100.6 K
Now set up an entropy balance for the process. The minimum pressure is required for a
reversible process.

2
1
n1s2 n1s3 n1s1 0
3
3
2
s2 s1 1 s3 s1 0
3
3
Assuming ideal gas behavior, we can express the changes in entropies using Equation 3.62:
400 K 3.280
1bar
4000

ln
s 2 s1 R
dT
0.593 10 3

3
T
P

300 K
100.6 K 3.280
1bar
4000

ln
s3 s1 R
dT
0.593 10 3

3
T
P

300 K

63

Substitute these expressions into the entropy balance and solve for P1:
P1 1.85 bar

64

3.30
A schematic of the process is illustrated below:

reversible

Tsurr = 25 oC
Psurr = 1 bar
Pure H 2O
P2 = 20 bar

Pure H2O

T 2 = ? oC

P1 = 60 bar
T1 = 540 oC

m = 5 kg

m = 5 kg

well-insulated

(a)
Since the process is adiabatic and reversible,
suniv 0
s sys 0
s surr 0

(b)
We can obtain the final temperature using the steam tables.

kJ
s1 6.999

kg K
kJ
s2 6.999

kg K

(540 C, 60 bar)
(20 bar)

The pressure and entropy of state 2 can be used to back out T2 from the steam tables.
T2 362.5 C

(c)
To obtain the value of work, perform an energy balance. The process is adiabatic, and potential
and kinetic energy effects can be neglected. Therefore, the energy balance is
U mu 2 u1 W

From the steam tables,


kJ
u2 2881.15
kg

(362.5 C, 20 bar)

65

kJ
u1 3156.12
kg

(540 C, 60 bar)

Hence,

kJ
kJ
W 5 kg 2881.15 3156.12 1373 kJ
kg
kg

(d)
The specific volume can be found in the steam tables

m3
v2 0.14173

kg

Therefore,

m3
3
V2 5 kg 0.14173
0.709 m

kg

66

3.31
A schematic of the process is shown below:
very tiny
leak hole

Tsurr = 25 oC
Psurr = 1 bar

Pure H2O

Pure H2O
P1 = 60 bar

P2 = 20 bar

T1 = 540 oC

T ' 2 = ? oC

m = 5 kg

well-insulated

In order to leave the system, the gas must do flow work on the surroundings. The initial state is
the same as for Problem 3.30 and the final pressures are the same. Since the water only expands
against 1 bar, the work is lower than that for the differential process described in Problem 3.30.
Thus, this adiabatic process looses less energy, leading to a higher final temperature.
Another way to view this argument is to look at this process as a closed system. This depiction
is the expansion analog of the compression process depicted for Example 2.5 in Figure E2.5B
(page 57). We can represent this process in terms of two latches, one that keeps the process in its
initial state at 60 bar and one that stops the expansion after the pressure has reduced to 20 bar.
The process is initiated by removal of the first latch and ends when the piston comes to rest
against the second latch. Such a process is depicted as Problem 3.31 below. The
corresponding reversible process of Problem 3.30 is shown next to it for comparison. Clearly the
process on the left does less work, resulting in a greater final temperature.

Problem 3.31

Problem 3.30

T surr = 25 oC
Psurr = 1 bar
latches
Pure H2O
P1 = 60 bar
T1 = 540 oC

irreversible

reversible

Tsurr = 25 oC
Psurr = 1 bar

Pure H2O

Pure H 2O

P2 = 20 bar

P2 = 20 bar

Pure H2O

T ' 2 = ? oC

T 2 = ? oC

P1 = 60 bar

m = 5 kg

m = 5 kg

T1 = 540 oC

m = 5 kg

m = 5 kg

well-insulated

67

3.32
(a)
Consider the tank as the system.

Mass balance
dm
m in m out m in
dt

Separating variables and integrating:


m2

m1

dm m in dt

or
t

m2 m1 m in dt
0

Energy balance
Since the potential and kinetic energy effects can be neglected, the open system, unsteady state
energy balance is
dU

m out hout m in hin Q W s


dt

sys out
in

The process is adiabatic and no shaft work is done. Furthermore, there is only one inlet stream
and not outlet stream. Therefore, the energy balance simplifies to
dU

m in hin
dt sys

The following math is performed


U2

U1 0

dU m

h dt hin m in dt

in in

U 2 m2u 2 m2 hin

where the results of the mass balance were used. Thus,


u 2 hin

68

From the steam tables,


kJ
hin 3456.5
kg

(3 MPa, 773 K)

Now the water in the tank is constrained. From the steam tables:
kJ
s2 7.743

kg K
3

m
v2 0.14749

kg

kJ
3 MPa, u 2 3456.5

kg

Compute the change in entropy. An entropy balance gives:


dS
dS
m in sin
dt univ dt sys

Integrating with sin constant


t

S univ m2 s2 sin m in dt m2 s2 sin


0

From the steam tables:


kJ
sin 7.2337

kg K

(3 MPa, 773 K)

Therefore,

0.05 m 3
kJ
kJ
kJ
7.743
0.173
7.2337
S univ

m 3
K
kg K
kg K
0.14749

kg

(b)
If it tank sits in storage for a long time and equilibrates to a final temperature of 20 C, some or
all of the vapor will condense and exchange heat with the surroundings. Let the subscript 3
designate the final state of the water when it has reached a temperature of 20 C. The change in
entropy between state 2 and state 3 is given by the following equation
69

S univ m2 s3 s2

Qsurr
T

Let the subscript 3 designate the final state of the water when it has reached a temperature of
20 C. First, find quality of the water. From the steam tables:
m3
v3sat
, l 0.001002 kg

(sat. water at 20 C)

m3
v3sat

57
.
79

,v
kg

Calculate the quality as follows:


sat
v3 v2 1 x v3sat
, l xv3, v

m3
m3
m3
0.14749 1 x 0.001002 x 57.79
kg
kg
kg

x 0.0025

Now calculate the entropy of state 3


sat
s3 1 x s3sat
, l xs3, v

Substitute values from the steam tables:

s3 1 0.0025 0.2966

kJ
s3 0.3175

kg K

kJ
kJ

0
.
0025
8
.
6671

kg K
kg K

Now calculate the amount of heat transferred to the surroundings:


Q surr Q m2 u 3 u 2

Calculate the internal energy of state 3:


sat
u 3 1 x u 3sat
,l xu 3,v

Substituting values from the steam tables:


70

kJ
u 3 89.74
kg

Therefore,

0.05 m 3
kJ
kJ
89.74 3456.5 1141 kJ
Q surr

m 3
kg
kg
0
.
14749

kg

Now calculate the change in entropy of the universe

S univ

S univ

3
kJ
kJ 1141 kJ

0.05 m

0
.3175
7
.
743

kg K
kg K 293 K
3

0.14749

kg

kJ
1.38
K

The entropy change for both processes can be fount by adding together the entropy change from
Part (a) and Part (b):

S univ (a) and (b) 1.55 kJ


K

71

3.33
A schematic is given below
valve maintains
pressure in system
constant
v
T1 = 200 oC
x1 = 0.4
V = 0.01 m3
l

Mass balance
dm
m in m out m out
dt

Separating variables and integrating:


m2

m1

dm m

out

dt

or
t

m2 m1 m out dt
0

Entropy Balance:
Q
Q
dS
dS
Suniv m out sout surr m out sout
Tsurr dt sys
Tsurr
dt sys

Integrating with sout constant


Suniv m2 s2 m1s1 m2 m1 sout

Q
Tsurr

(1)

From steam tables:

72

kJ
s1 (1 x)u f xu g 0.6 2.3309 0.4 6.4323 3.9715

kg K
kJ
kJ
s 2 6.4323
is the same as sout 6.4323

s2
kg K
kg K
Thus Equation 1 simplifies to

Suniv m1 ( s 2 s1 )

(2)

Tsurr

Energy Balance to find Q :


dU

m out hout Q
dt sys

Integrating

dU

m 2 u 2

m1u1

m out hout Q dt hout m out dt Q dt

Substituting in the mass balance and solving for Q


Q m 2 u 2 m 1 u 1 m 2 m 1 h out

To find the mass in each state:


m3
v1 (1 x)v f xv g 0.6 .001 0.4 0.1274 0.051

kg
m3
v2 0.1274

kg

V
0.01 m 3
0.196 kg
m1 1
v1
m3
0.051

kg

V
and m2 2
v2

From the seam tables:

73

0.01 m 3
0.0785 kg
m3
0.1274

kg

kJ
u1 (1 x)u f xu g 0.6 850.64 0.4 2597.5 1549
kg
kJ
u 2 2595.3
kg

and

kJ
hout 2793.2
kg

Plugging in:
Q m2u2 m1u1 m2 m1 hout 228 kJ

Back into Equation 2


kJ
228
kJ

Q
kg
s 0

S univ m1 ( s 2 s1 )
0.196 6.4323 3.9715

Tsurr
s
kg K 473 K
s

Note: Vaporization is reversible if Tsurr = Tsys.

74

3.34
The process can be represented as:

V1 = 1 m3
P1 = 10 bar

Process

V2 = 10 m3
P2 = ?

T1 = 1000 K

Work out
Q=0

T2 = ?

State 1

State 2

Solving for the number of moles:


PV
n 1 1 120.3 mol
RT1

The maximum work is given by a reversible process. Since it is also adiabatic, the entropy
change of the system is zero:
T
P
s 0 cP ln 2 R ln 2
T1
P1
5
Since cP R
2
T2

T1

5/ 2

P2 nRT2

P1 V2 P1

Solving for T2 gives


nRT 5 / 2
1
T2

V2 P1

2/3

215 K

An energy balance on this closed system gives:


T2

T2

T1

T1

U = Q W W n c v dT n cP R dT

Solving for work, we get


3
6
W n RT2 T1 1.18 10 J
2

75

3.35
The maximum efficiency is obtained from a Carnot cycle. From Equation 3.32
T
n 1 C
TH

where temperature is in Kelvin. Hence,


n 1

298.15 K
0.614
773.15 K

76

3.36
The labeling described in Figure 3.8 will be used for this solution. First, a summary of known
variables is provided.

State
1
2
3
4

P (bar)
30
0.1

T (C)
500

From our knowledge of ideal Rankine cycles, the table can be expanded as follows
State
1
2
3
4

Thermodynamic
State
Superheated Steam
Sat. Liq. and Vapor
Sat. Liquid
Subcooled Liquid

P (bar)

T (C)

30
0.1
0.1
30

500

The saturation condition constrains state 3. First, start with the turbine. Since the rate of heat
transfer is negligible and the expansion occurs reversibly in an ideal Rankine cycle, the
following is known
s1 s2

and

W s m h2 h1

From the steam tables:


kJ
s1 7.2337

kg K
kJ
s2l 0.6492

kg K
kJ
s2v 8.1501

kg K
kJ
h1 3456.5
kg
kJ
h2l 191.81
kg
h2v

kJ
2584.6
kg

(500 C, 30 bar)

(sat. H2O at 0.1 bar)

(500 C, 30 bar)

(sat. H2O at 0.1 bar)

77

The quality of the water can be calculated as follows


s1 (1 x) s2l xs2v

By substituting the steam table data, we find


x 0.878

Now the quality can be used for the following expression


h2 (1 x)h2l xh2v

Substituting the steam table data and the quality calculated above, we get
kJ
h2 2292.7
kg

Therefore,
kJ
kJ

kg
W s 100 2292.7 3456.5 116.38 MW
s

kg
kg

At state 3,
kJ
h3 191.81
kg
m3
v3 0.001010

kg

(sat. H2O(l) at 0.1 bar)

Therefore,

Q C m h3 h2 210.09 MW

Since the molar volume of water does not change noticeably with pressure and the compressor is
adiabatic, the work required for compressing the fluid can be calculated as follows
W c m v3 P4 P3
m3
kg
W c 100 0.00101 30 105 0.1 105 Pa 3.02 10 5 W 0.302 MW
s
kg

78

Now we find the rate of heat transfer for the evaporator. For the entire Rankine cycle, Equation
3.84 gives
Q net W net Q C Q H W s WC 0

We have
Q H W s WC Q C

Using the values calculated above:


Q H 326.2 MW

Equation 3.84:
W net W S WC
W net 116.08 MW

The efficiency can be calculated with Equation 3.82


W

116.08 MW

net
0.356
326.02 MW
QH

79

3.37
To make the Rankine cycle more efficient, we need to increase the area that represents the net
work in Figure 3.8. This can be done in a variety of ways:

1. Increase the degree of superheating of steam in the boiler. This process is sketched in the
upper left hand Ts diagram below. This change reduces the moisture content of steam
leaving the turbine. This effect is desirable since it will prolong the life of the turbine;
however, if the steam is heated too high, materials limitations of the turbine may need to be
considered.
2. Lower the condenser pressure. Lowering the pressure in the condenser will lower the
corresponding saturation temperature. This change will enlarge the area on the Ts diagram,
as shown on the upper right below. Thus, we may want to consider lowering the pressure
below atmospheric pressure. We can achieve this since the fluid operates in a closed loop.
However, we are limited by the low temperature off the heat sink that is available.
increase the superheating of steam in the boiler

lower the condenser pressure

1new

wnet

wnet
4new

4
2new

3new

2new

2 stage turbine with reheat

increase the boiler pressure

Turbine 1
Turbine 2

T
1

6
2
wnet

wnet

4
4
3

80

3. Increase the boiler pressure. This will increase the boiler temperature which will increase the
area as shown on the bottom left Ts diagram.
4. We can be more creative about how we use the energy available in the boiler. One way is to
divide the turbine into two stages, Turbine 1 and Turbine 2, and reheat the water in the boiler
between the two stages. This process is illustrated below.
Rankine cycle with reheat
Ws

1
Fuel
Air

Ws
Turbine
2

turbine
1
2

4
QH

Boiler

QC

cooling
water

6
5

condenser

Wc
compressor

The pressure in Turbine 1 will be higher than the pressure in turbine 2. This process is
schematically shown on the bottom right Ts diagram. This process leads to less moisture
content at the turbine exit (desirable) and limits the temperature of the superheat (desirable).

81

3.38
The labeling described in Figure 3.8 will be used for this solution. First, a summary of known
variables is provided.

State
1
2
3
4

P (bar)
100
1

T (C)
500

From our knowledge of ideal Rankine cycles, the table can be expanded as follows

State
1
2
3
4

Thermodynamic
State
Superheated Steam
Sat. Liq. and Vapor
Sat. Liquid
Subcooled Liquid

P (bar)

T (C)

100
1
1
100

500

The saturation condition constrains state 3. First start with the turbine (state 2). Since the rate of
heat transfer is negligible and the expansion occurs reversibly in an ideal Rankine cycle, the
following is known
s1 s2

From the steam tables,


kJ
s1 6.5965

kg K
kJ
s2l 1.3025

kg K
s2v

kJ
7.3593

kg K

(500 C, 100 bar)

(sat. H2O at 1 bar)

The quality of the water can be calculated as follows


s1 (1 x) s2l xs2v

By substituting the steam table data, we find


x 0.874

82

States 1, 2, and 3 are constrained; therefore, the enthalpies can be determined using the steam
tables. The enthalpy of state 4 can be calculated from Equation 3.80:
h4 h3 vl P4 P3
m3

where vl v3 0.001043
since specific volume of liquids are insensitive to pressure
kg
changes. From the steam tables:

State

kJ
3240.8
kg
Note: h2 (1 x)h2l xh2v
h

kJ
2391.0
kg

kJ
417.44
kg

kJ
427.8
kg

Equation 3.84:

W net Q H Q C
Therefore,

W net m h1 h4 m h3 h2 m h1 h4 h3 h2

and

W net

h1 h4 h3 h2

100 103 kW

kJ
kJ
kJ
kJ
3240.8 417.5 427.8 2391.0

kg
kg
kg
kg

kg
m 116.3
s
Now consider the non-ideal turbine and compressor. Use the definition of isentropic
efficiencies:

W s actual
turbine
Ws reversible
W S actual turbine W S reversible turbine m h2 h1 reversible

83

WC reversible
WC actual

WC reversible m h4 h3 reversible

compressor
WC actual

compressor

compressor

Equation 3.84 is used to find the mass flow rate

W net W s WC

h4 h3 reversible

W net m turbine h2 h1

reversible
compressor

Substituting the enthalpy values from the table shown above and the efficiency values gives

kg
m 147
s
A higher flow rate is needed as compared to the reversible process.

84

3.39
The labeling shown in Figure 3.8 will be used for this solution. First, a summary of known
variables is provided.

State

Thermodynamic
State
Sat. Vapor

1
2
3
4

P (MPa)
1.7
0.7

From our knowledge of ideal Rankine cycles, the table can be expanded as follows
State
1
2
3
4

Thermodynamic
State
Sat. Vapor
Sat. Liq. And Vapor
Sat. Liquid
Subcooled Liquid

P (MPa)
1.7
0.7
0.7
1.7

The enthalpy of states 1 and 3 can be found using the NIST website. For states 2 and 4, use the
following expressions to find the enthalpies

h2 (1 x)h2l xh2v
h4 h3 vl P4 P3 h3 v3 P4 P3
To find x, use the fact that the turbine is isentropic.
s1 s2
s1 (1 x) s2l xs2v

From the NIST website:

J
s1 146.38
mol K
J
s2l 90.45
mol K
J
s2v 150.07
mol K

(sat. vapor at 1.7 MPa)

(sat. mixture at 0.7 MPa)

By substituting the above values into the entropy relationship, we find

85

x 0.938

Also, from the NIST website:


J
h1 36046
mol
J
h2l 18416
mol
J
h2v 35353
mol
J
h3 18416
mol
3
5 m
v3 6.954 10

mol

(sat. vapor at 1.7 MPa)

(sat. mixture at 0.7 MPa)

(sat. liquid at 0.7 MPa)

Substituting these values into the expression for h4 and h2 yields


J
h2 34302.9

mol
J
h4 18485.5
mol
Equation 3.82 states

rankine

h2 h1 h4 h3
h1 h4

J
J
J
J
36046
18485.5
18416

34302.9

mol
mol
mol
mol
rankine
J
J
18485.5
36046

mol
mol
rankine 0.095
This efficiency is significantly lower than a conventional power plant.

86

3.40
(a)
Using the information in the problem statement and our knowledge of ideal vapor compression
cycles, the following table can be created

State

Thermodynamic
P (MPa)
State
1
Sat. Mixture
0.12
2
Sat. Vapor
0.12
3
Superheated Vapor
0.7
4
Sat. Liquid
0.7
Note: Refer to Figure 3.9 to review labeling convention.
Equation 3.85 states
Q C n h2 h1
which upon combination with Equation 3.89 gives
Q C n h2 h4
From the NIST website:
kJ
h2 39.295
mol
kJ
h4 24.181

mol

(sat. vapor at 0.12 MPa)


(sat. liquid at 0.7 MPa)

Therefore,

mol
kJ
kJ

Q C 0.5
39
.
295
24
.
181

mol
mol 7.557 kW
s

(b)
The power input to the compressor can be calculated with Equation 3.86:

WC n h3 h2
Equation 3.87 states
s 2 s3

87

From the NIST website:


J
s3 s2 sat. vapor at 0.12 MPa 177.89
mol K
Now, state 3 is constrained.
kJ
h3 43.031
mol

J
0.7 MPa, s3 177.89

mol

Therefore,

mol
kJ
kJ
39.295
WC 0.5
43.031
1.87 kW

s
mol
mol

(c)
Equation 3.90 states:

kJ
kJ

24
.
181
39.295
mol
h h
h h
mol
COP 2 1 2 4
4.05
h3 h2 h3 h2
kJ
kJ
39.295
43.031
mol
mol

88

3.41
Using the information in the problem statement and our knowledge of ideal vapor compression
cycles, the following table can be created

State

Thermodynamic
P (MPa)
State
1
Sat. Mixture
0.12
2
Sat. Vapor
0.12
3
Superheated Vapor
0.7
4
Sat. Liquid
0.7
Note: Refer to Figure 3.9 to review labeling convention.
From Equation 3.90
COP

Q C h2 h1

W C h3 h2

The enthalpy of state 2 can be found directly from the NIST website, but the enthalpies of states
1 and 3 require the use of additional information.
kJ
h2 39.295
mol

(sat. vapor at 0.12 MPa)

For the process between state 2 and state 3,


s 2 s3
From the NIST website:
J
s3 s 2 177.89
mol K

(sat. vapor at 0.12 MPa)

Now, state 3 is constrained.


kJ
h3 43.031
mol

J
0.7 MPa, s3 177.89

mol K

The process between state 4 and state 1 is also isentropic.


J
s1 s 4 115.09
mol K

(sat. liquid at 0.7 MPa)

The quality of the R134a can be calculated as follows

89

s1 1 x s1l xs1v

where
J
s1l 90.649
mol K
J
s1v 177.89

mol K

(sat. R134a(l) at 0.12 MPa)


(sat. R134a(v) at 0.12 MPa)

Therefore,

J
J
J
1 x 90.649
115.09
x177.89

mol K
mol K
mol K

x 0.280
Using the quality of R134a, the enthalpy of state 1 can be calculated as follows
h1 1 x h1l xh1v

From the NIST website:


kJ
h1l 17.412
mol
kJ
h1v 39.295

mol

(sat. R134a(l) at 0.12 MPa)


(sat. R134a(v) at 0.12 MPa)

Therefore,

kJ
kJ
h1 1 0.2817.412
0.28 39.295

mol
mol

kJ
h1 20.74
mol
Now, everything needed to calculate COP is available. Using Equation 3.90,
kJ
kJ
39.295
20.74

h h
mol
mol 4.97
COP 2 1
h3 h2
kJ
kJ
43.031
39.295

mol
mol

90

Is this modification practical?


No. An isentropic turbine adds significant level of complexity to the cycle. Turbines are
expensive and wear over time. Furthermore, the real turbine added to the cycle will not be 100%
efficient, so the COP will not increase as much. The cost of the turbine is not justified by the
increase in COP.

91

3.42
The subscript h refers to the hotter cycle, while c refers to the cooler cycle. The
(a)
The flow rate of the cooler cycle can be found by performing an energy balance on the
condenser/evaporator shared between the two cycles. An energy balance shows:
Q H , c Q C , h Q

Written in terms of enthalpy, the equation is


n c h8 h7 n h h2 h1
We can find the enthalpies for positions 2 and 8 directly from the thermodynamic tables because
the fluid is saturated in at these positions. To find the other enthalpies we must use the following
relationships:
s 6 s7
h1 h4
Using the NIST website:
J
h1 24181
mol
J
h2 40967
mol
J
h7 41510
mol
J
h8 21099
mol

(sat. R134 liquid at 0.7 MPa)


(sat. R134 vapor at 0.35 MPa)
J
)
(R134 vapor at 0.35 MPa with s 177.89
mol K
(sat. R134 liquid at 0.35 MPa)

Now the flow rate can be calculated:

n c

nc

J
J
0.5 mol/s 40967
24181

mol
mol

J
J
41510
21099

mol
mol

mol
0.411
s

92

(b)
Performing an energy balance we find

Q C n c h6 h5
We can also use the following relationship
J
h5 h8 21099
mol
From the NIST website:
J
h6 39295
mol

(sat. R134 vapor at 0.12 MPa)

Therefore,

mol
J
J
Q C 0.411
21099
39295
7.48 kW

s
mol
mol

(c)
The power input is calculated as follows:
WC , total WC , c WC , h

where
WC , c nc h7 h6
WC , h n h h3 h2

We have all of the required enthalpies except h3. State 3 is constrained because
J
s3 s 2 175.95
mol K
P3 0.7 MPa

(sat. R134a vapor at 0.35 MPa)

From the NIST website:


J
h3 42428
mol

J
)
(R134a vapor at 0.7 MPa with s 175.95
mol K

Now compute the power for each unit:

93

J
J
J
J
39295
40967
WC , total 0.411 mol/s 41510
0.5 mol/s 42428

mol
mol
mol
mol

W C , total 1.64 kW

(d)
The coefficient of performance is calculated using the following equation:
COP

Q C

WC , total

7.48 kW
4.56
1.64 kW

(e)
The COP for the cascade is 4.56, while the COP is 4.05 in Problem 3.40. The cascade systems
COP is 12.6% greater.

94

3.43
One design follows; your design may differ:

In order to cool a system to -5 C, the temperature of the fluid must be colder than -5 C so that
heat transfer will occur. We arbitrarily specify that the working fluid evaporates at -10 C.
Similarly, in order to eject heat to the 20 C reservoir, the fluid must condense at a temperature
greater than 20 C. Arbitrarily, we choose 25 C. One possible refrigeration cycle is presented
below.

valve
3

T
1
QH

evaporator
refrigerated
unit at -5 C

4
QC
20 C
reservoir

QH
2

1
3

condenser

Wc

QC

s
compressor

A number of fluids will work sufficiently for this system, but the design process will be
illustrated using R134a. For states 3 and 4, the pressure is constant, and for states 1 and 2, the
pressure is constant at a different value. From the NIST website, we find
P1 P2 0.201 MPa
(Tsat = -10 C)
P3 P4 0.665 MPa
(Tsat = 25 C)
(Note: The temperatures of each state are not constant. The listed saturation temperatures
are the temperatures at which the fluid evaporates and condenses.)

Now that the pressures are known, we can compute the required flow rate required in order to
provide 20 kW of cooling.
Q C n h2 h1
We can get h2 from the thermodynamic tables for saturated R134a. In order to find h1, we can
use the following relationship:
h1 h4

From the NIST website:


J
h2 40064
mol

(sat. R134 vapor at 0.201 MPa)

95

J
h4 23931
mol

(sat. R134 liquid at 0.665 MPa)

Now, calculate the required flow rate of R134a.


Q C
20,000 J/s

h2 h1
J
J
23931
40064

mol
mol

mol
n 1.24
s

96

3.44
The schematic of the process and the corresponding Ts is presented below
T = 5 oC
valve

Fridge
2
valve

QC,R

1
QH

evaporator

Wc

5 oC

T = -15 oC Freezer evaporator


QC,F
4

QH
5

high T
reservoir

-15 oC

QC,R
2

3
QC,F
4

condenser

Wc

s
compressor

A number of refrigerants will work for this system, but the design process will be illustrated for
R134a only. To define each state, we need to thermodynamically constrain each state. You
should note that the problem doesnt state what temperature the fluid condenses at. Therefore,
we can assume the temperature is 25 C. By using information in the Ts diagram and from the
NIST website, we find
P1 P2 0.34966 MPa
P3 P4 0.16394 MPa

P5 P6 0.66538 MPa

(T1=T2=Tsat = 5 C)
(T3=T4=Tsat = -15 C)
(T6 = 25 C)

Before solving for additional pressures and temperatures, we will list the known temperatures
and pressures.
State
1
2
3
4
5
6

Temperature (C)
5
5
-15
-15
25
25

Pressure (MPa)
0.24334
0.24334
0.16394
0.16394
0.66538
0.66538

States 4, 5, and 6 are completely constrained as confirmed by Gibbs phase rule. Now, we need
to find the liquid and vapor compositions of states 1, 2, and 3 to completely constrain the states.
Since the heat duties are equal for the refrigerator and the freezer, we have the following
relationship:
Q C , F Q C , R

97

which upon performing an energy balance becomes


h4 h3 h2 h1
Energy balances around the valves provide:
h1 h6
h2 h3
Substituting these relationships into the expression that equates the heat loads results in
h h6
h2 h3 4
2

From the NIST website:


J
h4 39755
mol
J
h6 23931
mol
Now constrain states 1, 2, and 3 by determining the enthalpies:
J
h1 h6 23931
mol
J
J
23931
39755

mol
mol 31843 J
h2 h3
mol
2
Technically, the states are now all constrained, but we would like also like to know the vapor and
liquid compositions of each state. The compositions of states 4, 5, and 6 are already known; we
can calculate the compositions of states 1, 2, and 3 as follows:
sat
h1 1 x1 h1sat
, l x1h1, v
sat
h2 1 x2 h2sat
, l x2 h2, v
sat
h3 1 x3 h3sat
, l x3 h3, v

From the NIST website,

98

J
h1sat
, l 21095 mol

J
h2sat
, l 21095 mol

J
h3sat
, l 18380 mol

J
h1sat
, v 40965 mol

J
h2sat
, v 40965 mol

J
h3sat
, v 39755
mol

Now, we can solve the composition of vapor for each state:


x1 0.143

x2 0.541

x3 0.630

The following table presents a summary of our results:


State

Temperature
(C)

Pressure
(MPa)

0.24334

0.24334

-15

0.16394

-15

0.16394

25

25

Phases

Saturated Liquid
and Vapor
Saturated Liquid
and Vapor
Saturated Liquid
and Vapor

Liquid
Vapor
Composition Composition

0.857

0.143

0.459

0.541

0.37

0.630

Saturated Vapor

0.66538

Superheated
Vapor

0.66538

Saturated Liquid

99

3.45
In this case, the working material is a solid. The four states of the magnetic material are shown
of the sT diagram below. Note the axis are shifted from the usual manner.

3
2

QH

QC
1

(a) The heat expelled by the cold reservoir can be approximated by:

qC TC s 2 s1
where T is the average temperature between states 1 and 2, which is approximately 1 K. Taking
values of S/R from the sT diagram, we get:
J
qC R1.9 1.3 5.0
mol K
(b) Similarly, the heat absorbed by the hot reservoir can be approximated by:

J
q H TH s 4 s 4 8.8R1.2 1.9 51
mol K
(b) The coefficient of performance is given by:

COP

qC
wC ,total

qC
0.11
q H qC

(d) The value of COP is much lower than a typical refrigeration process (COP= 4-6); in general,
refrigeration processes at these low temperatures are much less efficient.

F. Work is supplied to magnetize the material and to spin the wheel.


100

3.46
When the polymer is unstretched it is in a more entangled state. When stretched the polymer
chains tend to align. The alignment decreases the spatial configurations the polymer can have,
and therefore, reduces that component of entropy. If the process is adiabatic, the entropy of the
system cannot decrease. Consequently, its thermal entropy must increase. The only way this can
be accomplished is by increased temperature.

101

3.47
Assume the temperature is 298 K. The following data was taken from Table A.3.2:

Species

h f (kJ/mol)

g f (kJ/mol)

Cu2O (s)
O2 (g)
CuO (s)

-170.71
0
-156.06

-147.88
0
-128.29

The data listed above were used to create the next table using
s f

h f g f
T
Species

s f (kJ/mol K)

Cu2O (s)
O2 (g)
CuO (s)

-0.0766
0
-0.0932

Now the change in entropy of the reaction can be calculated in a method analogous to Equation
2.72

kJ
kJ

s rxn
vi s f 2 - 0.0766
4 - 0.0932

i
mol K
mol K

kJ

s rxn
0.22
mol
Does this violate the second law of thermodynamics? This problem shows that the entropy
change of the system is negative, but nothing has been said about the entropy change of the
universe. We must look at the change in entropy of the surroundings to determine if the second
law is violated. By looking at the enthalpies, we see that the reaction is exothermic, which
means that heat is transferred from the system to the surroundings. Therefore, the entropy of the
surroundings will increase during this reaction.

102

3.48
The temperature and pressure terms in the equation for entropy do not contribute anything to the
entropy change because they are constant. Therefore, the only remaining entropy contribution,
the randomness of the atomic arrangements, must be considered. The randomness does not
increase when CdTe forms. In pure crystals of Cd and Te, the location of each atom is known
because the crystal lattice constrains the atomic locations. In CdTe, the crystal lattice still
defines the location of each atom, so the randomness has not increased. Therefore, the change in
entropy is zero.
3.49
This argument is not scientifically sound. Morris is arguing that since evolution results in more
order, the second law of thermodynamics is violated, so evolution must be impossible. However,
the flaw in this argument is caused by ignoring the entropy change of the entire universe. The
second law states that the entropy of the universe will remain constant or increase for any
process. Morris argument was based on the entropy of the system undergoing evolution not
the entropy of the entire universe. A system can decrease in entropy if the entropy of the
surroundings increases by at least that much.

103

3.50
Entropy is related to the number of configurations that a state can have. The greater the number
of configurations, the more probable the state is and the greater the entropy. We can
qualitatively relate this concept to the possible hands in a game of poker, but it is more
interesting to quantify the results using some basic concepts of probability.

We consider a hand of poker containing 5 cards randomly draw from a deck of 52 cards.. There
are a finite number of permutations in which we can arrange a 52 card deck in 5 cards. For the
first card in the hand, we select from 52 cards, the second card can be any other card so we select
from 51 cards, the third card has 50 cards, and so on. Thus the number of permutations of 5
cards is:

P 52 51 50 49 48 311,875,200
However, we do not care the order in which the cards are dealt, so we must divide this number
by the number of ways we can come up with the same hand. We do this math in a similar way.
For a given hand there are five cards we can pick first, four we can pick second, . So the
number of ways we can make the same hand is:
N 5 4 3 2 1 120

The number of unique configurations can be found by dividing P by N . Thus, the cards can
display
C

P
2,598,960
N

or 2,598,960 unique configurations. To find the entropy of a given hand, we need to find out
how many of these unique configurations belong to the hand
Consider a four of a kind. There are 13 different possible ranks of four of a kind, one for each
number A, 2, 3, 4, 5, 6, 7,8, 9, 10, J, Q, K. The fifth card in the hand could be any of the other
48 cards. Therefore, the number of combinations of four of a kind is: 13 x 48 = 624
The probability of a four of a kind is: 624 / 2598960 = 1 / 4165 which is very unlikely. Thus the
entropy of this hand is low. In contrast, there are 1,098,240 to have a hand that has one pair;
therefore the probability of getting this hand is much greater, 1/2.4, and its entropy is high.
The probability of having nothing is 1/2 which represents the most likely hand in poker, or the
hand with the highest entropy. In fact, the rules of poker are defined so the hand of lower
entropy always beats the hand of higher entropy.

104

Chapter 4 Solutions
Engineering and Chemical Thermodynamics

Wyatt Tenhaeff
Milo Koretsky
Department of Chemical Engineering
Oregon State University
koretsm@engr.orst.edu

4.1.
(a)
Yes, the form of the equation is reasonable. This can be rewritten:
PA

RT
a
n
v A v A

It is equivalent to Pv RT , except the pressure term has been corrected to account for the ions
intermolecular forces. The coulombic forces between the gas molecules affect the system
pressure. This modification is similar to the van der Waals equation. Since we are limited to 1
parameter, we need to choose the most important interaction. Since net electric point charges
exert very strong forces, this effect will be more important than size.
(b)
The sign should be negative for a because the positively charged gas molecules repel each other
due to coulombic forces. Therefore, the system pressure increases, i.e.,:

PA Pideal
Coulombic forces are much stronger than van der Waals interactions, so a will be large much
larger than a values for the van der Waals EOS.
(c)
Coulombic repulsion is the primary intermolecular force present in the gas. Coulombic potential
energy is proportional to r-1. v is proportional to r3, so the coulombic potential goes as v-1/3.
Therefore,

1
3

a must have must have the following units to maintain dimensional homogeneity with the
pressure term:
a N J
v1/ 3 m 2 m 3
so

1/3
m mol
2

4.2
The attractive interactions are described by van der Waals forces. For both O2 and propane, the
dipole moments are zero. Therefore, the expression for the interactions reduce to

3 i j
2 r6

Ii I j

Ii I j

From Table 4.1:


I a 12.07 eV 1.933 10 11 erg

I b 10.94 eV 1.753 10 11 erg


Now obtain the requested expressions:

1.933 10

3 16 10 25 cm 3
aa
2
r6
aa

3.71 10 59
r

ab
ab

cm

cm

1.753 10

bb

1.933 10

erg

3 62.9 10 25 cm 3
bb
2
r6

5.20 10 58
r

cm

erg

3 16 10 25 cm 3 62.9 10 25 cm 3
2
r6

1.39 10 58

erg 1.933 10 11 erg

11
erg 1.933 10 11 erg
1.933 10

11

erg

11

11

erg 1.753 10 11 erg

1.933 10 11 erg 1.753 10 11 erg

erg 1.753 10 11 erg

1.753 10 11 erg 1.753 10 11 erg

(b)
Calculation:

5.20 10 58
3.71 10 59
6

cm erg
cm 6 erg
aa bb

r6
r6

aa bb

1.39 10 58

cm

erg
r
Note: Disregarded the positive value.
The value of

aa bb is equal to ab . The values are equal because the ionization energies are

similar.
(c)
An expression for the average intermolecular attraction in the mixture can be found using the
mixing rules
mix y a2 aa 2 y a yb ab yb2 bb
3.71 10 59 y a2 1.39 10 58 y a yb 5.20 10 58 yb2
mix
cm 6 erg
6
r

4.3
(a)
300 K, 10 atm. The intermolecular distance of molecules is greater at lower pressures.
Therefore, fewer intermolecular interactions exist, which cause less deviation from ideality.
(b)
1000 K, 20 atm. At higher temperatures, the kinetic energy of the molecules (speed) is greater.
The molecules interact less; thus, the compressibility factor is closer to unity.
(c)
Let subscript 1 denote BClH2 and 2 denote H2. For the mixture, we calculate the
compressibility factor as follows
B
z 1 mix
v

where

Bmix y12 B1 2 y1 y2 B12 y 22 B2


Since BClH2 is polar and H2 is non-polar and small
B1 Bmix B2
Therefore, the plot may look like the following.
quadradic, not
linear

zH2 close to 1
1

0.9

zBClH2 lowest

0.8

0.7

0.6
0

0.2

0.4

0.6

yBClH2

0.8

4.4
(a)
The intermolecular attractions and volume occupied by the styrene monomers will contribute to
the deviations from ideality. Since styrene monomers are essentially non-polar, the order of
importance is as follows

dispersion dipole - dipole induction


The van der Waals EOS is appropriate since it accounts for the occupied molar volume and
intermolecular forces, but it should be noted that more modern EOSs will give more accurate
results.
(b)
Use critical data to calculate the a and b parameters:
2

J
8.314
647.15 K

3
mol K
27 RTc
27
3.13 J m

2
64 Pc
64
39 105 Pa
mol
b

RTc

8 Pc

J
647.15 K
8.314
mol K

8 39 105 Pa

m3
1.72 10 4

mol

Now we can use the van der Waals EOS to solve for temperature.

RT
a

v b v2

J
8.314
T
mol K

5
10 10 Pa
3
4 m
3.0 10
1.72 10 4

mol
T 550.8 K 277.65 C

m3
mol

J m3
3.13
2
mol

3.0 10 4

m3
mol

The styrene will not decompose at this temperature.


(c)
The a parameter is related to attractive intermolecular forces. Dispersion is the controlling
intermolecular force in this system, and its magnitude is directly related to the size of the
molecules (polarizability component of dispersion). For the 5-monomer long polymer chain, a is
5 times the a value in Part (b). The b parameter is also related to the size of the molecule since it
accounts for the volume occupied by the molecules. Again, the b parameter for the reduced
polymer chain is 5 times the b value from Part (b).

J m3
a 15.65
2
mol
m3
b 8.6 10 4

mol

(d)
We must realize that if we initially believed there were 100 moles of styrene in the reactor, then
there can only be 20 moles of the 5-monomer long polymer chain. Therefore,

m3
v 1.5 10 3

mol
and
J m3

J
15
.
65
8.314
T
mol 2
mol K

10 10 Pa
2
3
3
3

m
m

4
3 m

1.5 10 3
8
.
6
10

mol 1.5 10

mol


mol
T 612.41K 339.26 C

Decomposition will occur.

4.5

There are many ways to solve this problem, and the level of complexity varies for each method.
To illustrate the principles in Chapter 4, two of the simplest solution methods will be illustrated.
Method 1. Polarizability of each atom
The polarizability of a molecule scales with the number of atoms; the polarizabilities of
individual atoms are additive. Using the first two molecules, solution of the following system of
equations
1 C 4 H 1 CH 4

2 C 6 H 1 C2 H 6
gives


cm

C 11.4 10 25 cm 3
H 3.65 10 25

Now, the polarizability of the chlorine atom can be found. For chloroform,
1 C 4 Cl 1 CCl 4

Therefore,

Cl 23.4 10 25 cm 3

The values of C3 H 8 , CH 3Cl , CH 2Cl2 , and CHCl3 are calculated with the polarizabilities found
above as follows, and compared to the values given.
Species
C3H8
CH3Cl
CH2Cl2
CHCl3

calculated
(x 1025 cm3)
63.4
45.8
65.5
85.3

reported
(x 1025 cm3)
62.9
45.6
64.8
82.3

% Difference
0.8
0.3
1.1
3.6

All agree reasonably well. Now, the polarizabilities of C 4 H10 and C 2 H 5Cl will be calculated.

H 10

4 11.4 10 25 cm 3 10 3.65 10 25 cm 3 82.1 10 25 cm 3

H 5 Cl

2 11.4 10 25 cm 3 5 3.65 10 25 cm 3 1 23.4 10 25 cm 3 64.45 10 25 cm 3

Method 2. Bond Polarizability


For this method, we calculate the molecules polarizabilities by adding the polarizability of each
bond, instead of the atoms. For the methane molecule
4 C H 1 CH 4

26 10 25 cm 3
6.5 10 25 cm 3
4

C H

To calculate the polarizability of a C-C bond, use ethane as follows:


6 C H C C 1 C 2 H 6

C C 44.7 10 25 cm 3 6 6.5 10 25 cm 3 5.7 10 25 cm 3

The C-C and C-H polarizability calculated above predict the given polarizability of propane well.
The polarizability of C-Cl bonds is calculable with the polarizability of chloroform.
4 C Cl 1 CCl 4
C Cl

105 10 25 cm 3
26.25 10 25 cm 3
4

The values of C3 H 8 , CH 3Cl , CH 2Cl2 , and CHCl3 are calculated with the polarizabilities found
above as follows, and compared to the values given.
Species
C3H8
CH3Cl
CH2Cl2
CHCl3

calculated
(x 1025 cm3)
63.4
45.8
65.5
85.3

reported
(x 1025 cm3)
62.9
45.6
64.8
82.3

% Difference
0.8
0.3
1.1
3.6

All agree reasonably well. This value predicts the polarizabilities of the other species in the table
reasonably well. Now, the polarizabilities of C 4 H10 and C 2 H 5Cl will be calculated.
1 C 4 H 10 3 C C 10 C H

H 10

3 5.7 10 25 cm 3 10 6.5 10 25 cm 3 82.1 10 25 cm 3

1 C 2 H 5 Cl 1 C C 5 C H 1 C Cl

H 5 Cl

1 5.7 10 25 cm 3 5 6.5 10 25 cm 3 1 26.25 10 25 cm 3 64.45 10 25 cm 3

Note both the atom method and the bond method yield identical results. More accurate values
for the polarizabilities can be calculated using more of the data given in the problem.

10

4.6
(a)
increases with molecular size. Therefore,

I S O
2

scales with magnitude of van der Waals interactions. Because these are non-polar, diatomic
molecules, only dispersion forces are present. Dispersion forces depend on the first ionization
potential and polarizability. Ionization energy is approximately equal for each molecule. The
polarizability scales with molecular size, so

I S O
2

(b)
increases with molecular size. Diethylether and n-butanol have the same atomic formula and
similar spatial conformations. Therefore, they should be about equal in size. Methyl ethyl
ketone has fewer atoms, but has two exposed electron pairs on the double-bonded oxygen. The
size of the molecular electron orbital of methyl ethyl ketone is approximately equal to the sizes
of diethyl ether and n-butanol, so

n - butanol diethyl ether methyl ethyl ketone


Methyl ethyl ketone and n-butanol are much more polar than diethyl ether due to their greater
asymmetry, so their values are greater than diethyl ethers. Now we must determine if there is
greater attraction in n-butanol or methyl ethyl ketone. scales with the magnitude of van der
Waals interactions. Since induction and dispersion forces are similar in these molecules, we
must consider the strength of dipole-dipole forces. There is greater charge separation in the
double bond of ketone, so

methyl ethyl ketone n - butanol diethyl ether

11

4.7
(a)
At 30 bar, the water molecules are in closer proximity than they are at 20 bar. Intermolecular
attractions are greater, so the magnitude of molecular potential energy is greater. The potential
energy has a negative value for attractive interactions. The molecular kinetic energy is identical
since the temperature is the same. Hence, the internal energy, the sum of kinetic and potential
energies, is less at 30 bar.
(b)
The key to this phenomenon is hydrogen bonding. At 300 K and 30 bar, isopropanol and npentane are both liquids. The hydrogen bonding and dipole-dipole interactions are present in
isopropanol, and dispersion is present in n-pentane. The intermolecular forces are greater in the
isopropanol, so the compressibility factor is smaller for isopropanol.

At 500 K and 30 bar, both species are gases. In the gas phase, hydrogen bonding does not play a
significant role. The dispersion forces in n-pentane are stronger than the dipole-dipole forces of
isopropanol. Therefore, the compressibility factor is smaller for n-pentane.

12

4.8
(a)
Ideal: For ideal NH3, the compressibility factor is equal to one. For real NH3, the strong
intermolecular forces (dipole-dipole and dispersion) cause the molar volume to decrease. They
outweigh the volume displaced by the physical size of NH3; thus, z will be less than one.
(b)
Internal energy will be greater for the ideal gas. In the real gas, intermolecular attractions are
present. Internal energy value is the sum of potential and kinetic energies of the molecules. The
absolute values of the kinetic energies are identical at identical temperature: however, the
potential energy decreases for real NH3 due to attractive interactions - so the internal energy is
less for the real NH3.
(c)
The entropy will be greater for the ideal gas. Entropy is a measure of possible molecular
configurations or randomness. Ammonia has an electric dipole in which positive and negative
charge are separated. The intermolecular forces in the real gas cause the molecules to align so
that the positive charge in one molecule is adjacent to a negative charge in a neighboring
molecule to reduce potential energy. Therefore, fewer possible configurations exist, which
creates less randomness and lower entropy.

13

4.9
(a)
We can determine which case has the higher compressibility factor by comparing the molar
volumes at constant T and P. With Ne, very weak intermolecular attractions are present, so
volume displacement becomes important. The compressibility factor will be slightly greater than
one. In NH3, the strong intermolecular attractions decrease the molar volume, so z is less than
one. The compressibility factor is greater for Ne.
(b)
Entropy is a measure of randomness. Both species are gases at these conditions. The
intermolecular attractions present in NH3 reduce the number of possible configurations. The
weak forces present in Ne have a much smaller effect. However, NH3 is asymmetrical, while Ne
is symmetrical. The asymmetry of NH3 results in more possible configurations that NH3 can
have. Therefore, it is difficult to qualitatively show for which case the entropy is greater. Since
both species are gases, intermolecular interactions are relatively weak, and we can guess that
entropy is greater in NH3.

14

4.10
(a)
To find the average distance between the two atoms of Ar, we can find the volume that each
atom occupies. The molar volume can be found from the compressibility factor. At 300 K and
25 bar,

Tr 1.99
Pr 0.513
From the generalized compressibility charts:
z 0.9859

Therefore,

v 0.9859

J
8.314
300 K
mol K

25 105 Pa

9.84 10

m3

mol

m3
v 1.63 10 27

atom
The second number was found by dividing the molar volume by Avogadro's number. To
estimate the distance between each atom, we note that the distance between molecules can be
related to the volume by:

r3 v
Consider the geometry shown below:
3

Ar

Ar

A rough estimate of r is

r 1.63 10 27 m 3

1 / 3 1.18 109 m
15

r 11.8 A

(b)
The potential energy due to gravity can be calculated as follows
G

G m 2Ar
r

m2
where G is the gravitation constant G 6.67 10 11
and mAr is the mass of an argon
2

kg s

atom. The mass of an argon atom is calculated as follows

1 kg
39.948 g Ar
1 mol
kg

6.633 10 - 26
m Ar

23

atom
1 mol Ar 6.023 10 atoms 1000 g
Using the distance calculated in Part (a), we get

G 2.49 10 -52 J
(c)
Equation 4.13 quantifies the potential energy due to London interactions
Ar Ar

3 Ar Ar
2 r6

I Ar I Ar

I Ar I Ar

From Table 4.1

I Ar 15.76 eV 2.52 10 -4 erg

Ar 16.6 10 25 cm 3

Using the distance calculated in Part (a) r 1.18 10 7 cm , we get

Ar Ar 1.93 10 10 erg 1.93 10 17 J


(d)
The potential energy due to London interactions is around 10 35 times greater than the potential
energy due to gravity. Clearly, London interactions are much more important, and gravitational
effects can be neglected.

16

4.11
We want to compare the Lennard-Jones potential to one with an exponential repulsion term. As
provided in the text, Equation 4.19, the Lennard-Jones potential is

12 6
4
r
r
To simplify further analysis, we can rewrite the equation in dimensionless quantities:

1
1

4 r *12 r *6

r*

where

We want to compare this to an exponential repulsive function


*
exp

c2
1

c1 exp

r * r * 12

We have two adjustable parameters, c1 and c2, to match the first (LJ) potential to the second
(exp) potential. We need to choose reasonable criteria to specify. For this solution we choose
equal well depths and equal values at =1. Other choices may be just as valid; you should
realize that you have two parameters to fit and so must specify two features.
Using the above criteria, we iterate on a spreadsheet, to get the solution:
c1 = 143,000 and

c2 = 11.8

This solution is shown at two magnifications in the plots on the following page:

17

Potential functions
0.3
0.2

Lennard-Jones
exponential

0.1
0
0.9

1.1

1.2

1.3

1.4

1.5

1.6

1.7

1.8

1.9

2.1

-0.1
-0.2
-0.3

Potential functions
20

Lennard-Jones

15

exponential
10

0
0.7

1.3

1.6

1.9

-5

We draw the following conclusions:


1. The most stable configuration (the bottom of the well) occurs at a greater separation for
the exp model.
2. The Lennard-Jones potential increases more steeply at small radii, i.e., it behaves more
like the hard-sphere potential.
3. The two models are in reasonable qualitative agreement

18

4.12
(a)
The bond strength of a sodium ion can be viewed as the amount of energy it would take to
remove the sodium ion from the crystal lattice. The interaction between the chlorine and sodium
ions is Coulombic attraction.

QNa QCl
4.803 10 10 esu 4.803 10 10 esu
6
5.01 10 11 erg
r
2.76 10 -8 cm
31.3 eV

(b)

QNa QCl
Q Q
12 Na Na
r
r

5.01 10 11 erg 12

4.803 10

2.089 10 11 erg 13.0 eV

esu 4.803 10 10 esu


3.90 10 -8 cm

10

(c)

QNa QCl
Q Q
Q Q
12 Na Na 8 Na Cl
r
r
r
4.803 10 10 esu 4.803 10 10 esu
2.089 10 11 erg 8
4.78 10 -8 cm
1.77 10 11 erg 11.1 eV
6

(d)

QNa QCl
Q Q
Q Q
Q Q
12 Na Na 8 Na Cl 6 Na Na
r
r
r
r
10
4.803 10 esu 4.803 10 10 esu
11
1.77 10 erg 6
5.52 10 -8 cm
7.37 10 12 erg 4.60 eV
6

19

4.13
The boiling points of the halides depend upon the strength of intermolecular attractions. The
stronger the intermolecular attraction, the higher the boiling point. Dispersion and dipole-dipole
interactions are present in all five species listed. The magnitude of the dipole-dipole interactions
is similar so the pertinent intermolecular force in these molecules is dispersion. The molecular
size increases from left to right. Polarizabilities are greater in larger molecules, which manifests
in larger dispersion forces. Therefore, the boiling point increases from left to right.

20

4.14
van der Waals forces hold the Xe atoms together in a molecule of Xe2. The potential energy can
be quantified with the Lennard-Jones potential function. The bond length is the r value where
the potential is a minimum. From Table 4.2:

k
and

229 K

4 .1 A

We start with Equation 4.19: to get


12 6
4
r
r
Differentiation with respect to r yields
12 12 6 6
d
4 0
dr
r r
r r
where we set this derivative equal to zero to find the minimum. Solving gives
1


2
r
6

or
r 6 2 1.12 4.60 A

21

4.15
The data in the following table were taken from Table A.1.1

Species

Tc K

He
CH4
NH3
H2O

5.19
190.6
405.6
647.3

Pc 10 -5 Pa
2.27
46.00
112.77
220.48

The van der Waals a parameter can be calculated using Equation 4.39.
27 RTc 2
a
64 Pc
The van der Waals b parameter can be calculated using Equation 4.40.
b

RTc
8 Pc

Using the data table and equations listed above, the following table was created; the values of
dipole moment and polarizability reported in Table 4.1 are also included.

Species
He
CH4
NH3
H2O

J m3
a

2
mol
0.00346
0.230
0.425
0.554

m3
b 105

mol
2.38
4.31
3.74
3.05

[D]

[cm x 1025]

0
0
1.47
1.85

2.1
26
22.2
14.8

The values of a for helium is two orders of magnitude less than the other species since it only has
weak dispersion forces (small atom, small The values of a for methane, ammonia, and water
are of the same magnitude because the sums of the intermolecular attractions for each molecule
are similar. All three molecules have comparable dispersion forces; although slightly weaker in
the ammonia and water. However, unlike methane, these two molecules also have dipole-dipole
and induction forces. In fact, the strong dipole in water gives it the largest value
The values of b are all of the same magnitude, as expected since b scales with size according to
the number of atoms in the molecule.
Size of Molecules:
Value of b:

CH 4 NH 3 H 2O He
bCH 4 bNH 3 bH 2O bHe

22

4.16
The van der Waals b parameter can be calculated using Equation 4.40.

RTc
8 Pc

Critical point data can be found in Table A.1.1. The following table was made:
Tc K

Species

Pc 10 -5 Pa

CH4
C6H6
CH3OH

190.6
46.00
562.1
48.94
512.6
80.96
J
Note: Used R 8.314
for calculation of a.
mol K

m3
b 10 5

mol
4.306
11.94
6.58

The equation from page 186 can be rewritten to calculate .

3b
2N A

The table listed below was created using this equation, and data from Table 4.2 are included
along with the percent difference.
Species

1010 m

CH4
C6H6
CH3OH

3.24
4.56
3.74

Table 4.2 Value


1010 m
3.8
5.27
3.6

23

Percent
Difference
14.7
13.5
3.8

4.17
The van der Waals a parameter can be calculated using Equation 4.39. The values for the above
equation were taken from Table A.1.1, and the following table was made:

Tc K

Species

J m3
a

2
mol
0.2303
1.883
0.9464

Pc 10 -5 Pa

CH4
C6H6
CH3OH

190.6
46.00
562.1
48.94
512.6
80.96
J
Note: Used R 8.314
for calculation of a.
mol K
The equation from page 187 can be used to find C6.
a

2N a2C6

3 3
3a 3
C6
2N a2
The values can be found in Table 4.2. The following table can now be created
Species

1010 m

CH4
C6H6
CH3OH

3.8
5.27
3.6

C 6 10 77 J m 6
1.66
36.3
5.82

To compare the values obtained from Equation 4.13, first calculate the potential energy with
Equation 4.13:

3 i j
2 r6

Ii I j

Ii I j

II
3
i j i j
2
Ii I j

Therefore

C6

We obtain the following values using this equation, and the corresponding percent differences
were calculated.

24

Species
CH4
C6H6
CH3OH

C 6 10 77 J m 6
Page 187
Equation 4.13
1.66
1.021
36.3
12.0
5.82
1.36

Percent Difference
63
202
328

The values for the van der Waals a constant have the correct qualitative trends and order of
magnitude; however, those values predicted from basic potential theory vary significantly from
corresponding states. The basic potential result presented in the text assumes that the species are
evenly distributed throughout the volume. It does not take into account the structure given to the
fluid through intermolecular forces. In fact, a more careful development includes a radial
distribution function, which describes how the molecular density of the fluid varies with r. The
radial distribution function depends on pressure and temperature of the fluid.

25

4.18

RT
a

v b v2

Redlich-Kwong

RT
a
1/2
v b T v(v b)

Peng-Robinson:

RT
a(T )

v b v(v b) b(v b)

van der Waals:

As you may have discovered, these equations are largely empirical with no theoretical
justification. They simply represent experimental data better. We can use our knowledge of
intermolecular forces, however, to explain why these may work better.
If we compare these equations to the van der Waals equation, we note that the first term
on the right hand side of all three equation is identical; we accounted for this term as a correction
for finite molecular size (or alternatively repulsive interaction due to the Pauli Exclusion
Principle). This form represents a hard sphere model.
The second term, that which deals with intermolecular attraction, is different in all three
models. Both of the later equations include a temperature dependence in this term. We have
seen that if attractive forces depend on orientation (dipole-dipole), they fall off with T as a result
of the averaging process (recall discussion of Equation 4.11).
Another explanation goes as follows: as T increases, the molecules move faster, reducing
the effect of intermolecular forces. If we say that the potential energy between two molecules
depends on the amount of time that they spend close to each other, then it would be inversely
related to velocity (The faster molecules are moving, the less time they spend in the vicinity of
other species). In this case, the correction term would go as V-1, where V is the molecular
velocity. If we relate molecular velocity to temperature
2
1
2 mV

32 kT

Therefore the correction term goes as T-1/2, as shown in the Redlich-Kwong equation.
The inclusion of a "b" term in the second term may help relax van der Waal's "hard
sphere" model with a more realistic potential function, i.e., something closer to a Lennard Jones
potential (Figure 4.8) than the Sutherland potential (Figure 4.7) upon which the van der Waals
equation is based. It makes sense that this should be included in the force correction since this is
taking into account repulsive forces. One example of a more detailed explanation follows:
If we look at the Redlich-Kwong equation, it says that if we have 2 species with the same
attractive strength (same a -> same magnitude of van der Waals forces), the larger species will
have less of an effect on P. The following sketch illustrates how 2 species could have the same
van der Waals attractive forces:

26

- +

London
= London
+
Dipole

Species 2
larger and non-polar
(larger b, same a)

Species 1
smaller and polar
(smaller b, same a)

In the case above, when the two species are the same distance apart, they have the same
attractive force; however, the smaller species (1) can get closer before its electron cloud
overlaps. Thus it has more opportunity for attractive interactions than the larger species. The
Peng-Robinson equation exhibits the most complicated form in an attempt to better fit
experimental data.

27

4.19
(a)
Work is defined as follows

W PdV
Substitution of the ideal gas law for P yields
W

nRT
dV
V

1 L
J
W 2.0 mol 8.314
1000 K ln

mol K
10 L

W 38.29 kJ
(b)
Instead of substituting the ideal gas law into the definition of work, the Redlich-Kwong equation
is used:
v2

W n

v1

RT
a
dv

1
/
2
v b T v (v b)

Substituting

J
R 8.314
mol K
T 1000 K
n 2 mol
J K1/2 m 3
a 14.24

2
mol

3
5 m
b 2.1110

mol
m3
v1 0.0005

mol
m3
v2 0.005

mol

28

and evaluating the resulting formula gives


W 37.35 kJ

(c)
Energy balance:

u q w
Since the process is reversible

q Ts
and
w u Ts

To use the steam tables conveniently, we need the initial and final pressures. We can calculate
these with the Redlich-Kwong EOS:
P1 1.65 MPa
P2 15.6 MPa

From the steam tables:

kJ
u1 3522.6
kg
kJ
u 2 3462.2
kg

kJ
s1 8.101

kg K
kJ
s2 7.00

kg K

Therefore,

kJ
kJ
kJ
kJ
w 3462.2 3522.6 1000 K 7.00
8.101


kg
kg
kg

K
kg

kJ
w 1040
kg
W 37.5 kJ

The answers from the three parts agree very well. Part (a) is not as accurate as Part (b) and Part
(c) because water is not an ideal vapor. The value from Part (b) is 1.1 % smaller than the value
from Part (c). Clearly, the Redlich-Kwong EOS or steam tables are appropriate for this
calculation.

29

4.20
First, we can obtain an expression for the compressibility factor with the van der Waals equation.

1
a
Pv
v
a

b RTv
RT v b RTv
1
v
To put this equation in virial form, we can utilize a series expansion:
1
1 x x 2 x 3 ...
1 x
Therefore,
2

1
b b
1 ...
b
v v
1
v
and
a

2
Pv
RT b

1
...
v
RT
v
This expression can also be expanded in pressure. From Equation 4.58, we know that
B'
C'

B
RT
C B2

RT 2

where
Pv
1 B' P C ' P 2 ....
RT
Substituting B and C found above, we get
B'

bRT a

RT 2

and C '

2abRT a 2

RT 4

Therefore,

30

bRT a
Pv
P 2abRT a P 2 ....
1
RT 2

RT
RT 4

31

4.21
Using the virial expansion, the pressure can be written as follows

1 B C
P RT

...

v v 2 v3
The virial expansion in pressure is
P

RT
1 B' P C ' P 2 ...
v

If we substitute the first expression into the second, we obtain


2

1 B C
1 B C
RT
1 B C

...

C ' RT

RT
1 B ' RT
2
v
v v 2 v 3
v
v
v3
v v 2 v3

If we combine like terms on the right side and set them equal to terms on the left, we find

B B' RT
C BB' RT C ' RT 2
Substitute the expression for B into the expression for C and solve for C:
C'

CB

RT 2

and
B'

B
RT

32

4.22
At the critical point we have:

Pc

RTc
a

vc b Tc vc2

(1)

RTc
2a
P

0
2
v Tc
vc b Tc vc3

(2)

2
2 RTc
6a
P

2 0
3
Tc vc4
v

vc b
Tc

(3)

If we multiply Equation 2 by 2 and Equation 3 by (v-b) and add them together:


0

4a
vc3

6avc b
vc4

this can be solved to give:


vc 3b
If we plug this back into Equation 2 and solve for a, we get:
a

9
vc RTc2
8

Finally, if we plug this back into Equation 1 we can solve for the Berthelot constants in terms of
the critical temperature and the critical pressure:

27 R 2Tc2
64 Pc

and

RTc
8Pc

33

4.23
Before we can find the reduced form, we need to find expressions for a and b in terms of critical
point data (Problem 4.22). We find

vc 3b

(1)

27 R 2Tc3
a
64 Pc
RTc
b
8 Pc

(2)
(3)

The Berthelot equation is


P

RT
a

v b Tv 2

First, substitute Equation 1 into the first term on the right-hand side and rearrange to get
RT
a
P b
3vr 1 Tv 2
Now, substitute Equation 3:
P

8 PcTr
a

3v r 1 Tv 2

Substitute Equation 2:

8 PcTr

3v r 1

27 R 2Tc3
64 Pc
Tv 2

From Equation 3:
R 2Tc2 64 Pc2 b 2
Substituting this result, we get
P

8PcTr 27 PcTc b 2

3v r 1
Tv 2

34

Finally, substitute Equation 1:


P

8PcTr 3PcTc vc2

3v r 1
Tv 2

This can be rewritten as


Pr

8Tr
3

3vr 1 Tr vr2

35

4.24
(a)
The virial equation is:
z

Pv
D
B C
1 2 3 ...
RT
v v
v

(1)

We can rewrite this equation:


C
Pv
1v B ...
RT
v

z 1v

(2)

PvT data from the steam tables are given in the steam tables and corresponding values of (z-1)v
and 1/v can be calculated. An illustrative plot of (z-1)v vs. 1/v for T = 300 oC is shown below.
Plot to determine the second virial coeff at 300 C
-115

(z -1)v (cm3 / mol)

-120

-125

(z -1)v
-130

1.5

0.5

-135

1/v (mol/l )

We see that at pressures above 1.5 MPa (15 bar) we see that the plot reaches a constant value of
around -118 cm3/mol. We may choose to report this value as B. A more careful examination of
Equation 2 suggests another possibility.
If we plot (z-1)v vs. 1/v, we should get a straight line at low to moderate pressures with an
intercept equal to the second virial coefficient, B, and a slope equal to the third virial coefficient,
C. At very low pressures, the curve is indeed linear and gives an intercept of -140 cm3/mol, as
shown on the next page. The slope of this region would yield the third virial coefficient, C.
Can you calculate C?

36

The first value uses much more data while the second method uses limited data in a better range.
What value would you be more apt to use? Water exhibits this odd general behavior at all
temperatures.
Plot to determine the second virial coeff at 300 C
-115

(z -1)v (cm3 / mol)

-120

y = 177.058x - 140.373
-125
(z -1)v
-130

lin

1.5

0.5

-135

1/v (mol/l )

If we do this at other values of T, we can compile a set of second virial coefficients vs.
temperature and get an idea of the differences in the two approaches. Temperatures of 200, 250,
300, 350, 400, 500, 600, 700, 800, 900, 1000, and 1200 oC analyzed in this manner. The results
are reported in the table and figure below. Also reported were the average value and the value at
the lowest pressure used. Note that the average value is indicative of the 1st method above while
the value at the lowest pressure used is indicative of the second value used.
T
200
250
300
350
400
500
600
700
800
900
1000
1200

B (Level)
-214
-156
-118
-89
-72
-49
-34
-24
-17
-13
-9
-3

B (Linear)
-226
-173
-140
-102
-103
-83
-73
-68
-66
-51
-50
-51

37

B (AVG)
-215
-157
-120
-92
-76
-54
-39
-30
-24
-19
-16
-11

B(1st value)
-220
-168
-134
-99
-95
-75
-63
-57
-54
-52
-52
-52

Second Virial Coefficient of Water by 4 Methods


0

B (water) cm 3 /mol

-50

-100
B (average)
B (lowest P)

-150

B (Level)
-200

B (Linear)
CRC
1500

1250

1000

750

500

250

-250

T (K)

For comparison, values of -142.2 cm3/mol and -7160 cm6/mol2 are reported for B and C,
respectively, at 260 oC1. Values of B reported in the CRC are also shown on the summary plot.
They agree most closely with the first (level) method.
(b)
There are several alternatives how to solve this problem, each of which comes up with a slightly
different result:

Alternative 1:
Rewrite the virial equation:

1 BH 2 O
P RT
...
v2

v
Take the derivative:

1 2 BH 2 O
P

0 RTc 2
v
v Tc
v 3c

so
cm3
v
BH 2O c 28

2
mol

J.H. Dymond and E.B. Smith, The Virial Coefficients of Pure Gases and Mixtures, A Critical Compilation, Oxford
University Press, Oxford, 1980.

38

Alternative 2:
From the virial equation:
cm3
Pv

BH 2O c c 1vc 43.2

mol
RTc

Alternative 3:

1 BH 2 O C H 2 O
P RT

...
v2
v3

v
so

1 2 BH 2O 3C H 2O
P

0 RTc 2
3
v
v Tc
v
v 4c
c

(1)

2P
6B
12C H 2O

0 RTc 2 H 2O
4
5
v 2
v3
vc
vc

Tc

(2)

and

Multiply Equation 1 by (4/vc) and add to Equation 2 to get:

2
v 3c

2 BH 2 O
v 4c

so
cm3
BH 2O vc 56

mol

39

4.25
A trial-and-error is the easiest method for solving this problem. The general method is as
follows

1. Guess P sat
2. Calculate values of molar volumes that result at the chosen P sat : vlow , vmid , vhigh
3. Find areas under the curves with the following expressions

v mid

sat

f v dv

v low

f v P

v high

sat

dv

v mid

[ f v represents the particular EOS implicit in molar volume being used.]


4. If the values of the expressions are not equal, repeat the process until they are.
(a)

1. Guess P sat :
P sat 6 bar

2. Calculate molar volume solutions for:


RT
a

P sat
1
/
2
v b T v (v b )
J K1/2 m 3

J
41
.
851

8.314
2
363.15 K
mol

mol
K

6 105 Pa

m3

3
v 0.0001
363.15 K 1 / 2 v v 0.0001 m

mol
mol

m3
m3
m3

0
.
000551

0
.
00459
vlow 0.000152
v
v

high
mid

mol
mol
mol
3.
0.000551

a
sat RT
dv 1157.19
P


v b T 1 / 2 v (v b )

0.000152
0.00459

a
RT
P sat dv 1325.78

v b T 1 / 2 v (v b )

0.000551

40

4. Repeat this process until the areas are equal. This occurs approximately at
P sat 6.38 bar

(b)

1. Guess Psat :
Psat 6 bar
2. Calculate molar volume solutions for
P sat

RT
a T

v b v(v b) bv b

using
m3
J m3
a 2.066
, b 9 10 6

, T 1.188
2
mol
mol
m3
m3
m3

0
.
000551

0
.
00459
vlow 0.000152
v
v

high
mid

mol
mol
mol
3.
sat

RT
a T
P v b v(v b) bv b dv 1568.48

0.000152
0.000551

RT
a T
sat
v b v(v b) bv b P dv 1100.19

0.000551
0.00459

4. Repeat this process until the areas are equal. This occurs approximately at
P sat 4.945 bar

The value calculated with the Redlich-Kwong EOS is 11.9% greater than the measured value of
5.7 bar. The Peng-Robinson EOS results in a value that is 13.25% less than the measured value.

41

4.26
First, calculate a , b , and T . From Table A.1.1:

Pc 48.84 Pa

Tc 305.4 K

w 0.099

Now we can calculate the required parameters.

J
305.4 K
0.45724 8.314

mol K

48.84 105 Pa

J m3
0.6036

2
mol

J
0.07780 8.314
305.4 K
mol K

4.045 10 5
b
5
48.84 10 Pa

T 1 0.37464 1.542260.099 0.269920.0992

T 1.12

m3

mol

243.15 K

3
05.4
K

Now, we can find the three solutions to the Peng-Robinson EOS:


P sat

RT
a T

v b v(v b) bv b

Using the values calculated above and


P sat 10.6 105 Pa
J
R 8.314
mol K
we get
m3
4
v 7.199 10 5
, v 2.167 10
mol

m3
3
3 m

,
v
1
.
578
10

mol
mol

The molar volume of saturated ethane liquid is the smallest value from the list above, while the
molar volume of saturated ethane vapor is the largest value. Therefore,

42

g
36.0694
3

MW ethane
mol 1 m
0.501 g

liq
3
liq
3
3

3 100 cm
v
cm
5 m
7.199 10

mol
g
36.0694
3

MW ethane
mol 1 m
0.0229 g

vap
3
3 100 3 cm 3
v vap
cm
3 m
1.578 10

mol
Both of the densities calculated with the Peng-Robinson EOS are larger than the reported values.
The liquid density is 7.05% larger, and the vapor density is 18.7% greater.

43

4.27
(a)
We will use the Redlich-Kwong EOS in order to obtain an accurate estimate. First, calculate the
a and b parameters:

J m 3 K1 / 2
0.42748R 2Tc2.5
1.56

Pc
mol 2

m3

mol
(Note: Critical data for nitrogen obtained in Table A.1.1.)
0.08664 RTc
2.69 10 5
b
Pc

Now use the EOS to find the molar volume:


P

RT
a

1
/
2
v b T vv b

Assume the temperature of the gas is 22 C, substitute values, and find the molar volume with a
solver function:
m3
v 1.97 10 4

mol
Now calculate the total volume of gas in the 30,000 units:
Vtotal 30000 units 43 liter/unit 1.29 10 6 liters 1290 m 3
Therefore, the number of moles is:
n

1290 m 3
6.55 10 6 mol
3
m
1.97 10 4

mol

and the mass

m 6.55 10 6 mol 0.02801 kg/mol 1.83 105 kg


Now, calculate the value of the gas:

Value $6.1 / kg 1.83 105 kg $1,116,300

44

If we use the ideal gas law, we find:

J
295 K
8.314
RT
mol K
1.98 10 4

v
P
12400000 Pa

m3

mol

Following the steps above,


Value $1,113,200
The value calculated using the ideal gas law is $3100 less than the value calculated using the
Redlich-Kwong EOS.
(b)
First, calculate the a and b parameters:

J m 3 K1 / 2
0.42748R 2Tc2.5
1.74
a

Pc
mol 2

m3

mol
(Note: Critical data for oxygen obtained in Table A.1.1.)
b

0.08664 RTc
2.21 10 5
Pc

Assume the temperature of the gas is 22 C, substitute values, and find the molar volume with a
solver function:
m3
v 1.53 10 4

mol
Following the steps presented in Part (a), we find that

Value $2,428,000
With the ideal gas law, we find
m3
v 1.64 10 4

mol
which provides

Value $2,265,000

45

The value found with the ideal gas law is $163,000 less than the value found using the RedlichKwong EOS.

46

4.28
First, rewrite the Redlich-Kwong equation in cubic form.

v3

RT 2 a
RT
ab

b b 2 v
0
P
P
PT 1 / 2
PT 0.5

or at the critical point


v3

RTc 2 a
RT
ab
v
c b b 2 v
0
0
.
5
0.5

Pc
P
P
T
P
T
c
c c
c c

Now expand v vc 3 0
v 3 3v 2 vc 3vvc2 vc3 0
Setting the coefficients equal we get the following expressions.
3vc

RTc
Pc

3vc2

vc3

(1)

PcTc0.5
ab

RTc
b b2
Pc

(2)
(3)

Pc Tc0.5

Using Equation 2, find an expression for a:

RT
a 3vc2 C b 2 Pc Tc0.5
PC

Substitute the above expression into Equation 3 to get


b 3 3vc b 2 3vc2b vc3 0
The one real root to this equation is

b 21 / 3 1 vc

(4)

Substitute Equation 4 into Equation 1:

47

0.08664 RTc
Pc

(Equation 4.48)

Now, substitute Equation 1 and Equation 4.48 into the expression for a to get
a

0.42748RTc2.5
Pc

(Equation 4.47)

Pv
To verify Equation 4.50, substitute Equation 1 into z c c c
RTc

RT
Pc c
3Pc
zc
RTc

1
3

(Equation 4.50)

The Redlich-Kwong equation is


P

RT
a

v b T 0.5vv b

We can use Equation 4 to get


0.2599 RT
a
b
P

vr 0.2599 T 0.5vvc vr 0.2599

Now, substitute Equation 4.48


P

3Tr Pc
a

0
.
5
v r 0.2599 T vvc v r 0.2599

Replacing a with Equation 4.47 yields


P

3Tr Pc
0.42748R 2Tc2.5

v r 0.2599 PC T 0.5 vvc v r 0.2599

From Equation 4.48


R 2Tc2

Pc2 b 2

0.086642

48

which upon substitution yields


P

3Tr Pc
Pc b 2
0.42748

v r 0.2599 0.08664 2 Tr0.5 vvc v r 0.2599

Now, use Equation 4:


P

3Tr Pc
Pc
0.42748 0.2599 2

v r 0.2599
Tr0.5 v r v r 0.2599
0.086642

Therefore,
Pr

3Tr
1

0
.
5
vr 0.2599 0.2599Tr vr vr 0.2599

49

(Equation 4.49)

4.29
Since we are concerned with liquid water, we can base all our calculations on saturated liquid
water since the molar volume of liquids are weakly dependent on pressure. Therefore, our
results are also applicable to sub-cooled water (the pressure is greater than the saturation
pressure). To find the thermal expansion coefficient, we will use the following approximation

v sat T 5 C v sat T C
1 v sat T 5 C v sat T C

10 K
v sat T T 5 C T 5 C v sat T

This approximation is valid even though the saturation pressure changes because molar volume
is weakly dependent on pressure. From the steam tables

m3
v sat 15 C 0.001001

kg
m3
v sat 20 C 0.001002

kg

m3
v sat 95 C 0.001040

kg
m3
v sat 100 C 0.001044

kg

m3
v sat 25 C 0.001003

kg

m3
v sat 105 C 0.001047

kg

Using these values, we can calculate the thermal expansion coefficients

m3
m3

0
.
001003
0
.
001001

kg
kg
m3

20 C 0.001002

10 K
kg

m3
m3
0.001040

1 0.001047

kg
kg
m3

100 C 0.001044

10 K
kg


100 C 6.71 10 K

20 C 2.0 10 4 K -1
4

-1

The accuracy of these values may be limited since they are based on the small differences
between liquid volumes.
To calculate the isothermal compressibility, a similar approximation will be used. It is

50

dv sat

v sat T dP
1

The derivatives are determined from the following graph.


Specific Volume of Liquid Water vs. Pressure

v = -5E-07*P + 0.001044
R2 = 0.9949

0.001040

Specific Volume [m /kg]

0.001050

0.001030

T = 20C
T = 100 C

0.001020

Linear (T = 100 C)

Linear (T = 20C)

0.001010

v = -5E-07*P + 0.001002
R2 = 0.9979

0.001000
0.000990
0

10

15

20

Pressure [MPa]

It is clear that
dv sat

dP

dv sat

5 10 7

dP
T 20 0 C
T 100 0 C

m3

kg MPa

Therefore,

m3
20 C 0.001002

kg

5 10 7

m3

kg
MPa

1
1
4.99 10 10

Pa
MPa

20 C 4.99 10 4

m3
100 C 0.001044

kg

5 10 7

m3

kg
MPa


1
1
4.79 10 10
100 C 4.79 10 4

MPa
Pa

51

25

4.30
(a)
Substitute critical data into the Rackett equation:

vcalc

8.314190.6 0.29056 0.087750.0081 1111 / 190


5
2/7

46 10
cm 3
vcalc 38

mol
Now, calculate the error.

v
calc vexp
Error

vexp

Error 0.8%

100 %

(b)

cm 3
vcalc 54.2

mol

Error 1.1%

cm 3
vcalc 161.2

mol

Error 10.8%

cm 3
vcalc 20.5

mol

Error 13.5%

cm 3
vcalc 68.5

mol

Error 19.7%

(c)

(d)

(e)

1-hexanol had the largest percent error, while ethane and methane had the smallest percent
errors. The alkenes have low percentile errors due to their nonpolar nature, which the alcohols
and acids had large percentile errors due to being polar substances and exhibiting hydrogen
bonds.

52

4.31
(a)
Since we are given the temperature and pressure, we can make use of the generalized
compressibility charts. Using data from Table A.1.1, the required quantities can be found.

343.15 K
T

1.12
305.4 K
Tc
30 bar
P

0.616
Pr
Pc 48.74 bar
w 0.099

Tr

By double interpolation of the charts


z 0 0.8376
z 1 0.0168

and

z z 0 wz 1 0.8376 0.099 0.0168 0.8393

Therefore,

znRT

30 kg
J
8.314
343.15 K
mol K
0.03007 kg/mol

0.8393

30 10

Pa

V 0.796 m 3

(b)
The Redlich-Kwong EOS should give reasonably accurate results. Room temperature was
assumed to be 25 C. The molar volume is required for the calculations, so

v V

V
0.1 m 3

7.518 10 5
n

40 kg

0
.
03007
kg/mol

m3

mol

Using data from Table A.1.1 and Equations 4.47 and 4.48,
J m 3 K1 / 2
a 9.88

2
mol

53

3
5 m

b 4.51 10

mol
Substituting these values into the Redlich-Kwong EOS and evaluating gives
P 191.3 bar

54

4.32
(a)
For propane

MW propane 0.0441 kg
mol

50 kg
1130 mol
kg
0.0441
mol

Now calculate the volume:

nRT

1130 mol 8.314

J
50 273.15 K
mol K

35 105 Pa

V 0.870 m 3

(b)
The Redlich-Kwong EOS is

RT
a

v b T 1 / 2 vv b

J m 3 K1 / 2
0.42748R 2Tc2.5
18.33

Pc
mol 2

where

m3

mol
(Note: Critical data for propane obtained in Table A.1.1.)
b

0.08664 RTc
6.28 10 5
Pc

Now that these values are known, there is only one unknown in the Redlich-Kwong EOS: v .
Using a numerical technique, e.g., the solver function on a graphing calculator
m3
v 0.000109

mol

V 0.124 m 3

55

(c)
The Peng-Robinson EOS is

RT
a T

v b vv b bv b

where

1 1 Tr
Tr

50 273.15 K 0.873
T

370 K
Tc

0.37464 1.542260.152 0.269920.11522 0.605

1.081
a

J m3
0.45724RTc 2
1.02

Pc
mol

0.07780 RTc
b
5.64 10 5
Pc

m3

mol

Now every variable in the Peng-Robinson EOS is known, except v.


m3
v 0.0000942

mol

V 0.107 m 3

(d)
We must calculate the reduce temperature and pressure to use the compressibility charts:

323.15 K
T

0.873
370 K
Tc
35 bar
P

0.825
Pr
Pc 42.44 bar

Tr

By double-interpolation on the compressibility charts (Appendix C),


z 0 0.1349
z 1 0.052

Therefore,

56

Pv
z z 0 wz 0 0.1349 0.152 0.052 0.127
RT
m3
0.127 RT
0.00009749
v

P
mol
V 0.111 m 3
(e)
From ThermoSolver
Using the Peng-Robinson equation,

m3
v 0.00009429

mol

V 0.107 m 3

Using the generalized compressibility charts:


m3
v 0.0000971

mol
Therefore,

V 0.110 m 3

57

4.33
Note: Multiple possibilities exist for which substance to use in the vial. The solution using one
possibility is illustrated below.
(a)
The substance must have a critical temperature above room temperature but below the
temperature of ones hand. From the Appendix A.1.2, we that for carbon dioxide

Tc 304.2 K 31.1 C
Clearly, CO2 is a suitable substance, and it is safe to use.
(b)
The vial must be able to withstand the pressure of the substance at its critical point. Therefore,
the vial must withstand 73.76 bar (the critical pressure of carbon dioxide).
(c)
Since the substance passes through its critical point, the molar volume at that state is constrained
by the critical temperature and pressure. To estimate the molar volume, we can use the PengRobinson EOS. Calculate the necessary parameters for the EOS:
2

a 0.45724

J
8.314
304.2 K 2

mol K

73.76 10 5 Pa

J m3
0.40

2
mol

J
8.314
304.2 K
mol K

b 0.07780
3.4 10 4
5
73.76 10 Pa
1

m3

mol

Substitute the above parameters and solve for the molar volume:
cm 3
m3
vc 1.18 10 4
118

mol
mol
Now, we can calculate the required amount of CO2.
n

100 cm 3
V

0.847 mol
vc
cm 3
118

mol

58

(d)
If the vial contains less substance than needed, the molar volume will be greater. Therefore, the
substance will not pass through the critical point. As the substance is heated, it will go through a
transition from a saturated liquid and vapor mixture to superheated vapor. Then, it will become a
supercritical fluid once the critical temperature is exceeded.
P

critical
point

liquid

part c

part d
not enough
CO2

vapor

liquid - vapor

59

4.34
Methane:
The following quantities are required to calculate the molar volume with the Peng-Robinson
equation.

T 1 1 Tr
T 0.9626

J m3
0.45724RTc 2
0.25

Pc
mol

0.07780 RTc
2.68 10 5
Pc

T Tr Tc 209.66 K

m3

mol

P Pr Pc 55.2 10 5 Pa
The molar volume is the only unknown in the following equation.
P

RT
a T

v b vv b bv b

v 1.83 10

m3

mol

Therefore,

3
4 m

55.2 10 Pa 1.83 10

mol

z
0.58

J
8.314
209.66 K
mol K

From the compressibility charts,


z 0 0.5984
z 1 0.0897

Therefore,
z z 0 wz 1 0.5984 0.0080.0897
z 0.599

60

Methanol:

Following the procedure outlined above, the Peng-Robinson equation produces


m3
v 3.14 10 4

mol
z 0.651
Using the compressibility charts:
z 0 0.5984
z 1 0.0897

From Table A.1.1,


w 0.559

Therefore,
z z 0 wz 1 0.5984 0.559 0.0897
z 0.649
Summary:

Methane
z 0.58 (Peng-Robinson)
z 0.599 (Compressibility charts)
The value from the Peng-Robinson EOS is 3.2% smaller than the value
from the charts.
Methanol
z 0.651 (Peng-Robinson)
z 0.649 (Compressibility charts)
The value from the Peng-Robinson EOS is 0.31% smaller than the value
from the charts.

61

4.35
From Table A.1.1:

Tc 405.6 K
Pc 112.77 bar
w 0.25
Calculate reduced temperature and pressure:

92 273.15 K 0.9
T

405.6 K
Tc
306.5 bar
P

2.718
Pr
Pc 112.77 bar

Tr

From interpolation of the compressibility charts:


z 0 0.4133

z 1 0.1351

Therefore,
Pv
z z 0 wz 1 0.38
RT
and
v

0.38RT
3.76 10 5
P

m3

mol

Since the temperature of the ammonia in this system is below the critical temperature, we know
the ammonia is not a supercritical fluid. The pressure is greater than the critical pressure, so the
ammonia is a liquid.

62

4.36
Let the subscript ace represent acetylene and but represent n-butane. First, convert the given
quantities of acetylene and n-butane into moles.
nace

nbut

30 kg
26.038 10

-3

50 kg
58.123 10

-3

kg
mol
kg
mol

1152.2 mol
860.2 mol

Therefore,
y ace 0.573
ybut 0.427
We can also calculate a and b parameters for pure species using the following equations
0.42748 R 2Tc2.5
a
Pc
b

0.08664 RTc
Pc

Substituting critical data from Table A.1.1 gives


J K1/2 m3
aace 8.03

2
mol

J K1/2 m 3
abut 29.07

mol

m3
bace 3.62 10 5

mol
m3
bbut 8.08110 5

mol

Now, we can use mixing rules to calculate the parameters for the mixture.
amix y12 a1 2 y1 y 2 a12 y 22 a2
J K1/2 m3
a12 8.03 29.07 1 0.092 13.87

2
mol

J K1/2 m 3
amix 14.72

2
mol

63

bmix y1b1 y 2b2 5.53 10

m3

mol

Substitution of the mixture parameters into the Redlich-Kwong EOS results in an equation with
one unknown.
m3
v 0.00111

mol

m3

V nace nbut v 1152.2 mol 860.2 mol 0.00111

mol

V 2.23 m 3

64

4.37
First, we need to calculate the a and b parameters for species (1) and (2). From Table A.1.1 and
A.1.2:

CO2 (1):

Toluene (2):

Tc 304.2 K

Pc 73.76 bar

a1

J m 3 K1/2
0.42748R 2Tc2.5
6.466

Pc
mol

b1

0.08664 RTc
2.97 10 5
Pc

m3

mol

Tc 591.7 K

Pc 41.14 bar

J m 3 K1/2
0.42748R 2Tc2.5
a2
61.17

Pc
mol

b1

0.08664 RTc
1.036 10 4
Pc

m3

mol

We can use the mixing rules to calculate bmix.


bmix y1b1 y 2b2
2 mol
5
bmix
2.97 10

5
mol

3
5 m
bmix 7.40 10

mol

m 3 3 mol
4


1.036 10

mol
5
mol

m3

mol

Now we can substitute the known values into the Redlich-Kwong EOS. Note that the molar
volume can be calculated as follows
v

m3
0.01 m 3
V

0.002

5 mol
ntot
mol

We can then solve for amix.


J m 3 K1/2
a mix 31.0

mol

65

From the mixing rules, we know


J m3 K1/2
2
2
amix 31.0
y1 a1 2 y1 y2 a12 y2 a2
mol

Therefore,
J m 3 K1/2
a12 16.55

mol

Equation 4.79 states


a12 a1a 2 1 k12

Substitution of the values provides:


k12 0.17

66

4.38
(a)
We can match the species by looking at the b parameter alone. The b parameter is directly
related to the size of the molecule. Because sizeC 2 H 6 size H 2 O size H 2 ,
bC 2 H 6 bH 2 O bH 2 . Therefore,

J m3
a

mol
0.564

m3
b

mol

Species

6.38 10 5

C2H6 (3)

0.025

2.66 10 5

H2 (1)

0.561

3.05 10 5

H2O (2)

These values are also consistent with what we would expect for the magnitude of van der Waals
interactions given by a.
(b)
The van der Waals parameters for the mixture can be calculated according to the mixing rules.
From Equations 4.81 and 4.82
a mix y12 a1 y1 y 2 a12 y1 y3 a13 y 2 y1 a21 y 22 a 2 y 2 y3 a 23 y3 y1 a31 y3 y 2 a32 y32 a3
a mix y12 a1 2 y1 y 2 a12 2 y1 y3 a13 y 22 a2 2 y 2 y3 a 23 y32 a3
bmix y1b1 y 2b2 y3b3

Calculate mole fractions:


n1
5 mol

0.5
n1 n2 n3 10 mol
y 2 0 .4
y3 0.1
y1

Since binary interaction parameters are not available, we must use Equation 4.78 to calculate
a12 , a13 , a23 .
J m3
a12 a1a2 0.118

mol
J m3
a13 0.119

mol

67

J m3
a23 0.562

mol
Now we can find the numerical values for the van der Waals parameters.
J m3
amix 0.206

mol
m3
bmix 3.19 105

mol
(c)
For the mixture, the van der Waals equation is
P

a
RT
mix
v bmix
v2

The molar volume is calculated as follows


v

m3
0.0125 m 3
V

0.00125

10 mol
n1 n2 n3
mol

Therefore,
J m3

J
0
.
206

8.314
300 K
mol

mol
K

3
3
m3
0.00125 m 3.19 10 5 m

0.00125

mol
mol

mol
P 1.92 MPa

68

4.39
(a)
To calculate the molar volume of the mixture, we will use the virial expansion in pressure since
it is more accurate at moderate pressures. But first, we must calculate the second virial
coefficient for the mixture.

Bmix y12 B1 2 y1 y 2 B12 y 22 B2

cm 3
cm 3
cm 3
2

0
.
75
110
2
0
.
25
0
.
75
153
Bmix 0.252 625

mol
mol
mol

cm 3
Bmix 158.3

mol
Therefore,
RT Bmix RT
P
Bmix
1
P
RT P
cm 3
m3
v 2445.6
0.002446

mol
mol

(b)
We can estimate k12 using an EOS where the a parameter is the only unknown. b can be
calculated as follows

bmix y1b1 y 2b2


Using the Redlich-Kwong EOS,

bmix 0.25 5.83 10 5

m3
5

0.75 2.97 10

mol

m3
3
5 m

3.69 10

mol
mol

Substitute the known properties into the Redlich-Kwong EOS and solve for a.
J K1/2 m 3
amix 8.693

2
mol

Now we can calculate k12 as follows


a mix y12 a1 2 y1 y 2 a12 y 22 a2

69

J K1/2 m 3
a1 28.03

2
mol

J K1/2 m 3
a2 6.466

2
mol

J K1/2 m 3
a12 8.81

2
mol

(Calculated using Equation 4.47)

From Equation 4.79


a12 a1a 2 1 k12
8.81
k12 1
0.35
6.466 28.03

70

4.40
During this process the gas in Tank A undergoes an expansion from 7 bar to 3.28 bar. During
this isentropic process, the gas cools. If attractive forces are present, they will manifest
themselves more in state 1 at the higher pressure than in state 2. (While T has some effect to
counter this trend, we expect it will be secondary to the effect of pressure). Thus, in the nonideal case, additional energy must be supplied to pull the molecules apart. Since the tank is
insulated, the only place that this can come from is the kinetic energy of the molecules. Thus,
they slow down even more than in the ideal gas case, and the final temperature is lower. If we
had an equation of state that appropriately described the non-ideal behavior, we could apply the
concepts of the thermodynamic web that we have learned in this chapter to solve for the final
temperature.

71

4.41
Select the Equation of State Solver from the main menu. Enter the pressure and temperature
and solve for the molar volume using the Lee-Kesler EOS. The program provides

m3
v 1.07 10 4

mol
To calculate the volume occupied by 10 kilograms of butane, we need to know the number of
moles present in 10 kg.
n

10 kg
172.1 mol
0.05812 kg/mol

Therefore,

V 172.1 mol1.07 10 4

m3
3

0.018 m

mol

The answers provided in Example 4.9 agree well with this answer. The answer found using the
Redlich-Kwong EOS is 16.7% larger than the answer from ThermoSolver, but the answer from
the compressibility charts is only 5.56% larger.

72

4.42
Using ThermoSolver should be straightforward; thus, only the answers are provided.
(a)

Tc 305.4 K

Pc 48.84 bar

kJ
h f ,298 84.68

mol

(b)

Tsat 299.42 K

(c)
The percent difference is calculated as follows

v LK v PR

z LK z PR
% Difference
or
100 %
v LK
z LK

(i).
Quantity
v
z

Lee Kessler EOS


m3
8.34 10 5

mol
0.1383

Peng Robinson EOS


m3
8.70 10 5

mol
0.1444

% Difference

Lee Kessler EOS


m3
3.68 10 4

mol
0.5859

Peng Robinson EOS


m3
3.58 10 4

mol
0.5711

% Difference

4.3
4.4

(i).
Quantity
v
z

73

2.7
2.5

Chapter 5 Solutions
Engineering and Chemical Thermodynamics

Wyatt Tenhaeff
Milo Koretsky
Department of Chemical Engineering
Oregon State University
koretsm@engr.orst.edu

5.1
(a)
Following the example given by Equation 5.5a in the text
u
u
du
dT dP
P T
T P

(b)
u
u
du
dT ds
T s
s T
(c)
u
u
du dh ds
s h
h s

5.2.
The internal energy can be written as follows
u
u
du
dT dv
T v
v T
Substituting Equations 5.38 and 5.40

u
u
P

cv and T
v T T v
T v

into the above expression yields

du cv dT T
P dv
T v

From the ideal gas law, we have

R
P


T v v
Therefore,

RT

du cv dT
P dv
v

which upon noting that P

RT
for an ideal gas, becomes
v

du cv dT

5.3
The heat capacity at constant pressure can be defined mathematically as follows

h
u Pv
u
v
cP


P

T
T P
P T v
T P
For an ideal gas:

R
v


T P P
Therefore,

u
cP
R
T v
One mathematical definition of du is

u
u
du
dT dP
T P
P T
u
We can now rewrite
:
T v
u
u T u P




cv
T v T P T v P T T v
For an ideal gas:

u
0
P T
so

u
cv

T P
Substituting this result into our expression for c P gives

c P cv R

5.4
In terms of P, v, and T, the cyclic equation is

P T v
1


T v v P P T
For the ideal gas law:
Pv RT

so the derivatives become:

R
P


T v v
P
T


v P R
RT v
v
2
P
P T
P
Therefore,

P T v
R P v



1
T v v P P T v R P
The ideal gas law follows the cyclic rule.

5.5
For a pure species two independent, intensive properties constrains the state of the system. If we
specify these variables, all other properties are fixed. Thus, if we hold T and P constant h cannot
change, i.e.,

h
0

v T , P

5.6
Expansion of the enthalpy term in the numerator results in
Ts vP
h

T
T s
s
P
h

v
T s
T s

Using a Maxwell relation


s
h
v

v P
T s
s T
h


v
T P v P
T s
We can show that
c
s
P

T P T

(use thermodynamic web)

1 RT
2a 2ab
T


v P R v b v 2 v 3

(differentiate van der Waals EOS)

Therefore,
v
vc RT 2a 2ab
2a b
h
c P

1
P

2
3
T s RT v b v
v
v b vRT v

5.7
h
:
P T

s
Ts vP
h
T v

P
P T
T
P T

R
v
s
2
1 B' P C ' P

P
T P
P T

RT
h
1 B ' P C ' P 2 v v v

P
P

T
h
0
P T

h
:
P s
h Ts vP
v

P
s
P s
RT
h
1 B' P C ' P 2

P
P s

h
:

T P
h
cP

T P

(Definition of cP)

h
:

T s
P
Ts vP
h

T
T s
s
T s
c T
c
P
s P
P
P
P

T v P T R 1 B' P C ' P 2
T P s T
T s

Pv
1
1 B' P C ' P 2
h

P
P
RT 1 B' P C ' P 2
T s
1 B' P C ' P 2
h
cP

T s

5.8
(a)
A sketch of the process is provided below

well
insulated

s = 0

T1
P1

T2
P2

The diagram shows an infinitesimal amount of mass being placed on top of the piston of a
piston-cylinder assembly. The increase in mass causes the gas in the piston to be compressed.
Because the mass increases infinitesimally and the piston is well insulated, the compression is
reversible and adiabatic. For a reversible, adiabatic process the change in entropy is zero.
Therefore, the compression changes the internal energy of the gas at constant entropy as the
pressure increases.
(b)
To determine the sign of the relation, consider an energy balance on the piston. Neglecting
potential and kinetic energy changes, we obtain

U Q W
Since the process is adiabatic, the energy balance reduces to
U W

As the pressure increases on the piston, the piston compresses. Positive work is done on the
system; hence, the change in internal energy is positive. We have justified the statement

u
0
P s

10

5.9
(a)
By definition:
1 v

v T P


and

1 v

v P T

Dividing, we get:
v

v P
T P

v
T P v T

P T

where derivative inversion was used. Applying the cyclic rule:


v P T
1
T P v T P v

Hence,

P

T v
(b)
If we write T = T(v,P), we get:

T
T
dT dv dP
P v
v P

(1)

From Equations 5.33 and 5.36


ds

cv
c
P
v
dT dv P dT dP
T
T
T v
T P

We can solve for dT to get:

11

dT

T P
T v
dP
dv
c P cv T v
c P cv T P

(2)

For Equations 1 and 2 to be equal, each term on the left hand side must be equal. Hence,

T
T

v P c P cv

P

T v

or

v
P v
c P cv T T
T P
T v T P
where the result from part a was used. Applying the definition of the thermal expansion
coefficient:

Tv
P v
c P cv T

T v T P

12

5.10
We need data for acetone, benzene, and copper. A table of values for the molar volume, thermal
expansion coefficient and isothermal compressibility are taken from Table 4.4:

m3
v 10 6

mol
73.33
86.89
7.11

Species
Acetone
Benzene
Copper

103 K -1

1010 Pa -1

1.49
1.24
0.0486

12.7
9.4
0.091

We can calculate the difference in heat capacity use the result from Problem 5.9b:
c P cv

vT 2

or

73.33 10 6

c p cv

Species

Acetone
Benzene
Copper

m3
3
-1 2

293 K 1.49 10 K

mol
12.7 10

10

Pa
-1

J
c p cv
mol K
37.6
41.6
0.5

J
cp
mol K
125.6
135.6
22.6

J
37.6
mol K

% difference
30%
31%
2%

We can compare values to that of the heat capacity given in Appendix A2.2. While we often
assume that cP and cv are equal for condensed phases, this may not be the case.

13

5.11
We know from Equations 4.71 and 4.72

1 v

v T P

and

1 v

v P T

Maxwell relation:
P
s


v T T v

Employing the cyclic rule gives


P v
P

v T T P
T v

which can be rewritten as


1 v

v T P
P
s


v T T v 1 P

v v T
Therefore,

s

v T
Maxwell Relation:
s
v

P T
T P
From Equation 4.71:
v

v
T P
Therefore,
s
v
P T

14

5.12
(a)
An isochor on a Mollier diagram can be represented mathematically as

h

s v
This can be rewritten:
h
Ts vP
P

T
s
s v
v
s v
Employing the appropriate Maxwell relation and cyclic rule results in
h
T s
T v

s v v T
s v
We know
T
T
s
P
and

s v c v
v T T v
For an ideal gas:
R
s
P


v T T v v
Therefore,

T R
R
h
T 1
T v
cv v
cv
s v

(b)
In Part (a), we found

T
h
T v
cv
s v

T v

For a van der Waals gas:

15

R
P


T v v b
Therefore,
RT
h
T
cv
s v

v b

16

5.13
(a)
The cyclic rule can be employed to give

T
T s



s P P T
P s
Substitution of Equations 5.19 and 5.31 yields
T
T


P s c P

T P

For an ideal gas:


R
v


T P P
Therefore,
RT 1
v
T

P cP cP
P s
(b)
Separation of variables provides

T
R P

T
cP P
Integration provides
T
ln 2
T1

P
ln 2

P1

cP

which can be rewritten as


R
P2 cP

T2

T1 P1

The ideal gas law is now employed

17

P2 v 2 P2 cP

P1v1 P1
R
1
c
P2 P

R
1
c
P1 P

v
1

where
1

R c P R cv 1

cP
cP
cP k

If we raise both sides of the equation by a power of k, we find


P2 v 2k P1v1k

Pv k const.
(c)
In Part (a), we found

T v
T

P s c P T P
Using the derivative inversion rule, we find for the van der Waals equation
Rv 3 v b
v


T P RTv 3 2av b 2

Therefore,
1
RTv 3 v b
T


P s c P RTv 3 2av b 2

18

5.14
The development of Equation 5.48 is analogous to the development of Equation E5.3D. We
want to know how the heat capacity changes with pressure, so consider

c P

which can be rewritten as


c P

h
h



T P T P T T P T P

h
Consider the term:
P T
h
Ts vP
s
v
T v T
v

P
P T
T P
P T
T
c
Substitution of this expression back into the equation for P results in
P T
v

c P
T

v
P T T T P
P
2v
T v
c P

T T P
P T
T P T P
2v
c P

T 2
T
P T

Therefore,
c Preal

Preal

c Pideal

Pideal

dc P

2v
dP
T
2

T
P

and
ideal

c Preal c P

Preal
2v
T
2
Pideal T


dP

P

19

5.15
In order to solve this problem we need to relate the change in entropy from 10 to 12 bar to the
change in molar volume (for which we have complete data). First, we can rewrite the change in
entropy as
12 bar

s s 2 s1

s
dP
P T
10 bar

Applying a Maxwell relation, we can relate the above equation to the change in molar volume:
12 bar

s 2 s1

12 bar

s
v
P T dP s1 T P dP
10 bar
10 bar

As 10 bar:
v
v
4


5.60 10
T P T P

m3

kg K

At 12 bar:
v
v
4


4.80 10
T P T P

m3

kg K

v
To integrate the above entropy equation, we need an expression that relates
to pressure.
T P
Thus, we will fit a line to the data. We obtain

v
10

4.0 10
T P

m3
3
4 m

P 9.6 10

kg K Pa
kg K

Now integrate the equation to find the entropy:

4.0 10

1.210 6 Pa

s 2 s1

1.010 6 Pa

10

P 9.6 10 4 dP 5.4960 0.104 kgkJ K 5.392 kgkJ K

20

5.16
A schematic of the process follows:

Propane in
v1 = 600 cm3/mol
T1= 350 oC

Turbine

ws
Propane P = 1 atm
2
out

We also know the ideal gas heat capacity from Table A.2.1:

cP
1.213 28.785 10 3 T 8.824 10 6 T 2
R
Since this process is isentropic (s=0), we can construct a path such that the sum of s is zero.
(a) T, v as independent variables
Choosing T and v as the independent variables, (and changing T under ideal gas conditions), we
get:

s=0

s1

step 1

s2

volume

v2 ,T2

Ideal
step 2 Gas

v1 ,T 1

Temperature

or in mathematical terms:
s
s
ds dT dv 0
T v
v T
However, From Equation 5.33:

21

c
P
ds v dT
dv
T v
T
To get s1
P

a
RT
2
vb v

so

R
P

T v v b

and
v
dv 2 R dv R ln v2 b


v1 b
v1 T v
v1 v b

v 2 P

s1 ds

or, using the ideal gas law, we can put s1 in terms of T2:
RT2 b
P

s1 R ln 2
v1 b

For step 2
T2

T2

T1

623.15 K

c
s 2 v dT R
T

0.213 28.785 10 3 T 8.824 10 6 T 2


dT
T

Now add both steps


s s1 s 2 0
RT2

P b
T2
8.824 10 6 2
3
0.213 ln

ln 2

28
.
785

10
T

623
.
15
K

T2 623.15 K 2

2
2
v1 b
623.15

Substitute
T1 623.15 K

v1 600 cm 3 /mol
P2 1 atm

cm 3 atm
R 82.06

mol K

22

and solve for T2:


T2 448.3 K

(b) T, P as independent variables

Choosing T and P as the independent variables, (and changing T under ideal gas conditions), we
get:

P1,T1

step 1

Pressure

s=0

step 2

s1

Ideal
Gas

s2

P2,T2

Temperature

Mathematically, the entropy is defined as follows


s
s
ds
dT dP 0
P T
T P

Using the appropriate relationships, the expression can be rewritten as


c
v
ds P dT
dP 0
T
T P

For the van der Waals equation


R
v b


T P
RT
2a

2
v 3
v b
Therefore,

23

T2

T1

cP
dT
T

P2

R
v b

2a
P1

v b 2 v 3

dP 0

RT

We cant integrate the second term of the expression as it is, so we need to rewrite dP in terms
of the other variables. For the van der Waals equation at constant temperature:
2a
RT
dP
dv
3
v b 2
v
Substituting this into the entropy expression, we get
1.213 28.785 10 3 T 8.824 10 6 T 2
s
dT
T
T2

v2

T1

v1

Upon substituting
T1 623.15 K
v1 600 cm 3
v2

RT2
(gas acts ideally at 1 atm)
P2

cm 3
b 91

mol
cm 3 atm
R 82.06

mol K

we obtain one equation for one unknown. Solving, we get


T2 448.3 K

24

v b dv 0

5.17
(a)
Attractive forces dominate. If we examine the expression for z, we see that at any absolute
temperature and pressure, z 1. The intermolecular attractions cause the molar volume to
deviate negatively from ideality and are stronger than the repulsive interactions.
(b)
Energy balance:
h2 h1 q

Alternative 1: path through ideal gas state


Because the gas is not ideal under these conditions, we have to create a hypothetical path that
connects the initial and final states through three steps. One hypothetical path is shown below:
P [bar]
P,T1

P,T2

q = h

50

step 3

step 1

h1

h
3

h2 step 2

ideal gas
300

500

Choosing T and P as the independent properties:


h
h
dh
dT dP
T P
P T
or using Equation 5.46
v

dh c P dT T
v dP
T P
The given EOS can be rewritten as
1

v R aT 1 / 2
P

25

T [K]

Taking the derivative gives:


R
v
0.5

0.5aRT
T P P

so

dh cP dT 0.5aRT 0.5 dP
For step 1

0.5aRT1

0.5

h1

50 bar

dP 0.5aRT10.5 P 252 molJ

For step 2

3.58 3.02 10

500 K

h2 R

300 K

J
T 0.875T 0.5 dT 7961

mol

For step 3:

0.5aRT2

50 bar

h3

0.5

dP 0.5aRT20.5P 323 molJ

Finally summing up the three terms, we get,


J
q h1 h2 h3 7888
mol
Alternative 2: real heat capacity
For a real gas
h c Preal

From Equation 5.48:


2v
T
T 2
ideak

P
P real

c Preal c Pideal

dP

For the given EOS

26


1
v R aT 1 / 2
P

Therefore,
2v

0.25aRT 1.5
T 2

and
2v
T
T 2
ideak

P
P real

P 50 bar

dP
0.25aRT 0.5 dP 0.875 K 1/2 RT 0.5

P
P ideak 0 bar
real

We can combine this result with the expression for c Preal and find the enthalpy change.

3.58 3.02 10

500 K

h R

T 0.875T 0.5 dT

300 K

J
q h 7888
mol
The answers is equivalent to that calculated in alternative 1

27

5.18
(a)
Calculate the temperature of the gas using the van der Waals equation. The van der Waals
equation is given by:

RT
a
2
v b v

First, we need to find the molar volume and pressure of state 1.

m3
V
Al 0.1 m 2 0.4 m

0.00016
v1 1

n
n
250 mol
mol
P1

mg
Patm
A

10000 kg 9.81 m2

0.1 m 2

1.01325 10 5 Pa 1.08 10 6 Pa

Substituting these equations into the van der Waals equation above gives
J m3

J
0
.5

8.314
T1
mol
mol K

6
1.08 10 Pa

2
3

m3
3

5 m
m
0.00016
0.00016
4 10

mol
mol
mol

T1 297.5 K
Since the process is isothermal, the following path can be used to calculate internal energy:
v2 ,T2 = T 1

u
s

v1 ,T1

Thus, we can write the change in internal energy as:

28

u
u
u
du
dT dv dv
v T
v T
T v
Using Equation 5.40
v2

T T v P dv

v1

For the van der Waals EOS:


P

RT
a
2
v b v

so
R
P


T v v b
Therefore,
v2

v 2 dv

v1

We can assume the gas in state 2 is an ideal gas since the final pressure is atmospheric.
Therefore, we calculate v 2 ,
v2

m3
RT2
0.0244

P2
mol

and
J m3
0
.
5

0.0244
mol
J
dv 3104.5
u
mol
v2
0.00016
or
U 776.1 kJ

29

(b)
From the definition of entropy:
suniv s sys s surr

First, lets solve for s sys using the thermodynamic web.


s
s
ds sys
dT dv
T v
v T
Since the process is isothermal,
s
ds sys dv
v T
v2

P
s sys
dv
T v

v
1

Again, for the van der Waals equation,


R
P


T v v b
Substitution of this expression into the equation for entropy yields
v2

s sys

v b dv

v1

J
8.314
mol K dv 44.17 J
s sys
mol K
3
0.00016 v 4 10 5 m

mol
J
S sys 11042.5
K
0.0244

The change in entropy of the surroundings will be calculated as follows


s surr

Qsurr
Tsurr

where
30

Qsurr Q

(Q is the heat transfer for the system)

Application of the first law provides


Q U W

We know the change in internal energy from part a, so lets calculate W using
v2

W n Pdv
v1

Since the external pressure is constant,


m
250 mol1.01325 10 5 Pa 0.0244

W 614030 J

m3
0.00016

mol
mol

Now calculate heat transfer.


Q 776100 J 614030 J 1.39 10 6 J

Therefore,

S surr

1.39 10 6 J
J
4672
297.5 K
K

and the entropy change of the universe is:


J
J
J
S univ 11042.5 4672 6370.5
K
K
K

31

5.19
First, calculate the initial and final pressure of the system.

Pi 10 10

P f 10 10

20000 kg 9.81 m/s 2


Pa
4.92 10 6 Pa
2


Pa 30000 kg9.81 m/s 6.89 10
0.05 m
0.05 m

Pa

To find the final temperature, we can perform an energy balance. Since the system is wellinsulated, all of the work done by adding the third block is converted into internal energy. The
energy balance is
u w

To find the work, we need the initial and final molar volumes, which we can obtain from the
given EOS:

vi 8.37 10 4 m 3 /mol
vf

8.314T f

25
6.89 10 6 1
Tf

3.2 10 -5 m 3 /mol

Now, calculate the work

8.314T f
w Pf v f vi 6.89 10 6 Pa
3.2 10 -5 8.37 10 4
25

6
.
89
10
1

Tf

We also need to find an expression for the change in internal energy with only one variable: Tf.
To find the change in internal energy, we can create a hypothetical path shown below:

step 2
v

ideal gas

step 3

step 1

vi ,Ti
u = -P (v -v )
f f i
vf ,Tf

500

Tf

32

For step 1, we calculate the change in internal energy as follows


v RT / Plow

u1

vi

u1

u
dv
v T

v RT / Plow

vi

P dv
T
T v

v RT / Plow

2
2
aRTi dv aRTi ln RTi / Plow b

T a 2 v b T a 2
vi b

i
i

vi

Similarly, for step 3:


vf

f
P

u
dv
T
P
u3

v T
T v dv
v RT / Plow
v RT / Plow

aRT 2 dv

aRT f 2
vf b
f

ln
T a 2 v b T a 2 RT f / Plow b

f
v RT / Plow f

vf

u3

Insert the expression for the final molar volume into the equation for u3 :

8.314T f
u3
ln
T f a 2 6.89 10 6 1 25 / T f RT f / Plow b

aRT f 2

Since the pressure is low (molar volume is big) during the second step, we can use the ideal heat
capacity to calculate the change in internal energy.
u 2

Tf

Tf

Ti 500 K

Ti 500 K

cv dT

20 R 0.05T dT

u 2 11.686 T f 500 0.025 T f2 500 2

If we set the sum of the three steps in the internal energy calculation equal to the work and
choose an arbitrary value for Plow, 100 Pa for example, we obtain one equation with one
unknown:

33

RT / P b
11.686 T f 500 0.025 T f2 500 2
ln i low
2
Ti a vi b

aRTi 2

8.314T f

ln

2
6
Tf a
6.89 10 1 25 / T f RT f / Plow b

8.314T f
6.89 10 6 Pa
3.2 10 -5 8.37 10 4

25
6

6.89 10 1

Tf

aRT f 2

Solving for Tf we get


T f 536.2 K

The piston-cylinder assembly is well-insulated, so


suniv s sys

Since the gas in the cylinder is not ideal, we must construct a hypothetical path, such as one
shown below, to calculate the change in entropy during this process.

P
Pf ,Tf

6.9

step 1

step 3

Pi ,Ti

4.9

ideal gas
Plow

step 2
536

500

For steps 1 and 3


Plow

Plow

s
s1 dP
P T
Pi
Pi

v
dP

T P

34

Pf

f
v
s
s3 dP
dP
P T
T

P
Plow
Plow

We can differentiate the given EOS as required:


Plow
RTi 2a Ti
RTi 2a Ti Plow
dP
ln
s1

2
2

a
T
P
a
T

Pi
i
i

Pi

s3

RT 2a T
f
f
a T 2 P
f
Plow
Pf

dP RT f 2a T f ln Pf

a T f 2 Plow

For step 2
Tf

Tf

Tf

c
s
20

s 2
dT P dT 0.05 dT
T
T P
T

Ti
Ti
Ti
Tf
0.05 T f Ti
s2 20 ln

T
i

Sum all of the steps to obtain the change in entropy for the entire process
suniv s sys s1 s 2 s3

RT f 2a T f
Tf
Pf
RTi 2a Ti Plow
0.05 T f Ti

20 ln
ln
ln
T
P
2
a Ti 2

a
T
i
low
Pi

f
Arbitrarily choose Plow (try 100 Pa), substitute numerical values, and evaluate:

suniv

J
suniv 0.388
mol K

J
J
S univ 2 mol 0.388
0.766

mol K
K

35

5.20
A schematic of the process is given by:
well
insulated

V=1L

V=2L

V=1L

T = 500 K

T=?

Vacuum

nCO =1 mole

nCO=1 mole

State i

State f

(a)
The following equation was developed in Chapter 5:

2P
T T 2 dv
v
v ideal
v

cvreal cvideal

For the van der Waals EOS


2P

0
T 2

Therefore,
cvreal cvideal
From Appendix A.2:

3100
J
R 22.0
cvreal R 3.376 5.57 10 4 500
mol K
500 2

(b)
As the diaphragm ruptures, the total internal energy of the system remains constant. Because the
volume available to the molecules increases, the average distance between molecules also
increases. Due to the increase in intermolecular distances, the potential energies increase. Since
the total internal energy does not change, the kinetic energy must compensate by decreasing.
Therefore, the temperature, which is a manifestation of molecular kinetic energy, decreases.

36

(c)
Because the heat capacity is ideal under these circumstances we can create a two-step
hypothetical path to connect the initial and final states. One hypothetical path is shown below:
v

vf ,Tf
step 2

u
2
s2

step 1

vi ,Ti

u1, s1

Tf

500

For the first section of the path, we have


Tf

Tf

u1

cvreal dT

cvideal dT
Ti

Ti

Tf

3100
4
2.376 5.57 10 T 2 dT
T
500 K i
25773.4
u1 2.32 10 3 T f2 19.75T f
10507.4
Tf

u1 R

For the second step, we can use the following equation


vf

u 2

v T dv

vi

If we apply Equation 5.40, we can rewrite the above equation as


vf

u 2

T
P

T v dv
vi

For the van der Waals EOS, P

RT
a
2 ,
v b v

37

R
P


T v v b
Therefore,
v f 0.002

v f 0.002

a
RT

J
u1
v b P dv v 2 dv 73.7 mol
v i 0.001
v i 0.001
Now set the sum of the two internal energies equal to zero and solve for Tf:

u1 u 2 2.32 10 3 T f2 19.75T f

25773.4
10507.4 73.7 0
Tf

T f 497 K

(d)
Since the system is well-insulated
suniv s sys

To solve for the change in entropy use the following development:


s
s
ds sys
dT dv
v T
T v
Using the thermodynamic web, the following relationships can be proven
cv
s

T v T
P
s


v T T v
For the van der Waals EOS
R
P


T v v b
Now we can combine everything and calculate the change in entropy

38

497 K

s sys

500 K

cv
dT
T

0.002

v b dv
0.001

0.002
497 K 2.376
dv
3100
dT

s sys R
5.57 10 4

5
500 K T
T3
0.001 v 3.95 10
J
suniv s sys 5.80

mol K

39

5.21
A schematic of the process is given by:
well
insulated

V = 0.1 m3

V = 0.1 m3

T = 300 K

V = 0.2 m3
T=?

Vacuum

nA =400 moles

nA=400 moles

State f

State i

Energy balance:
u 0

Because the gas is not ideal under these conditions, we have to create a hypothetical path that
connects the initial and final states through three steps. One hypothetical path is shown below:
step 2
v [m 3/mol]

ideal gas
step 1

step 3

0.2/400

vf ,Tf
u

0.1/400

vi ,Ti

Tf

300

For the first section of the path, we have


v

u1

u
dv
v T
vi

If we apply Equation 5.40, we can rewrite the above equation as


v

u1

P dv
T
T v

vi

40

T [K]

For the van der Waals EOS

R
a
P


2
T v v b T v 2
Therefore,
v

u1

RT

a
2a
J
P

4 v b Tv 2 dv 4 T v 2 dv 1120 mol
i
vi 2.510
vi 2.510

Similarly for step 3:


v f 510 4

u3

RT

a
v b 2 P dv
Tv

v f 510 4

2a

Tf v

dv

168000 J
mol K
Tf

For step 2, the molar volume is infinite, so we can use the ideal heat capacity given in the
problem statement to calculate the change in internal energy:

u 2

3
R T f 300 K
2

If we set sum of the changes in internal energy for each step, we obtain one equation for one
unknown:

168000
J 3
J
u1 u 2 u3 1120
8.314
T f 300 K
0

Tf
mol K
mol 2

Solve for Tf:


T f 261.6 K

41

5.22
A schematic of the process is shown below:
Ethane 3 MPa; 500K

T surr = 293 K
Initially:
vacuum

(a)
Consider the tank as the system. Since kinetic and potential energy effects are negligible, the
open system, unsteady-state energy balance (Equation 2.47) is
dU

nin hin n out hout Q W s


dt

sys in
out

The process is adiabatic and no shaft work is done. Furthermore, there is one inlet stream and no
outlet stream. The energy balance reduces to
dU
nin hin

dt sys

Integration must now be performed


U2

U1

dU n

h dt

in in

n2 u 2 n1u1 hin n in dt nin hin (n2 n1 )hin


0

Since the tank is initially a vacuum, n1=0, and the relation reduces to:
u 2 hin

42

As is typical for problems involving the thermodynamic web, this problem can be solved in
several possible ways. To illustrate we present two alternatives below:
Alternative 1: path through ideal gas state
Substituting the definition of enthalpy:

u 2 uin Pin vin

or
u2 at 3 MPa, T uin at 3 MPa, 500 K Pin vin

(1)

From the equation of state:

J
Pin vin RT 1 B' P 8.314
552 K 1 2.8 10 8 3 10 6 Pa 3,800

mol
K

J
mol (2)

The change in internal energy can be found from the following path:
P

step 1

Plow

u1

u3

step 3

3 MPa

u2
step 2 ideal gas
T2

500 K

For steps 1 and 3, we need to determine how the internal energy changes with pressure at
constant temperature: From the fundamental property relation and the appropriate Maxwell
relation:

u
s
v
v
v
T P T
P
P T
P T
P T
T P
P T
From the equation of state

RT
u
1 B'P P RT2 B' RT

P
P T
P

43

So for step 1:
u
u1 dP B ' RTdP B ' RT Pin 349 [J/mol]
P T
Pin
Pin
0

(3)

and for step 3:


u
u 3 dP B ' RTdP B ' RT P2 0.7T
P T
0
0
P2

P2

(4)

For step 2
T
T
h Pv
u
u 2
dT c P R dT

dT
T P T P
T P
500
500
500
T

or
u 2 R

0.131 19.225 10

T2

T 5.56110 6 T 2 dT

T1 500 K

Substituting Equations 2, 3, 4, and 5 into 1 and solving for T gives:


T2 552 K

Alternative 2: real heat capacity


Starting with:

u 2 hin

The above equation is equivalent to


h2 P2 v2 hin
h2 hin P2 v2

To calculate the enthalpy difference, we can use the real heat capacity
ideal
c Preal c P

P 2
v
T
2

Pideal T


dP

P

For the truncated viral equation,

44

(5)

2v

0
T 2

Therefore,
ideal
c Preal c P

Now, we can calculate the change in enthalpy and equate it to the flow work term.
T2

cP

ideal

dT P2 v2

T1 500 K

1.131 19.225 10

T2

T 5.561 10 6 T 2 dT P2 v2 RT2 1 B' P2

T1 500 K

Integrate and solve for T2:


T2 552 K

(b)
In order to solve the problem, we will need to find the final pressure. To do so, first we need to
calculate the molar volume. Using the information from Part (a) and the truncated virial
equation to do this

RT
1 B' P
P

J
552 K
8.314
mol K

3 10 Pa

1 2.8 10 3 10 Pa
8

m3
v 0.0014

mol

This quantity will not change as the tank cools, so now we can calculate the final pressure.

m3
P2 0.0014

mol

1 2.8 10 8 P2

J
293 K
8.314
mol K

Solve for P2 :
45

P2 1.66 10 6 Pa

The entropy change of the universe can be expressed as follows:


S univ S sys S surr

To solve for the change in entropy of the system start with the following relationship:
s
s
ds sys
dT dP
T P
P T

Alternative 1: path through ideal gas state


Using the proper relationships, the above equation can be rewritten as
ds sys

cP
v
dT
dP
T
T P

We can then use the following solution path:


P

s3

1.66 MPa
step 1

s 1
s 2

Plow

step 2 ideal gas


500 K

Choosing a value of 1 Pa for Plow, for step 1:


R
v
'
s1
dP 1 B P dP
T P
P
1.66 MPa
1.66 MPa
1 Pa

1 Pa

For step 3,
3 MPa

s1

1 Pa

dP
T P

3 MPa

1 Pa

step 3

3 MPa

R
1 B ' P dP
P

46

T2

For step 2:
293 K
cP
1.131

dT
R

19.225 10 3 5.561 10 6 T dT

T
T

552 K
552 K
293 K

s 2

Adding together steps 1, 2 and 3:


J
s sys 46.9
mol K
______________________________________________________________________________
Alternative 2: real heat capacity
Using the proper relationships, the above equation can be rewritten as
c real
v
ds sys P dT
dP
T
T P

For the truncated virial equation


v
1

R B '
T P
P

Now, substitute the proper values into the expression for entropy and integrate:

293 K

s sys R

552 K

1.6610 6 Pa

1.131
3
6
T 19.225 10 5.561 10 T dT R

B' dP
P

310 ^ Pa

J
s sys 46.9
mol K
______________________________________________________________________________

In order to calculate the change in entropy of the surroundings, first perform an energy balance.
u q

Rewrite the above equation as follows


h Pv q

47

Since the real heat capacity is equal to ideal heat capacity and the molar volume does not change,
we obtain the following equation
Tf

cP

ideal

dT v P f Pi q

Ti
T f 293K

1.131 19.225 10

Ti 552 K

J
q 15845
mol

Therefore,
J
q surr 15845
mol

and
J
s surr 54.08
mol K

Before combining the two entropies to obtain the entropy change of the universe, find the
number of moles in the tank.
n

m3
6
6
T 5.561 10 6 T 2 dT 0.0014
1.66 10 Pa - 3 10 Pa q

mol

0.05 m 3
75.7 mol
m3
0.0014

mol

Now, calculate the entropy change of the universe.

J
J
S univ 75.7 mol 54.08
46.9

mol K
mol K

J
S univ 544
K

48

5.23
First, focus on the numerator of the second term of the expression given in the problem
statement. We can rewrite the numerator as follows:

uTr , v r uTideal
uTr , v r uTideal
uTideal
uTideal
r ,vr
r ,vr
r ,vr
r ,vr

For an ideal gas, we know


uTideal
uTideal
0
r ,vr
r ,vr

Therefore,
uTr , v r uTideal
uTr , v r uTideal
,vr
,vr
r
r

Substitute this relationship into the expression given in the problem statement:
uTdep
,v
r

RTc

uTr , v r uTideal
uTr , v r uTideal
,vr
,vr
r
r

RTc
RTc

Now, we need to find an expression for uTr , vr uTideal


. Note that the temperature is constant.
r ,vr
Equation 5.41 reduces to the following at constant temperature:
P

duT T
P dv
T v

The pressure can be written as


P

zRT
v

and substituted into the expression for the differential internal energy
RT z
RT 2 z
RT
zRT

duT T
dv

dv

v
v
v
T

v
v T v

Applying the Principle of Corresponding States

49

T 2 z
r
RTc vr Tr

duTr


dvr
v r

If we integrate the above expression, we obtain


vr 2
uTr , v r uTideal
T z
r, vv

r
RTc
RTc
v r Tr v
v
v
r
v

duTr

dv r

Therefore,
uTdep
,v
r

RTc

vr 2
uTr , v r uTideal
uTr , v r uTideal
T z
v
,
,vr
r
r
r

r
RTc
RTc
v r Tr v
v
r

50

dv r

5.24
We write enthalpy in terms of the independent variables T and v:

h
h
dh
dT dv
T v
v T

using the fundamental property relation:


dh Tds vdP

At constant temperature, we get:


P
P
dhT T
v dv
v T
T v

For the Redlich-Kwong EOS


1
R
a
P


3
/
2
T v v b 2 T
vv b
RT
a
a
P


v T v b 2 T 1 / 2 v 2 v b T 1 / 2 vv b 2

Therefore,
RT

3
RTv
a
a

dhT
dv
v b v b 2 2 T 1 / 2 vv b T 1 / 2 v b 2

To find the enthalpy departure function, we can integrate as follows


v

dep

dhT

RT

RTv
a
a
3

v b v b 2 2 T 1 / 2 vv b T 1 / 2 v b 2 dv

v
v

Since temperature is constant, we obtain


h dep

3a
RTb
a
v

ln

v b 2bT 1 / 2 v b T 1 / 2 v b

To calculate the entropy departure we need to be careful. From Equation 5.64, we have:

gas
gas
ideal gas
gas
sT , P sTideal
sT , P sTideal
sTideal
,P
, P 0 sT , P
, P 0

51

However, since we have a P explicit equation of state, we want to put this equation in terms of v.
Lets look at converting each state. The first two states are straight -forward
sT , P sT , v

and
gas
ideal gas
sTideal
, P 0 sT , v

For the third state, however, we must realize that the ideal gas volume v at the T and P of the
system is different from the volume of the system, v. In order to see this we can compare the
equation of state for an ideal gas at T and P
P

RT
v'

to a real gas at T and P


P

RT
a

vb
T vv b

The volume calculated by the ideal gas equation, v, is clearly different from the volume, v,
calculated by the Redlich-Kwong equation. Hence:
gas
gas ideal gas
gas
sTideal
s ideal' gas sTideal
s '
sTideal

,P
,v
,
v
,
T ,v
T
v

Thus,

gas
gas
ideal gas
gas ideal gas
gas
sT , v sTideal
sTideal
sTideal
sT , P sTideal

,P
, v sT , v
, v sT , v '
,v

Using a Maxwell relation:


ds
P

dv T T v

Therefore,
P
dsT
dv
T v

52

For the Redlich-Kwong EOS


1
R
a
P


3
/
2
T v v b 2 T
vv b
so

ideal gas
T , v sT , v

v
R

a
1

dv
3/ 2

2
v
b

T
v
v
b

For an ideal gas


R
P


T v v

so

gas
gas
sTideal
sTideal
,v
,v

R
v dv
v

Finally:
v'
dv
v'
RT
ideal gas
ideal gas
s '
sT , v
R
R ln R ln
T ,v
v
v
Pv
v

Integrating and adding together the three terms gives:


s dep R ln

v b
v

RT
v
ln
R ln

Pv
2bT 3 / 2 v b
a

53

5.25
Calculate the reduced temperature and pressure:

Tc 647.3 K

Pc 220.48 bar

(Table A.1.2)

w 0.344
300 bar
1.36
220.48 bar
673.15 K
Tr
1.04
647.3 K
Pr

By double interpolation of data from Tables C.3 and C.4

h dep
Tr , Pr
RTc

( 0)

2.921

h dep
Tr , Pr
RTc

(1)

s dep
Tr , Pr
R

(1)

h dep
Tr , Pr
w
RTc

1.459

From Tables C.5 and C.6:


s dep
Tr , Pr
R

( 0)

2.292

1.405

Now we can calculate the departure functions


dep
hTr , Pr
h dep RTc

RTc

( 0)

(1)

J
J
h dep 8.314
647.3 2.921 0.344 1.459 18421

mol K
mol

dep
sTr , Pr
s dep R

( 0)

s dep
Tr , Pr
w
R

(1)

J
J
s dep 8.314
2.292 0.344 1.405 23.07

mol K
mol K

54

To use the steam tables for calculating the departure functions, we can use the following
relationships.
h dep hT , P hTideal
,P
s dep sT , P sTideal
,P

From the steam tables


kJ
kJ
hT , P 2151.0 and sT , P 4.4728

kg
kg K

We need to calculate the ideal enthalpies and entropies using the steam tables reference state.
vap
0 .01 C
hTideal
, P h

673.15 K
c ideal
dT
p
273.16 K

kJ
from the steam tables and heat capacity data from Table A.2.2.
We can get h vap 45.1
mol
Using this information, we obtain
kJ
kJ
hTideal
0.008314

, P 45.1

mol
mol K

673.15 K

3.47 1.45 10
273.16 K

kJ
hTideal
, P 59.14
mol

Now, calculate the ideal entropy.


vap
0 .01 C
sTideal
, P s

673.15 K c ideal
p

273.16 K

P
dT R ln 2
P1

From the steam tables:


kJ
s vap 0.01 C 0.165
mol K

Substitute values into the entropy expression:

55

0.121 10 5
T
dT
T2

673.15 K
5
kJ
3.47
30

3 0.121 10

dT

1
.
45
10
ln

mol K 273.16 K T
0.000613
T3

sTideal
, P 0.165 0.008314

J
sTideal
, P 107 mol K

Now, calculate the departure functions:


kJ
kJ
kJ
h dep 2151.0 0.0180148 kg/mol 59.14
20.4

mol
mol
kg
kJ
0.0180148 kg/mol 0.107 kJ 0.0264 kJ
s dep 4.4728

mol K
mol K
kg K

Table of Results
Generalized
Percent Difference
Steam Tables
Tables
(Based on steam tables)
kJ
h dep
mol
kJ
s dep
mol K

-18.62

-20.4

9.9

-0.0231

-0.0264

12.5

56

5.26
State 1 is at 300 K and 30 bar. State 2 is at 400 K and 50 bar. The reduced temperature and
pressures are

30 bar
0.616
48.74 bar
300 K
T1, r
0.982
305.4 K
P1, r

50 bar
1.026
48.74 bar
400 K
T2,r
1.31
305.4 K
P2,r

and

0.099
By double interpolation of data in Tables C.3 and C.4
( 0)

h dep
T1, r , P1, r
RT
c

h dep
T2 , r , P2 , r
RT
c

(1)

0.825

h dep
T1, r , P1, r
RT
c

0.711

h dep
T2 , r , P2 , r
RT
c

( 0)

0.799

(1)

0.196

Therefore,
h dep
T1, r , P1, r
RT 0.825 0.099 0.799 0.904
c

h dep
T2 ,r , P2 ,r 0.711 0.099 0.196 0.730
RTc

The ideal enthalpy change from 300 K to 400 K can be calculated using ideal cP data from Table
A.2.1.
400 K

hTideal
R
T
1

1.131 19.225 10

T 5.56110 6T 2 dT 717.39 R

300 K

The total entropy change is

57

dep

dep

h hT , P hTideal
hT , P
T2
1, r
1, r
1
2,r
2,r
h R 0.904TC 717.39 0.730TC

J
h 8.314
0.904305.4 K 717.39 K 0.730305.4 K
mol K

J
h 6406.2

mol

Using the data in Table C.5 and C.6


s dep
T1, r , P1, r

s dep
T2 , r , P2 , r

0.601 0.099 0.756 0.676

0.394 0.099 0.224 0.416

Substituting heat capacity data into Equation 3.62, we get


400 K 1.131 19.225 10 3 T 5.561 10 6 T 2
50 bar
dT ln
s ideal R

T
30 bar
300 K
s ideal 1.542 R

Therefore,
s sT , P s ideal sT , P R0.676 1.542 0.416
1, r
1, r
2,r
2,r
dep

dep

J
s 14.98

mol K

58

5.27
The turbine is isentropic. Therefore, we know the following
dep

dep

s sT , P s ideal sT , P 0
1, r
1, r
2,r
2,r

Using the van der Waals EOS, we can find P1,r, which leaves one unknown in the above
equation: T2.

P1

3
82.06 cm atm 623.15 K

mol K

3
3
600 cm 91 cm

mol
mol

atm cm 3
91 10 5

mol

3
600 cm

mol

P1 75.19 atm 76.19 bar

Calculate reduced temperature and pressures using data from Table A.1.1
76.19 bar
1.8
42.44 bar
623.15 K
T1, r
1.68
370.0 K
P1, r

P2,r

1.013 bar
0.024
42.44 bar

Also,

0.152
From Tables C.5 and C.6:
s dep
T1, r , P1, r

0.327 0.152 0.102 0.343

Substituting heat capacity data into Equation 3.62, we get


s

ideal

T2 1.213 28.785 10 3 T 8.824 10 6 T 2

1 atm

R
dT ln

75.19 atm
T

623.15 K

Therefore,

59


s dep
T2
1.213 28.785 10 3 T 8.824 10 6 T 2

T2 , r , P2 , r
R 0.343
dT

4
.
32

T
R

623.15 K

We can solve this using a guess-and-check method


T2 600 K : T2,r 1.62

J
s 33.84

mol K
T2 450 K : T2,r 1.22

J
s 0.77

mol K
T2 446.6 K : T2,r 1.21

J
s 0
mol K
Therefore,
T2 446.6 K

60

5.28
A reversible process requires the minimum amount of work. Since the process is reversible and
adiabatic
s 0

which can be rewritten as


dep

dep

s sT , P s ideal sT , P 0
1, r
1, r
2,r
2,r
Calculate reduced temperature and pressures using data from Table A.1.1
1 bar
0.0217
46.0 bar
300 K
T1, r
1.57
190.6 K
P1, r

P2,r

10 bar
0.217
46.0 bar

From Tables C.5 and C.6:


s dep
T1, r , P1, r

0.00457 0.008 0.0028 0.0046

Substituting heat capacity data into Equation 3.62, we get


s

ideal

T2 1.702 9.081 103T 2.164 106 T 2

10 bar
R
dT ln

T
300 K
1 bar

Therefore,

s dep
T2
3
6 2

T
T
1
.
213
28
.
785
10
8
.
824
10

T2 , r , P2 , r
s R 0.0046
dT 2.303
T
R

300 K

We can solve using a guess-and-check method


T2 400 K : T2,r 2.10
J
s 4.98

mol K
T2 385 K :

T2,r 2.02

61

J
s 1.42

mol K
T2 379 K :

T2,r 1.99
J
s 0.018

mol K

Therefore,
T2 379 K

An energy balance reveals that


h2 h1 h ws
We can calculate the enthalpy using departure functions. From Tables C.3 and C.4:
h dep
T1, r , P1, r
RT
0.0965 0.008 0.011 0.0966
c

h dep
T2 , r , P2 , r
RT
0.0614 0.00890.015 0.0613
c

Ideal heat capacity data can be used to determine the ideal change in enthalpy
379 K

h ideal R 1.702 9.081 10 3 T 2.164 10 6 T 2 dT


300 K

Therefore,
379 K

J
3
6 2

190
.
6
K
0
.
0966
0
.
0613
1
.
702
9
.
081
10
T
2
.
164
10
T
dT
h 8.314

mol K

300 K

J
w s h 3034.2

mol
and

mol
J

3034
.
2
W S 1 / 30

mol 101.1 W
s

62

5.29
Equation 4.71 states

1 v

v T P
v
v

T P
This can be substituted into Equation 5.75 to give

JT

vT 1
cP

63

5.30
For an ideal gas

R
v


T P P
Therefore,
RT

P
v v 0
JT
cP
cP
This result could also be reasoned from a physical argument.

64

5.31
The van der Waals equation is given by:

RT
a

v b v2

(1)

The thermal expansion coefficient is given by:


1 v
1 T

v T P v v P

(2)

Solving Equation 1 for T:


a v b

T P 2

v R

Differentiating by applying the chain rule,

a 1 v b 2a Pv 3 av 2ab
T

P 2
v P
v R R v3
Rv 3
Substitution into Equation 2 gives

Rv 2
Pv 3 av 2ab

Substituting Equation 1 for P gives b in terms of R, T, v, a , and b:

Rv 2 v b
2
RTv 3 2av b

The isothermal compressibility is given by:


1 v
1 P

v P T
v v T

From the van der Waals equation:


RT
2a RTv 3 2av b
P


2
v b 2 v 3
v 3 v b
v T

so

65

(3)

v 2 v b
2
RTv 3 2av b
2

For the Joule-Thomson coefficient, we can use Equation 5.75:

JT
c Pideal

T v
T P

Preal
2
v
T 2 dP
Pideal
T P

Substituting the van der Waals equation into Equation 3 gives


RTv 3 2av b
T
2av b
T


3
v b Rv
v b Rv 3
v P
2

(4)

Thus, the second derivative becomes:


2T
1 T
2a 6av b
T
2

3
2
Rv 4
v b v b v P Rv
v P

or simplifying using Equation 4,


2T
2a v 3b
2
Rv 4
v P

(5)

Substituting Equations 5 and 4 into Equation 5.75 gives:


bRTv 3 2avv b
2
RTv 3 2av b

Preal
RTv 4
dP
c Pideal
2av 3b
Pideal
2

JT

At a given temperature the integral in pressure can be rewritten in terms of volume using the van
der Waals equation to give:

66

bRTv 3 2avv b
2
RTv 3 2av b
vreal
RTv RTv 3 2av b 2
dv

2av 3b
v b 2
videal
2

JT
c Pideal

67

5.32
We can solve this problem by using the form of the Joule-Thomson coefficient given in Equation
5.75. The following approximation can be made

v
v

T P T P
At 300 C,
v350 C,1MPa v250 C,1MPa
v


350 250 C
T P
m3
m3
0.28247

0.23268
v
kg
kg


350 250 C
T P
m3
m3
v

0.0005

0.0005
T P
kg C
kg K

A similar process was followed to find cP.



h
h
c P

dT P T P

At 300 C,
h
h350 C,1MPa h250 C,1MPa


T
350 250 C

kJ
kJ
3157.7 2942.6
kg
kg

350 250 C

P
h

h 2.15 kJ 2.15 kJ
c P

kg C
kg K
dT P T P
Now, JT can be found.

68

3
3

v
573.15 K 0.0005 m 0.25794 m

T v

kg
kg
K

JT
c P
kJ
2.15

kg K

m3 K

kJ

JT 0.0133

69

5.33
At the inversion line, the Joule-Thomson coefficient is zero. From Equation 5.75:

JT
c Pideal

v
T v

T P
0
Preal
2
v
T 2 dP
T P
Pideal

This is true when the numerator is zero, i.e.,


v

T v 0
T P

For the van der Waals equation, we have


P

RT
a
2
v b v

Solving for T:
a v b

T P 2

v R

so
a 1 v b 2a Pv 3 av 2ab
T

P 2

v R R v3
Rv 3
v P
Substituting for P:
RTv 3 2av b
T

v b Rv 3
v P

Hence,
bRTv 3 2avv b
v
T v 0
2
RTv 3 2av b
T P

Solving for T:
2avv b
bRv 3

(1)

70

Substituting this value of T back into the van der Waals equation gives

2avv b a a2v 3b
2
bv 3
v
bv 2

(2)

We can solve Equations 1 and 2 by picking a value of v and solving for T and P. For N2, the
critical temperature and pressure are given by Tc = 126.2 [K] and Pc = 33.84 [bar], respectively.
Thus, we can find the van der Waals constants a and b:
2

RT
Jm 3
a 27 c = 0.137
2
64 Pc
mol

m3
RTc
= 3.88 10 -5

8 Pc
mol

Using these values in Equations (1) and (2), we get the following plot:
Joule-Thomson inversion line
1000

800

600

400

200

0
0

100

200
T [K]

71

300

400

5.34
We can solve this problem using departure functions, so first find the reduced temperatures and
pressures.

50 bar
0.99
50.36 bar
273.15 K
T1, r
0.967
282.4 K
P1, r

P2,r

10 bar
0.2
50.36 bar

Since the ethylene is in two-phase equilibrium when it leaves the throttling device, the
temperature is constrained. From the vapor-liquid dome in Figure 5.5:
T2,r 0.76
T2 214.6

The process is isenthalpic, so the following expression holds


dep

dep

h hT , P hTideal
hT , P 0
T2
1, r
1, r
1
2,r
2,r
Therefore,
dep

dep

hT , P hT , P hTideal
T2
2,r
2,r
1, r
1, r
1
From Table A.2.1:

1.424 14.394 10

214.6 K

hTideal
1 T2

T 4.392 10 6 T 2 dT

273.15 K

From Tables C.3 and C.4 0.085 :


h dep
T1,r , P1,r
RT
c

3.678 0.085 3.51 3.976

Now we can solve for the enthalpy departure at state 2.

72

hTdep, P
2,r

2,r

RTc
hTdep, P
2,r

2,r

RTc

214.6 K

1
3
6 2

1
.
424
14
.
394
10
T
4
.
392
10
T
dT
3.976

282.4 K

273.15 K

3.01

We can calculate the quality of the water using the following relation
hTdep, P
2,r

2,r

RTc

1 x

hTdep,, liq
P
2,r

2,r

RTc

hTdep,, vap
P
2,r

2,r

RTc

where x represents the quality. From Figures 5.5 and 5.6:


hTdep,, liq
P
2,r

2,r

RTc
hTdep,, vap
P
2,r

2,r

RTc

4.6 0.085 5.5 5.068


0.4 0.0859 0.75 0.464

Thus,
x 0.447

55.3% of the inlet stream is liquefied.

73

5.35
Density is calculated from molar volume as follows:

MW
v

2
Substitute the above into the expression for Vsound
:

P
2
Vsound
MW

v

1 P
MW 1 / v s

The following can be shown using differentials:


v
1

v
v2

Therefore,
P
v 2 P
2
Vsound


MW v s
s

74

5.36
From Problem 5.35:
2
Vsound

P
v 2 P


MW v s
s

The thermodynamic web gives:


P s
T P
T s P s
P





v s
s v v P v s T s
s v v T s T T P
T
P

v s
cv

c
s P c P

P
cv
v T s T T

P T

T v v P

If we treat air as an ideal gas consisting of diatomic molecules only


R
P


T v v

cP 7

cv 5

P
T


v P R

Therefore,
P
7 P

v s
5 v
and
Vsound

v 2 7 P

MW 5 v

7 RT

5 MW

Vsound 343 m/s

The lightening bolt is 1360 m away.

75

5.37
From Problem 5.35:
2
Vsound

P
v 2 P


MW v s
s

The thermodynamic web gives:


P s
T P
T s P s
P





v s
s v v P v s T s
s v v T s T T P
so
T P T c P
P



v s
cv T v v P T

For liquids c P cv water at 20 C, so


P
P T

v s
T v v P
However, the cyclic rule gives:
P T v
1


T v v P P T
So
P
P

v s v T
From the steam tables, for saturated water at 20 oC:
m3

.
001002
P = 2.34 kPa and v

kg
For subcooled water at 20 oC:
P = 5 MPa and v . 0009995

m3

kg

76

So
kg kPa
5000 2.34
P
P
P

v s v T v .0009995 .001002 m 3
and
P
Vsound v 2 1414 m/s
v s

77

5.38
(a)
The fundamental property relation for internal energy is

dU Qrev Wrev
Substituting the proper relationships for work and heat, we obtain
dU TdS Fdz

The fundamental property relation for the Helmholtz energy is


dA dU d TS dU TdS SdT

Substitute the expression for the internal energy differential:


dA Fdz SdT

(b)
First, relate the entropy differential to temperature and length.
S
S
dS
dT dz
T z
Z T

Now we need to find expressions for the partial derivatives.


u
TS Fz
S
nc z n

T

T
T z
z
T z
Therefore,
nc
S
a

z n b

T
T z
T

The following statement is true mathematically (order of differentiation does not matter):

A
A


T z T T Z T z

Furthermore,

78

A
S

Z T
T z T

A
F


Z T z T z

S
F

k z z0
Z T
T z

Substituting the expressions for the partial derivatives into the expression for the entropy
differential, we obtain
a

dS n b dT k z z 0 dz
T

(c)
First, start with an expression for the internal energy differential:

U
U
dU
dT
dz
T z
Z T
From information given in the problem statement:
U

na bT
T z
Using the expression for internal energy developed in Part (a) and information from Part (b)
U
S

T F T k z z 0 kT z z 0 0
Z T
z T
Therefore,
dU na bT dT 0dz na bT dT

(d)
We showed in Part (c) that

U
FU
0
z T

79

S
Using the expression for developed in Part B, we obtain
z T
S
FS T kT z z0
z T
(e)
First, perform an energy balance for the adiabatic process.
dU W

Substitute expressions for internal energy and work.

na bT dT Fdz kT z z 0 dz
Rearrangement gives
dT kT z z 0

dz na bT
The right-hand side of the above equation is always positive, so the temperature increases as the
rubber is stretched.

80

5.39
The second law states that for a process to be possible,

suniv 0
To see if this condition is satisfied, we must add the entropy change of the system to the entropy
change of the surroundings. For this isothermal process, the entropy change can be written
ds

cv
P
P
dT dv dv
T
T v
T v

Applying the van der Waals equation:


ds

R
dv
v b

Integrating
s sys R ln

v2
J
11.5
v1
mol K

For the entropy change of the surroundings, we use the value of heat given in Example 5.2:
J
q q surr 600
mol
Hence the entropy change of the surroundings is:
s surr

q surr 600
J

1.6
373
Tsurr
mol K

and
J
suniv s sys ssurr 9.9
mol K
Since the entropy change of the universe is positive we say this process is possible and that it is
irreversible.
Under these conditions propane exhibits attractive intermolecular forces (dispersion). The closer
they are together, on average, the lower the energy. That we need to put work into this system
says that the work needed to separate the propane molecules is greater than the work we get out
during the irreversible expansion.

81

5.40
A schematic of the process is given by:
Gas A in
Pi = 100 bar

Turbine

ws

Ti = 600 K

Gas A
out

Pf = 20 bar
Tf = 445 K

The energy balance for this process is provided below:


h wS
Because the gas is not ideal under these conditions, we have to create a hypothetical path that
connects the initial and final states through three steps. One hypothetical path is shown below:
Pi ,Ti

100
ideal gas
P [bar]

step 1

h
Pf ,Tf

Plow

step 3

20
step 2

445

600

For the first section of the path, we have


P 0

h1

h
dP
P T
P1

If we apply Equation 5.45 we can rewrite the above equation as


P 0

h1

v
v dP
Ti
T P

Pi

For the given EOS:

82

T [K]

R aP
v

2
T P P Ti

Therefore,
P 0

v 0

2aP

aP
J

dP 2467

P
v
dP
b
h1

Ti
T
mol

Pi 10010
Pi 10010 5 i
Similarly for step 3
Pf 2010 5

h3

P 0

2aP

b dP 250

Tf

mol

For step 2, the pressure is zero, so we can use the ideal heat capacity given in the problem
statement to calculate the enthalpy change.
h2

Tf

445 K

Ti

600 K

c P dT 30 0.02T dT 6270 mol

Now sum each part to find the total change in enthalpy:


J
h h1 h2 h3 8487
mol
J
ws 8487
mol
In other words, for every mole of gas that flows through the turbine, 8487 joules of work are
produced.

83

Chapter 6 Solutions
Engineering and Chemical Thermodynamics

Wyatt Tenhaeff
Milo Koretsky
Department of Chemical Engineering
Oregon State University
koretsm@engr.orst.edu

6.1
(a)
The Clausius-Clapeyron equation:

P sat h vap 1
1
ln i
i
R T 373 [K]

101 kPa

vap

dPisat hi dT
2
sat
RT
Pi

or

so

Pi

sat

h vap 1
1
101 kPaexp i

R T 373 [K]

(b) and (c)


Using

kJ
h vap 40.626
mol
we obtain the following table
T [K]

Eqn 6.24
[kPa]

Steam
Tables [kPa]

%
Difference

273.156
278.15
283.15
288.15
293.15
298.15
303.15
308.15
313.15
318.15
323.15
328.15
333.15
338.15
343.15
348.15
353.15
358.15
363.15
368.15
373.15

0.84
1.16
1.58
2.13
2.84
3.76
4.93
6.40
8.24
10.54
13.36
16.82
21.04
26.13
32.26
39.58
48.28
58.56
70.67
84.84
101.35

0.6113
0.87
1.23
1.71
2.34
3.17
4.25
5.63
7.38
9.59
12.35
15.76
19.94
25.03
31.19
38.58
47.39
57.84
70.14
84.55
101.35

37.30%
33.02%
28.30%
24.51%
21.51%
18.62%
15.94%
13.68%
11.71%
9.86%
8.19%
6.75%
5.50%
4.40%
3.42%
2.58%
1.87%
1.25%
0.75%
0.34%
0.00%

The logarithmic trend is well-represented. However, at lower temperatures the Clausius kJ


Clapeyron equation is up to 37% off. The actual heat of vaporization changes from 2501.3
kg
kJ
at 0.01 oC to 2257.0 at 100 oC, a difference of around 10%.
kg
100.00

10.00

Eqn 6.23
Tables

1.00

0.10
273

283

293

303

313

323

333

343

353

363

373

Temperature [K]

(d)
For 100 C to 200 C, we obtain the following table:
T [K]

Eqn 6.24
[kPa]

Steam Tables
[kPa]

%
Difference

373.15
378.15
383.15
388.15
393.15
398.15
403.15
408.15
413.15
418.15
423.15
428.15
433.15
438.15
443.15
448.15
453.15
458.15
463.15

101.35
120.51
142.64
168.11
197.30
230.63
268.55
311.54
360.11
414.82
476.24
545.00
621.74
707.16
801.99
906.98
1022.93
1150.68
1291.10

101.35
120.82
143.28
169.06
198.53
232.1
270.1
313
361.3
415.5
475.9
543.1
617.8
700.5
791.7
892
1002.2
1122.7
1254.4

0.00%
0.26%
0.44%
0.56%
0.62%
0.63%
0.57%
0.47%
0.33%
0.16%
0.07%
0.35%
0.64%
0.95%
1.30%
1.68%
2.07%
2.49%
2.93%

468.15
473.15

1445.10
1613.62

1397.8
1553.8

3.38%
3.85%

10000.00

Eqn 6.23

1000.00

Tables

100.00
373

383

393

403

413

423

433

443

453

463

473

Temperature [K]

Over this range the Clausius-Clapeyron equation represents the data well and is no more than 4
% off. The actual heat of vaporization changes from 2257.0 kJ/kg at 100 oC to 1940.7 kJ/kg
at 200 oC, a difference of around 15%.
(e)
The heat of vaporization can be corrected for temperature as follows
hvap T

Tb

l
cP
dT

hvap Tb c Pv dT
Tb

We can acquire heat capacity data from Appendix A.2, but to simplify the analysis, we will use
an average heat capacity for the vapor.
hvap T 75.4373.15 T 40626 34.13T 373.15

hvap T 56026 41.27T

Substitute this expression into the Clausius-Clapeyron equation


dPisat
Pisat

56026 41.27T dT
RT 2

Integrate:

1
1
1
T
Pisat 101.35 kPa exp 56026
41.27 ln

T 373.15
373.15
R

Now plot the data as before from 0.01 C to 200 C.


T [K]

Eqn 6.24
[kPa]

Steam Tables
[kPa]

%
Difference

273.16
278.15
283.15
288.15
293.15
298.15
303.15
308.15
313.15
318.15
323.15
328.15
333.15
338.15
343.15
348.15
353.15
358.15
363.15
368.15
373.15
378.15
383.15
388.15
393.15
398.15
403.15
408.15
413.15
418.15
423.15
428.15
433.15
438.15
443.15
448.15
453.15
458.15
463.15

0.64
0.91
1.28
1.78
2.43
3.29
4.39
5.81
7.60
9.86
12.66
16.12
20.35
25.49
31.68
39.10
47.91
58.32
70.54
84.80
101.35
120.46
142.40
167.48
196.00
228.29
264.70
305.56
351.26
402.16
458.64
521.10
589.92
665.51
748.27
838.59
936.89
1043.55
1158.98

0.6113
0.87
1.23
1.71
2.34
3.17
4.25
5.63
7.38
9.59
12.35
15.76
19.94
25.03
31.19
38.58
47.39
57.84
70.14
84.55
101.35
120.82
143.28
169.06
198.53
232.1
270.1
313
361.3
415.5
475.9
543.1
617.8
700.5
791.7
892
1002.2
1122.7
1254.4

4.95
4.96
4.24
3.88
3.86
3.65
3.35
3.17
3.03
2.78
2.51
2.28
2.06
1.84
1.58
1.34
1.09
0.82
0.57
0.29
0.00
0.30
0.61
0.94
1.28
1.64
2.00
2.38
2.78
3.21
3.63
4.05
4.51
4.99
5.49
5.99
6.52
7.05
7.61

468.15
473.15

1283.56
1417.67

1397.8
1553.8

8.17
8.76

Water Saturation Pressure as a Function of Temperature


10000.00

Equation

Steam Tables

Psat (kPa)

1000.00

100.00

10.00

1.00
250

300

350

0.10

400

450

500

T (K)

The agreement between the two values at lower temperatures improves significantly at lower
temperatures, but actually worsens at higher temperatures. The agreement could potentially be
improved by not averaging the heat capacity.

6.2
We can find the required pressure by applying the Clapeyron equation:
dP
hl h s

dT
vl v s T

We can find the molar volume of water ice from any number of reference books. At 0 C and 1
bar:
kg
1000
m3
l

v 1.80 10

m3

mol

m3
v s 1.97 10 5

mol

kg

m3

s 917

Also at 0 C and 1 bar,

J
h l h s 6010
mol

If we assume that h l h s and v l v s are independent of temperature and pressure, we can


separate variables in the Clapeyron equation and integrate.
P2 P1

T
ln 2
v v
T1

hl h s

P2 1 10 5 Pa
so

1.80 10

6010 J/mol
5

1.97 10

P2 66.1 bar

268.15 K

ln
273
.
15
K
m / mol

6.3
(a)
At 1 bar, the gas will act as an ideal gas.

J
300 K
8.314
m3
RT
mol K
0.0249
v

P
1 10 5 Pa
mol
The number of moles of vapor are found as follows (neglect molar volume of liquid)
V
n
v
v

0.001 m 3
m3
0.0249

mol

n v 0.0402 mol

(b)
At 21 bar, the gas will not behave ideally. Since we are assuming that the molar volume of
liquid is negligible and the heat of vaporization is independent of temperature, the Clapeyron
equation becomes

dP h vap
v
dT
vT
The molar volume using pressure expansion of the virial equation is
vv

RT 1 B ' P
1

RT
B'
P
P

Substituting this expression into the Clapeyron equation yields

dP

dT

h vap
1

R B ' T 2
P

Separation of variables yields


P2 21 105 Par

P1 1 105 Pa

h vap
1
'
B dP
R
P

T2

dT
2
T1 300 K T

and integration results in

P
ln 2
P1

h vap
B ' P2 P1
R

1 1

T2 T1

We can substitute values for given quantities and constants to solve for T2.
T2 523.3 K

(c)
Using the virial equation,
3

1
1

J
7 m

v v RT B ' 8.314
523
.
3
K
1
10

P

mol K
J
21 10 Pa

m3
v v 0.00164

mol

We can assume the volume occupied by the liquid is negligible. Therefore,


nv

vv

0.001 m 3
m3
0.00164

mol

n v 0.61 mol

6.4
We can use the following computational path to solve for pressure at which graphite and
diamond are in equilibrium at 25 oC.
graphite

P = 1 [atm]
g 2866

g1

diamond

J
mol

v graphdP

g 3

v diam dP
P

P
graphite

g 2 0

diamond

Summing together the three steps we get:


J
g 1[atm] 2866
g1 g 2 g 3
mol
To find the change in Gibbs energy with pressure, we apply the fundamental property relation,
Equation 5.9. At constant temperature:
0
dgi v idP sidT
If the solid is assumed incompressible, we can integrate to get

g i vi dP vi P
Thus the sum of Gibbs energy becomes
P

J
g 1[atm] 2866
v graph dP 0 v diam dP v graph v diam P 1
mol 1
P

Solving

1 cm 3 g 1 m 3
J 1
5

2866

3 P 1.01 10 Pa
12

mol 2.26 3.51 g mol 10 cm

or

P = 1,514 [MPa] = 15,143 [bar]

10

6.5
From the Clausius-Clapeyron equation:
s
hTfus
h lAl h Al
dP

s
s
dT
v lAl v Al
T
v lAl v Al
T

(I)

where h Tfus is the enthalpy of fusion at temperature T. We can get the molar volumes from the
densities:
kg
0.027
MW
mol 1.17 10 5
v lAl

kg
l
2,300

m3

m3

mol

kg
0.027
MW
mol 1.00 10 5
s
v Al

kg
s
2,700

m3

m3

mol

and

so
m3
s
v lAl v Al
1.7 10 6

mol
We can use the following path to calculate for h Tfus .
solid

liquid

T
h Tfus

933

cP dT

933

cPl dT

T = 933 [K]
solid

h fus 10,711

J
mol

liquid

11

hTfus

T
933.45
J
s
l
c P dT 10,711
cP
dT

mol
T
933.45

J
J
l
Using c Ps 20.608 0.0138T
and c P 31.748
, we get:
mol K
mol K
h Tfus 5,819.9 11.68T 0.0069T 2

Back into Equation (I) gives:


5,819.9 11.68T 0.0069T 2
dP
dT
1.7 10 6 T

Integrating:
100 [bar]

5,819.9 11.68T 0.0069T 2


dT
dP
6

1
.
7
10
T

1
933.45 [K]
T

or

100 1105 1.7 10 6 5819.9 ln 933T.45 11.68T 993.45 0.00345T 2 993.452


solving for T gives
T = 934.91 [K]

12

6.6
We can assume that silver acts as an ideal gas at 1500 K. We can also assume the molar volume
of the vapor is much greater than the molar volume of liquid. Therefore, we can use Equation
6.22

dP sat
P sat

h vap dT
RT 2

This can be rearranged to show


dP sat h vap P sat

dT
RT 2
This relation assumes
1. vv>>vl
2. Silver acts as an ideal gas
We can differentiate the expression for pressure in the problem statement to obtain
14260 0.458
14260

0.458 ln T 12.23
exp
2
dT
T
T
T

sat
dP
14260 0.458 sat

P
2
dT
T
T

dP sat

Therefore,
vap
14260 0.458 h

2
T RT 2
T

Therefore,
h vap R14260 0.458T

At 1500 K,

J
14260 0.4581500 K
h vap 8.314
mol K

kJ
h vap 112.8
mol

13

6.7
For a single component system:

Gi g i g
From the fundamental property relation given by Equation 5.9:

dg sdT vdP
We can identify a phase transition from the vertical line of the g vs. T plot, as indicated below.
Since this transition is vertical, i.e., the temperature is constant, the pressure must also be
constant. Thus, we can differentiate the Gibbs energy with respect to temperature at constant
pressure to get:
g

s
T P

Hence the slope of a plot of g (or ) vs. T at any temperature must be the negative of the value of
entropy on the plot for s vs. T. The resulting curve is sketched below.

phase transition;
vertical line indicates
P = const

Thick line denotes


lowest value of

Slope = the negative


value of s at the same
T on the curve above

T*

14

6.8
The ferrite phase has stronger bonds. At room temperature, iron is in the ferrite phase. The
heating to 912 C has the effect of increasing the entropy contribution to the Gibbs energy. At a
high enough temperature, the austenite phase becomes stable, so that its entropy must be greater
than the ferrite phase. If the entropy of the austenite phase is greater, the enthalpy of the ferrite
phase must be greater or else the austenite phase would be stable over the entire temperature
range. Hence, the ferrite phase has stronger bonds.

15

6.9
Since the pressures are low, we can assume ideal gas behavior. We can also assume that the
molar volume of the vapor is much greater than the molar volume of liquid and the heat of
vaporization is independent of temperature. Therefore, we can rearrange Equation 6.24 to obtain

h vap

h vap

J 760 torr
8.314
ln

mol K 400 torr

1
1

353.25 K 333.75 K
kJ
32.3
mol

This value is 7.7% smaller than the reported value.

16

6.10
(a)
The freezing point occurs where there is a discontinuity in the g vs. T plot, as indicated below.
The liquid is at a temperature higher than the freezing point and the solid at lower temperature.
These are demarked below. The melting temperature is 250 K, which occurs at a value g = 3,000
[J/mol]

Pure species "a"


6000

Gibb's Energy, g (J/mol)

1 unit
up
New Freezing
point

4,000
solid
Freezing
point

2,000
liquid

100

200

1.2
units
up

300

Temperature, T (K)
(b)
At constant pressure, the entropy can be found from Equation 5.14. For the solid we have:
g 1,000
g
J

10
s

T
10
T P
mol K

And for the liquid, we get:


g 2,000
g
J

40
s

T
50
T P
mol K

(c)
As we change pressure, we can see how the Gibbs energy changes at any given temperature by
Equation 5.14:

17

g
v
P T

Assuming the molar volumes of the liquid and vapor stay constant over the temperature range
around the melting point, we see that the Gibbs energy of the liquid increases by 1.2 times the
Gibbs energy of the solid, since the molar volume of the liquid is 20% larger. The Gibbs energy
of the new freezing point at higher pressure is schematically drawn on the plot above. For
convenience, we choose the solid to increase by 1 unit on the plot. Thus, the liquid increases by
1.2 units. As the sketch shows, the freezing point, where the two lines intersect, will shift to
higher temperature.

18

6.11
For a single component system the fundamental property relation, Equation 5.9, gives:

dg sdT vdP
We can identify a phase transition from the vertical line of the g vs. P plot, as indicated below.
Since this transition is vertical, i.e., the pressure is constant, the temperature must also be
constant. Thus, we can differentiate the Gibbs energy with respect to pressure at constant
temperature to get:
g
v
P T

Hence the slope of a plot of g vs. P must have a slope that matches the plot for v vs. T. Since the
molar volume of phase is about twice the value of phase , its slope should be twice as big.
The resulting curve is sketched below.
v

phase transition;
vertical line indicates
T = const

v
v

Slope of top line is

Straight line since


v is constant

Thick line denotes


lowest value of g

P*

19

about twice as big as


the slope of bottom
line

6.12
The saturation pressure can be found using the Clausius-Clapeyron equation with the assumption
that the heat of vaporization is independent of temperature. First, we need to use the given data
for the 63.5 C and 78.4 C to find the heat of vaporization.

P sat
R ln 2
P sat
1

J 760 torr
ln

8.314
mol K 400 torr

vap

1
1

1
1
351.55 K 336.65 K

T2 T1
kJ
h vap 42.39
mol
Now we can calculate the vapor pressure at 100 C.
h vap
sat
sat
P3 P2 exp

kJ
42.39

1
1
1
mol

760 torr exp


kJ 373.15 K 351.55 K
T3 T2
0.008314

mol K

P3sat 1760 torr 2.32 atm

In comparison, ThermoSolver gives a value of 2.23 atm, using the Antoine equation.

20

6.13
We can show using the Chain Rule that

gi

T
T

Tg i g i T


T2
1 g i g i

T
T T P T 2

P
P

Using fundamental property relations, Equation 5.14 states


g i

si
P

Therefore,

gi

T
T

Tsi g i Tsi hi Tsi hi

T2
T2
T2

21

6.14
Let T1 922 K, T2 1,300 K
g 2 h2 T2 s 2
dh c P dT
1,300 K

h2 h1

c P dT

922 K

J
h2 h1 c P T2 T1 39,116
mol
c
ds P dT
T
T
J
s2 s1 c P ln 2 85.10
mol K
T1
J
g 2 h2 T2 s2 71,500
mol
Alternative solution using the result from Problem 6.13:

d g
T

dT

g

T 2

hT h1 c P T T1 We must leave h as a function of T

T2
T2

h c T T
g
d T 1 PT 2 1 dT
922 K
g
1300 K


T 1

1 1
T
g
g
J
h1 c PT1 c P ln 2 55.01
T 2 T 1
mol K
T2 T1
T1

J
g 2 71,500
mol

22

6.15
A possible hypothetical solution path is presented below:
monoclinic

orthorhombic
T = 298 [K]

g
1

g
3

T = 368.3 [K]
monoclinic

g = 0
2

orthorhombic

From the diagram, we see that the Gibbs energy for steps one and three can be calculated as
follows:
g m
g1
T
298 K
368 K

dT
P

and
g o
T dT
368 K
P
respectively. We can apply Equation 5.14 from the thermodynamic web
298 K

g 3

s
T P
At 368 K, sulfur undergoes a phase transition, so
g 3m68o
K 0
mo
Using these above relationships, the expression for g 298
K becomes
mo
g 298
K

368 K
m

dT 0

298 K

298 K
o

368 K

dT

368 K

298 K

298 K

368 K

13.8 0.066T dT 11 0.071T dT

J
m o
g 298
K 79.5
mol
Therefore, the transition from the monoclinic to orthorhombic state occurs spontaneously. The
orthorhombic state is more stable.
23

6.16
At the phase transition, the following is true
g
g


T Sr ( s ) T Sr (l )

Using the thermodynamic web, the following can be shown (see Problem 6.13)

h
g / T

2
T P T
The enthalpies can be written as follows
T

h l T 49179

35.146 dT 35.146T 3540

h s T 20285

1500 K
T

37.656 dT 37.656T 16305.4

900 K

g
can also be calculated at 900 K for solid Sr and 1500 K for liquid Sr.
T
gl

83.85

mol K
ref

gs

68.68

mol K
ref

g
We can find at any temperature using the differential equation as follows
T
g

h
g / T
dT 2 dT
T P
T

d T

Substituting our expressions, we get


g l /T

35.146T 3540
g
dT
d T
2
T

83.85
1500 K

24

gl

35.146 ln(T ) 3540 176.04

g s /T

37.656T 16305.4
g
dT
d T
2
T

68.68
900 K
gs

gl
Set
T

37.646 ln(T ) 16305.4 205.5

gl

and solve for T:

T melt 1059.8 K

The enthalpy of melting is defined as

h fus h s T melt h l T melt

Using the expressions developed above


kJ
kJ
kJ
7.41
33.71
h fus 26.30

mol
mol
mol

25

6.17
At the phase transition, the temperature and Gibbs energy of both phases must be equal.
Mathematically, this is equivalent to
g
g


T SiO2 ( s ) T SiO 2 (l )

Using the thermodynamic web, the following can be shown (see Problem 6.13)
h
g / T

2
T P T

Using the definition of enthalpy, we can write the following


hT

href

Tref

dh c P dT

hT href

c P dT

Tref

The enthalpies can be written as follows


h T 738440
l

h s T 856840

85.772 dT

2500 K
T

53.466 0.02706T 1.27 10

5 2

1100 K

g
can also be calculated at 1100 K for solid SiO2 and 2500 K for liquid SiO2.
T
gl

487.3

mol K
ref

gs

903.5

mol K
ref

26

T 2.19 10 9 T 3 dT

We can substitute our expressions for

g
and hT into the above differential equation and
T

separate variables to obtain


T

dT
738440
85
.
772
l

g /T
T

g
2500 K

dT
T
2
T

487.3
2500 K

5 2
9 3

856840
5
3.466
0
.
02706
1
.
27
10
2
.
19
10
T
T
T
dT
s

g /T
T

g
1100 K

dT
T
2
T

903.5
1100 K

Integration provides
g l 9.5268 10 9 85.772 ln(T ) 1052.4T

487.5
T
T
g s 1.82 10 10 T 4 2.12 10 6 T 3 0.01353T 2 927190.9 53.466T ln(T ) 1230T

903.5
T
T

gl
If we plot
T

gs

vs. T, we obtain the following:

27

There are three solutions, but only the solution between 1100 K and 2500 K is physically
meaningful. If we magnify the plot near the middle solution, we find
T 1983 K

The enthalpy of fusion is defined as

h fus h s T melt h l T melt

Using the expressions developed above


kJ
kJ
kJ
9.72
h fus 792.5
782.78

mol
mol
mol

28

6.18
From the Clausius-Clapeyron equation
sat
dPCS

dT

h vap

T vv vl

Assuming:
v v v l

we get
sat
dPCS

dT

h vap

(I)

Tv v

The saturation pressure is given by:


sat
ln PCS
62.7839
2

4.7063 10 3
6.7794 ln T 8.0194 10 3 T
T

(II)

sat
5
4.48 10 Pa. Taking the derivative of Equation II
At T = 373 K, PCS
2

sat
d ln PCS

dT

sat
dPCS

sat
PCS

dT

4.7063 10 3
T2

6.7794
8.0194 10 3 (III)
T

Plugging Equation III into Equation I,


sat h vap
4.7063 10 3 6.7794

8.0194 10 3 PCS

2
T
T2
Tv v

kJ
vap
Solving for vv using hCS
gives:
24.050
2
mol

h vap 4.7063 10 3 6.7794


v

8.0194 10 3

sat
T
TPCS
T2

Pv
B
0.878 1
RT
v
29

m3
6.08 10 3

mol

or
m3
cm3
B 7.4 10 4
740

mol
mol

This value is about 50% higher than the reported value.

Alternative solution:
Following similar development as Problem 6.3:
sat

dPCS
dT

h vap

1
RT 2
B '
sat

PCS 2

h vap
1
sat
sat B' dPCS

2 dT
2
P
RT
CS2

We must be careful about the limits of integration. We need to pick a value of T close so
enthalpy of vaporization is not too different, but far enough away to avoid round off error. If we
sat
5
choose T = 378 K, Equation I gives PCS
5.04 10 Pa. Integrating:
2

4.4810 5 Pa

373
h vap
sat
sat B dPCS

2 dT
2
P
RT
5

378
5.04 10 Pa CS2

'

4.48 105 '


h vap 1
1
5
5

B
ln
4.48

10

5.04

10

5
R 373 378
5.04 10

Solving for B gives:


B' 2.55 107 Pa

or
m 3
B B ' RT 7.9 10 4
mol

30

6.19
Calculate vA, vB, v, VA, VB, and V from the ideal gas law:

RT
0.05 m 3 / mol
P
RT
vB
0.05 m 3 / mol
P
RT
v
0.05 m 3 / mol
P
vA

V A n A v A 0.1 m 3

V B n B v B 0.15 m 3
V ntot v 0.25 m 3

We calculate the partial molar volumes as follows


V

n A n B RT RT 0.05 m 3 / mol

V A

P
P
n A T , P, n B n A
V
V B
n B

n A n B RT RT 0.05 m 3 / mol

P
P
T , P, n A n B

To find the remaining quantities, we can apply Equations 6.44 and 6.46
Vmix n A V A v A n B V B v B
Vmix 20.05 0.05 30.05 0.05 0

vmix x A V A v A x B V B v B
vmix 0

31

6.20
(a)
For a pure species property

va v y a 1
Substitution yields

cm 3
va 1001 800 2.510 100

mol
(b)
From Equation 6.29

V
Va
na

n b ,T , P

We can find V by multiplying the given expression for molar volume by the total number of
moles.

V na nb 100 ya 80 yb 2.5

y a yb
n n
100na 80nb 2.5 a b
y a yb
na nb

Differentiating with respect to na we get,

Va

na

n n
nb
na nb
100na 80nb 2.5 a b 100 2.5
2.5
na nb n
na nb
na nb 2

so

Va 100 2.5 yb 1 ya 100 2.5 yb2


To find the molar volume at infinite dilution, we can use the following relation
Va lim Va
ya 0

cm 3
Va 102.5

mol

32

(c)
Since species A contributes more to a mixture than to a pure species,

v mix 0
Note: The Gibbs-Duhem equation says that species B also contributes more.

33

6.21
Calculate mole fractions:
n1
1 mol

0.2
ntot 5 mol

y1

y3 0.4

y 2 0.4

Calculate v.
Obtain an expression for v:

RT
P

B
2 A
1 P RT y1 y 2 RT

Substitute values:

3
82.06 cm atm 500 K

mol K

50 atm

1 50 9.0 10
2

0.2 0.4 3.0 10 5

cm 3
v 919.0

mol
Calculate V.

V ntot v

cm 3
V 5 mol 919.0

mol

V 4595 cm 3

Calculate v1.
The value of v1 can be found by substituting y1=1 into the expression for v1.

v1

3
82.06 cm atm 500 K

mol K

50 atm

1 50 9.0 10
2

cm 3
v1 698

mol

34

1 0 3.0 10 5

Calculate v2.

v2

3
82.06 cm atm 500 K

mol K

50 atm

1 50 9.0 10
2

0 1 3.0 10 5

cm 3
v2 1067

mol

Calculate v3.

v3

3
82.06 cm atm 500 K

mol K

50 atm

1 50 3.0 10
2

cm 3
v3 882

mol
Calculate V1 .
From Equation 6.29:

nv
V

V1

n1 n 2 , n 3 ,T , P n1 n 2 , n 3 ,T , P
We can substitute the expression for V into this derivative and use the fact that
ntot n1 n2 n3 to obtain
V1

RT
B

2 A

n
n
n
P
n
n
n
n
n
1
2
3
1
2
1
2
3

RT

n1 P
RT
n 2 , n 3 ,T , P

Differentiating we get
V1

RT
P

B
2 A
1 P RT RT

Substitute values:

35

V1

3
82.06 cm atm 500 K

mol K

50 atm

1 50 9 10
2

cm 3
V1 697.5

mol

36

3 10 5

6.22
(a)
By definition:

H
H a
na

T , P , n b , n c

h 5,000 xa 3,000 xb 2,200 xc 500 xa xb xc J/mol


n na nb nc
H nh 5,000na 3,000nb 2,200nc 500

na nbnc

na nb nc 2

H
nbn c
2na nbnc

5,000 500

na T ,P,n ,n
na n b nc 2 na nb n c 3

b c
H
Ha
5,000 500xb x c 1 2xa J/mol
na T ,P,n ,n
b c
(b)
xa xb xc

1
3

Ha 5,018.5 J/mol
(c)
xa 1 , xb xc 0

Ha 5,000 J/mol
(d)
xb 1 , xa xc 0

Hb hb 3,000 J/mol

37

6.23
Let the subscript 1 designate CO2, and 2 designate propane. To calculate the partial molar
volumes, the following formulas will be used:

V1 v y 2

dv
dy 2

v y1V1 y 2V2
Expressions cant be obtained for the molar volume with the van der Waals EOS; therefore, the
problem will be solved graphically. First, obtain an expression for the pressure that contains the
mole fractions of CO2 and propane:
a mix y12 a1 2 y1 y 2 a1a 2 y 22 a 2
bmix y1b1 y 2 b2
y12 a1 2 y1 y 2 a1a2 y 22 a2
RT
P

v y1b1 y 2 b2
v2
Solve for a and b using data from the appendices.
J m3
a1 0.366

mol 2
J m3
a 2 0.941

2
mol

mol
b1 4.29 10 5

m3
mol
b2 9.06 10 5

m3

Now we can create a spreadsheet with the following headings:


y1

y2

amix

bmix

The last column contains the molar volumes obtained by solving the van der Waals equation
with the spreadsheets solver function. After the table is completed, we create the following
graph.

38

v vs. y 2
1.48E-03
1.46E-03

v (m 3 mol -1)

1.44E-03
1.42E-03
1.40E-03
1.38E-03
1.36E-03
y = -0.00008x 2 - 0.00008x + 0.00147
R2 = 0.99946

1.34E-03
1.32E-03
1.30E-03
0

0.1

0.2

0.3

0.4

0.5

0.6

0.7

0.8

0.9

y2 (Mole Fraction)

From the line of best fit, we find


v 8 10 5 y 22 8 10 5 y 2 0.00147

Therefore,
dv
1.6 10 4 y 2 8 10 5
dy 2
and

V1 v y 2 1.6 10 4 y 2 8 10 5

We can find the partial molar volume of propane from the following relationship
v y1V1 y 2V2
V2

v y1V1
y2

Tabulate the values of the partial molar volumes in the spreadsheet and create the following
graph

39

Partial Molar Volumes as a Function of Carbon Dioxide Mole


Fraction

Partial Molar Volume (m

mol -1)

0.0016

V1
V2

0.00155
0.0015
0.00145
0.0014
0.00135
0.0013
0.00125
0

0.1

0.2

0.3

0.4
0.5
0.6
y1 (CO 2 mole fraction)

40

0.7

0.8

0.9

6.24
(a)

ga 40

kJ
mol

Ga g x b

dg
dx b

dg
40 60 RT ln xa 1 ln x b 1 5x a 5xb
dx b
Ga 40xa 60x b RT xa ln x a x b ln xb 5xa x b

x b40 60 RT ln xa 1 ln x b 1 5x a 5xb

Ga 40xa xb RT xa xb ln x a 5xb2

Ga 40 RT ln x a 5 xb2
Ga 40

kJ
kJ
8.314(300)
ln 0.2 5(0.64)
40.8
mol
mol
1000

Ga
Gmix ng x a ga xb gb
g xa ga x bgb RT x a ln xa xb ln x b 5 x a xb
Gmix nT RT x a ln xa xb ln x b 5xa x b 2.2 kJ

(b)

gmix hmix Tsmix

Assume the entropy of mixing is ideal:


hmix 5x a xb 0

so
h hmix hsensibleheat 0

so

hsensibleheat 0 and T goes down

41

6.25
To find V1 and V2 , we can read values directly from the graphs. Calculate mole fractions

x1

1
0.2
5

At x1 0.2 ,
cm 3
V1 46.5

mol
cm 3
V2 69.8

mol
The following relationships are employed to calculate the molar volumes of pure species

v1 lim V1
x1 1

v 2 lim V2 lim V2
x 2 1

x1 0

From the graph

cm 3
v1 50

mol
cm 3
v2 70

mol
Therefore,


280 cm

V1 50 cm 3
V2

To calculate the total volume, we can use

V n1V1 n2V2
Substituting the values, we find

V 146.5 469.8 325.7 cm3

42

Therefore

V
325.7

ntot
1 4

cm 3
v 65.14

mol

Using Equation 6.43 we can calculate the change in volume.

Vmix V V1 V2 325.7 50 280

Vmix 4.3 cm 3

43

6.26
(a)
Expression for hmix :
hmix X i H i X i hi X Cd H Cd X Sn H Sn X Cd hCd X Sn hSn

Multiply both sides by the total number of moles

H mix nCd H Cd n Sn H Sn nCd hCd n Sn hSn


Therefore,

H mix Cd

H mix

nCd

H Cd hCd
n Sn ,T , P

(b)
We can show by repeating Part (a) for Sn that

H mix Sn H Sn hSn
Equation 6.65:

dhmix
dX Sn
H mix Sn hmix X Cd dhmix
dX Cd

H mix Cd

hmix X Sn

Since,
hmix 13000 X Cd X Sn

We get,

dhmix
d
13000 X Cd X Sn 13000 X Cd X Sn

dX Sn
dX Sn
dhmix
d
13000 X Cd X Sn 13000 X Sn X Cd

dX Cd
dX Cd
Therefore, for 3 moles of cadmium and 2 moles of tin at 500 C:

44

2
H Cd hCd H mix Cd 13000 X Cd X Sn X Sn13000 X Cd X Sn 13000 X Sn

J
H Cd hCd H mix Cd 2080
mol
and

J
2
H Sn hSn H mix Sn 13000 X Cd
4680
mol
(c)
Equation 6.37:
nCd d H mix Cd n Sn d H mix Sn 0

Differentiate with respect to XCd:

nCd

d H mix Cd
dxCd

n Sn

d H mix Sn
dxCd

where

d H mix Cd
dX Cd

d H mix Sn
dX Sn

26000 X Sn 26000 26000 X Cd


26000 X Cd

Therefore,

nCd

d H mix Cd
dxCd

nSn

d H mix Sn
dxCd

xCd ntot 26000 26000 xCd 1 xCd ntot 26000 xCd

Inspection of the above expression reveals that

nCd

d H mix Cd
dxCd

n Sn

d H mix Sn
dxCd

(d)
A graphical solution can be found using the tangent-slope method discussed on pages 285-287:

A plot of a line tangent to the enthalpy of mixing curve at XCd = 0.6, is given below:

45

Heat of mixin g in cadmium (Cd )-Tin (Sn) s ystem


6000
5400

mol

4800
4200
3600

data

hmix

3000
2400
1800
1200
600
0
0

0 .2

fit to:
J
hmix 13, 000 xCd x Sn

mol

0 .4

0 .6

0 .8

xCd

The intercepts give the respective partial molar quantities as follows:

J
H Cd hCd 2050
mol
J
H Sn hSn 4800
mol
The values using the graphical method are reasonably close to the analytical method.

46

6.27
The following can be shown with the Gibbs-Duhem equation

0 x1V1 x 2V2
Differentiation with respect to x1:
0 x1

dV1
dV
x2 2
dx1
dx1

If the partial molar volume of species 1 is constant, the Gibbs-Duhem equation simplifies to
0

dV2
dx1

Therefore, the partial molar volume of species 2 is also constant.


Note that in this case, since the partial molar volume of species 1 is constant:

V1 v1
and similarly for species 2:
`

V2 v2

Hence, the molar volume can be written:

v x1V1 x2V2 x1v1 x2v2


This is known as Amagats law.

47

6.28
(a)
Let species 1 represent HCl and species 2 represent H2O. An expression for the enthalpy of the
solution is

h x1h1 x 2 h2 hmix
which can be written
~
h x1h1 x2 h2 x1hs

Therefore,
~
H n1h1 n2 h2 n1hs

To use the heat of solution data in Table 6.1, we need to determine the values of n1 and n2
consistent with the convention used in the table. As seen in Example 6.6,

x1

n1
1

n1 n2 1 n

For this problem


x1 0.2

Therefore,

n1 1
n2 n 4
Now we can find expressions for the partial molar enthalpies.

H H 2 O H 2

n
2 n1 , T , P
~
dhs
H H 2 O H 2 h2 n1
dn2

~
dhs
H H 2 O hH 2 O H 2 h2 n1
dn2
Using the data in Table 6.1 for n 4 ,

48

~
J

hs 61,204
mol solute
J
J

64049
56852
~
~
~

dhs hs n 5 hs n 3
mol solute
mol solute

5 mol 3 mol
5 mol 3 mol
dn2
~
dhs
J

3598.5
dn2
mol mol solute
Therefore,

J
H H 2 O hH 2 O 1 mol solute 3598.5
3599

mol mol solute


mol

We can calculate H HCl hHCl using Equation 6.46

H HCl hHCl

hmix x H 2 O H H 2 O hH 2 O
x HCl

xHCl h~s xH O H H O hH O
2

x HCl

J
J
0.8 3599

0.2 61204

mol
mol
J

H HCl hHCl
46808
0.2
mol
(b)
For n1 2 and n2 80 ,
n n2
82
1
n 1
1 40
2
n1

The new values for the number of moles consistent with Table 6.1

n1 1 mol

n2 40 mol
Using the data in the table for n 40 ,
~
~
~
dhs hs n 50 hs n 30

50 mol 30 mol
dn2
Interpolating the data in Table 6.1

49

~
J

hs n 30 72428
mol solute
Therefore,
J
J

73729
72428
~

dhs
J

mol solute
mol solute 65.05
and

50 mol H 2O 30 mol H 2O
dn2
mol mol solute

J
J

H H 2 O hH 2 O 1 mol solute 65.05


65.05

mol mol solute

mol H 2 O

50

6.29
First perform an energy balance on the mixing process.

hmix q
We can calculate hmix using data from Table 6.1. Referring to Equation E6.7A, we find
~
hmix x HCl hs

Calculate x HCl :

x HCl

wHCl
0.30
MW HCl
36.46

0.175
0
.
30
0.70
w
wHCl
H 2O

MW HCl MW H 2O 36.46 18.0148

Heats of data are tabulated for a solution containing one mole of the solute for various amounts
of water. Thus, we need to calculate how many moles of water must be added to HCl to obtain
the above mole fraction.
1 mol HCl
; where n is the number of moles of H2O
1 mol HCl n
n 4.71 mol H 2 O

x HCl

By interpolation of data from Table 6.1, we get


~
J
hs 63224
mol

(for n 4.71 )

Therefore,

J
J
hmix 0.175 63224
11064

mol
mol

and

J
q hmix 11064
mol

51

6.30
To calculate the enthalpy of mixing from Table 6.1, we must use the following expression

~
hmix x H 2 SO 4 hs
The mole fraction of sulfuric acid is

x H 2 SO4

1
1 n

where n is the number of moles of water.


Equation 6.47 states

hmix 74.4 x H 2 SO 4 x H 2 O 1 0.561x H 2 SO

For n 1 , x H 2 SO 4 0.5 and x H 2 O 0.5


Table 6.1:

~
J
hs 31087
mol

J
J
15543.5
hmix 0.5 31087

mol
mol

Equation 6.47: hmix 74.40.50.51 0.5610.5

J
hmix 13383
mol
The following table was made
n [mol H2O]

x H 2 SO 4

1
2
3
4
5
10
20
50
100

0.5
0.333333
0.25
0.2
0.166667
0.090909
0.047619
0.019608
0.009901

hmix [kJ/mol]

hmix [kJ/mol]

(Table 6.1)

(Eq. 6.47)

-15543.5
-14978.7
-13001.8
-11414
-10174.2
-6367.27
-3548.43
-1497.22
-762.238

-13382.7
-13441.6
-11993.5
-10568.4
-9367.17
-5835.17
-3284.01
-1414.49
-725.289

52

% Difference
14.94
10.82
8.07
7.69
8.26
8.72
7.74
5.68
4.97

As you can see, the percent difference between the two methods decreases as the mole fraction
of sulfuric acid decreases. Although Equation 6.47 fit data at 21 C, while the Table 6.1
tabulates data taken at 25 C, we do not expect the temperature dependence to account for all the
observed difference. The table and equation come from different experimental data sets, and
also represent measurement uncertainty. Nevertheless, the agreement is reasonable.

53

6.31
To calculate the enthalpy of mixing from Table 6.1, we must use the following expression
~
hmix x HCl hs

The mole fraction of HCl is

x HCl

1
1 n

where n is the number of moles of water. The following table was made using these two
equations.
n [mol H2O]

x1

~
J

hs
mol HCl

J
hmix
mol

1
2
3
4
5
10
20
50

0.5
0.333
0.25
0.2
0.167
0.091
0.048
0.020

-26225
-48819
-56852
-61204
-64049
-69488
-71777
-73729

-13112.5
-16273
-14213
-12240.8
-10674.8
-6317.09
-3417.95
-1445.67

100

0.0099

-73848

-731.168

54

6.32
A schematic for the process is given below. The inlet streams are labeled 1 and 2 and the
exit stream 3.
q

50 wt% NaOH

10 wt% NaOH

50 wt% H2O

90 wt% H2O

Stream 1

Stream 3

H 2O

Stream 2

The energy balance for this process reduces to

Q H 3 H 2 H1
We first convert from weight percentage to mole fraction. For stream 1,

x NaOH ,1

wNaOH
0.50
MW NaOH
40

0.311
0.50
0.50
w H 2O
wNaOH

MW NaOH MW H 2O 40 18.0148

and for stream 3,

x NaOH ,3

wNaOH
0.10
MW NaOH
40

0.048
0.10
0.90
wH 2 O
wNaOH

MW NaOH MW H 2O 40 18.0148

We now calculate the moles of water per mole of NaOH so that we can use Table 6.1:

x NaOH

1
1 nH 2 O

Therefore, for every mole of NaOH


n H 2 O,1 2.21

55

n H 2 O,3 19.8

Since enthalpy is a state function, we can choose any hypothetical path to calculate the change in
enthalpy. One such path is shown below. The box in our original schematic is depicted with
dashed lines below. We pick a basis of 1 mole NaOH. In step A, the inlet stream is separated
into its pure components. In step B, 17.6 additional moles of water are added to the pure water
stream. Finally the H2O and NaOH streams are remixed

1 mol NaOH

Step C

Step A

1 mol NaOH

2.2 mol H2O

1 mol NaOH
19.8 mol H2O

Step B

hmix=0
2.2 mol H2O

19.8 mol H2O

17.6 mol H2O

The enthalpy change is found by adding each step

H 3 H 2 H 1 H A H B H C
Since H B represents the mixing of water with water, H B 0 .
The enthalpies of mixing for steps A and C can be related to enthalpy of solution data from Table
6.1:
~
J

hs ,1 23906
mol NaOH
~
J

hs ,3 42858
mol NaOH
Note: The enthalpy of solution for Stream 1 is calculated by extrapolation. Generally,
extrapolation should be avoided, but it is necessary to complete this problem, and we are
not extrapolating very far.
For step A, we need the negative value of the heat of solution of stream 1. Thus for a basis of 1
mole NaOH:
H A 23906 J

56

while for step C:

H C 42858 J
Now adding the enthalpies of each step per 1 mole of NaOH:
H 23906 0 42858 18952 J

To get the total heat that must be removed per mole of product solution, we divide by the number
of moles of product per mol of NaOH:
q

H
J
910
n NaOH
mol

57

6.33
The partial molar property can be written as follows:

n K
K1 T
n1 T , P ,n2 ,n3
Applying the chain rule to the above relationship:

n
K1 k T
n1

nT
T , P ,n2 ,n3
n1 T , P ,n2 ,n3

K1 k nT
n1 T , P ,n2 ,n3

(1)

. At constant T and P, we can write,


Now focus on nT

n
1 T , P ,n2 ,n3
k
k
k

dx3
dx2
dx1
dk
x1 T , P, x2 , x3
x2 T , P, x1 , x3
x3 T , P, x1 , x2
Therefore,

k
k
x1
k
x 2



n1 T , P ,n2 ,n3 x1 T , P , x2 , x3 n1 T , P ,n2 ,n3 x2 T , P , x1 , x3 n1 T , P ,n2 ,n3
k
x3


x3 T , P , x1 , x2 n1 T , P ,n2 ,n3
but,
x1

n1
n1 n2 n3

so
x1
n1
1
1

1 x1
2
nT
n1 T , P ,n2 ,n3 n1 n2 n3 n1 n2 n3
Similarly,

58

(3)

(2)

x 2
n2
x

2
2
nT
n1 n2 n3
n1 T , P ,n2 ,n3

(4)

x3
n3
x

3
2
nT
n1 n2 n3
n1 T , P ,n2 ,n3

(5)

and

Substituting Equations 2, 3, 4, and 5 into the expression 1 for K1 and simplifying:

1 x1 k
x2 k
x3
K1 k
x1 T , P, x 2 , x 3
x 2 T , P, x1 , x 3
x3 T , P, x1 , x 2
Utilize the fact that x1 x 2 x3 1

k
k

(6)
x3 k


K1 k x 2

x1 T , P, x , x x3 T , P, x , x
x1 T , P, x , x x 2 T , P, x , x
2
3
1
3
2
3

1
2

When we hold species 3 constant:


k
k

dk
dx1
dx 2
x 2 T , P, x1 , x3
x1 T , P, x2 , x3
k
k
dx1 k
dx2


x 2 T , P , x3 x1 T , P , x2 , x3 dx2 x2 T , P , x1 , x3 dx2
Thus,
k
k
k



x2 T ,P , x3
x1 T ,P , x2 , x3 x2 T ,P , x1 , x3

(7)

When we hold species 2 constant, a similar analysis shows:


k
k
k



x1 T , P , x2 , x3 x3 T , P , x1 , x2
x3 T , P , x2
Substituting Equations 7 and 8 into Equation 6 gives

59

(8)

k
k

K1 k x 2
x3
x2 T , P , x3
x3 T , P , x2
Furthermore, the above analysis can be extended to m components. In general,
k

K i k xm
m i
xm T , P , x ji ,m

60

6.34
The expression can be found by employing the Gibbs-Duhem equation:

0 n1V1 n2V2
Differentiate with respect to x1 and then divide by the total number of moles:
0 x1

dV
dV
dV
dV1
x 2 2 x1 1 1 x1 2
dx1
dx1
dx1
dx1

Differentiate the expression given in the problem statement.


dV1
5.28 5.28 x1
dx1
Substitution of this result into the Gibbs-Duhem equation and rearrangement yields
dV2 5.28 x12 5.28 x1

5.28 x1
dx1
x1 1

Integrate:
cm 3
V2 2.64 x12 C

mol
To determine C, we can use the density information given in the problem statement.
V2 x 2 1 V2 x1 0 C v 2
where
C v2

1
2

MW2

cm 3
109.58

mol
0.768 g/cm 3

84.16 g/mol

Therefore,
cm 3
V2 2.64 x12 109.58

mol

61

6.35
An expression for the enthalpy of the solution is

h x1h1 x 2 h2 hmix
which is equivalent to
~
h x1h1 x 2 h2 x1hs

Multiplication by the total number of moles yields


~
H n1h1 n2 h2 n1hs

To use the heat of solution data in Table 6.1, we need to determine the values of n1 and n2
consistent with the convention used in the table. As seen in Example 6.6,
x1

n1
1

n1 n2 1 n

For this problem


x1 0.33

Therefore,
n1 1
n2 n 2
Now we can find expressions for the partial molar enthalpies.
H

H H 2 O H 2
n2 n1 ,T , P
~
dhs
H H 2 O H 2 h2 n1
dn2

~
dhs
H H 2 O hH 2 O H 2 h2 n1
dn2
Using the data in Table 6.1 for n 2 ,

62

J
J

2,787
812
~
~
~

dhs hs n 3 hs n 1
mol solute
mol solute

3 mol 1 mol
3 mol H 2O 1 mol H 2O
dn2
~
dhs
J

987.5
dn2
mol mol solute
Therefore,

J
H H 2 O hH 2 O 1 mol solute 987.5
987.5

mol mol solute


mol

Calculate the partial molar enthalpy:


J
H H 2O hH 2 O 987.5
mol
From the saturated steam tables at 25 C:
kJ
hH 2 O 104.87
kg
kJ
hH 2 O 1.89
mol
Now we can find the partial molar enthalpy

kJ
J
J
H H 2 O 1.89
0.988
0.90

mol H 2 O
mol H 2 O
mol H 2 O

63

6.36
(a)
Calculate the mole fraction of sulfuric acid

x1

w1
MW 1
w1
w2

MW 1 MW 2

0.20
98.078

0.044
0.20
0.80

98.078 18.0148

Calculate n to use in Table 6.1:


1
0.044
1 n
n 21.7 mol H 2 O
x1

Interpolating from Table 6.1


~
J
hs 74621
mol
Now calculate the heat transfer
~

q hmix x1hs 0.044 74621


mol

J
q 3283
mol
(b)
Calculate the mole fraction of pure sulfuric acid. Consider a mixture of 20 kg of 18 M sulfuric
acid and 80 kg of water. Find the mass of sulfuric acid present.

VH 2 SO4

mH 2 SO4

H 2 SO4

20 kg
10.9 L
1.84 kg/L

nH 2 SO4 VM 10.9 L 18 mol/L 196.2 mol

m H 2 SO 4 n H 2 SO4 MW H 2 SO 4 196.2 mol0.098 kg/mol 19.2 kg

Since both the initial (i) and final (f) states contain mixtures, to get the enthalpy of mixing, we
need to calculate the relative differences follows:

64

~
~
q hmix x1, f hs, f x1,i hs ,i
calculate the mole fraction in the final state
w1
MW 1

0.192
98.078

0.042
x1, f
w1
w2
0.192
0.808

MW 1 MW 2 98.078 18.0148

Calculate n to use in Table 6.1:


1
0.042
1 n
n 22.8 mol H 2 O
x1, f

Interpolating from Table 6.1


~
J
hs , f 74689
mol
For the initial 18 M sulfuric acid:
w1
MW 1

0.192
98.078

0.81
x1,i
w1
w2
0.192
0.008

MW 1 MW 2 98.078 18.0148

Calculate n to use in Table 6.1:


1
0.81
1 n
n 0.23 mol H 2 O
x1,i

We must extrapolate from Table 6.1. To do this we wish to extend the trend at low water
concentration. A plot of the data in Table 6.1 is useful. A semi-log plot follows:

65

3200 [J/mol]
enthalp of solution [J/mol solute]

-10000
-20000
-30000
-40000
-50000
-60000
-70000
-80000
-90000
0.1

10

100

~
J
hs ,i 3200
mol
Now calculate the heat transfer
~
~

J
J
q hmix x1, f hs , f x1,i hs,i 0.042 74689
0.81 3200

mol
mol

J
q 710
mol

(c)
Calculate the mole fraction of sodium hydroxide

x1

w1
MW 1
w1
w2

MW 1 MW 2

0.20
40

0.101
0.20
0.80

40 18.0148

Calculate the n value to use in Table 6.1:


1
0.101
1 n
n 8.9 mol H 2 O
x1

Interpolating from Table 6.1


~
J
hs 41458
mol

66

Now calculate the heat transfer


~

q hmix x1hs 0.101 41458


mol

J
q 4187
mol
(d)
Calculate the mole fraction of ammonia

x1

w1
MW 1
w1
w2

MW 1 MW 2

0.20
17.03

0.209
0.20
0.80

17.03 18.0148

Calculate the n value to use in Table 6.1:


1
0.209
1 n
n 3.78 mol H 2 O
x1

Interpolating from Table 6.1


~
J
hs 33153
mol
Now calculate the heat transfer
~

q hmix x1hs 0.209 33153


mol

J
q 6929
mol

67

6.37
Let species 1 designate ethanol and species 2 designate water. We need to obtain an expression
for the molar volume, so first, convert the given the mass fractions and densities to mole
fractions and molar volumes.

x1

Mole fractions:

Specific molar volumes:

w1
MW 1
w1
w2

MW 1 MW 2

v vMW mixture
where MW mixture x1 MW 1 x 2 MW 2

Using this set of equations, the following table was made.


Mole Frac. EtOH
0.000
0.042
0.089
0.144
0.207
0.281
0.370
0.477
0.610
0.779
1.000

Mole Frac. H2O v (ml/mol)


1.000
18.05
0.958
19.54
0.911
21.18
0.856
23.11
0.793
25.47
0.719
28.34
0.630
31.85
0.523
36.19
0.390
41.65
0.221
48.73
0.000
58.36

The following graph plots the data. The trendline relates v to x1.
60.00

v (ml/mol)

50.00
40.00
2
v = 4.5491x1 + 35.918x1 + 17.957

30.00

R2 = 1

20.00
10.00
0.00
0.000

0.200

0.400

0.600

Mole Fraction EtOH (x1)

68

0.800

1.000

Now, we can calculate V1 .


V
nv


V1
n1 n 2 , n 3 ,T , P n1 n 2 , n 3 ,T , P
We can substitute the trendline for V into this derivative and use the fact that ntot n1 n2 to
obtain

n12

V1
4.5491
35.918n1 17.957n1 n2

n1 n2
n1

n 2 ,T , P

Differentiating we get
2n n n n 2
2
1 53.875 ml
V1 4.5491 1 1

mol

n1 n2 2

ml
V1 4.5491 2 x1 x12 53.875
mol

Calculate V2 :
V2

n2

4.5491 n1
35.918n1 17.957n1 n2

n1 n2
n1 ,T , P

Differentiating we get
n2
ml
1
V2 4.5491
17.957

mol
n1 n2 2
ml
V2 4.5491x12 17.957
mol

Plotting V1 and V2 vs. x1 we obtain

69

60
59

20
19

58
57
56
55

18
17
16
15

54
53
52

14
13
12
0

0.2

0.4

0.6

0.8

H2O Partial Molar


Volume (ml/mol)

EtOH Partial Molar


Volume (ml/mol)

Partial Molar Volumes vs. EtOH Mole Fractions

EtOH
H2O

Mole Fraction EtOH (x1)

(b)
v mix can be calculated using Equation 6.46

v mix x H 2 O V H 2 O v H 2 O x EtOH V EtOH v EtOH


From the data table in Part (a),
ml
v EtOH v x1 1 58.36
mol
ml
v H 2 O v x1 0 18.05
mol
Using the expressions for partial molar volumes

ml
V1 4.5491 20.5 0.52 53.875 57.29
mol
ml
V2 4.54910.52 17.957 16.82

mol
Therefore,

ml
ml

0.5 57.29 58.36


v mix 0.516.82 18.05

mol
mol

ml
vmix 1.15

mol

70

6.38
We can use the density data given in the problem statement to determine the pure species
properties. For pure ethanol x1 1 :

46 g/mol

cm 3

58.55
v1

1
0.7857 g/cm 3
mol
MW1

cm 3
3
V1 n1v1 3 mol 58.55
175.7 cm

mol

For pure formamide x1 0 :


45 g/mol

cm 3
39.77

2
1.1314 g/cm 3
mol

cm 3
3
V2 n2 v 2 1 mol 39.77
39.77 cm

mol

v2

MW2

To calculate V and v, interpolate in the data table to obtain the density of the mixture
when x1 0.75 :

0.8550 g/cm 3

Therefore,
v

where

MW

MW 0.7546 g/mol 0.2545 g/mol 45.75 g/mol

Substitute numerical values:


v

45.75 g/mol

0.8550 g/cm

53.51 cm 3 /mol

cm 3
3
V nv 4 mol 53.51
214.0 cm

mol

Now, we can calculate the volume change of mixing by using Equation 6.43:

71

Vmix V V1 V2 214 cm 3 175.7 cm 3 39.77 cm 3

Vmix 1.47 cm 3

The intensive volume change of mixing:

Vmix 1.47 cm 3
v mix

0.368 cm 3 / mol
n
4

We can use Equation 6.65 to determine the partial molar volume of formamide:
V2 v x1

dv
v
v x1
dx1
x1

From the provided data table


45.8
45.7

v
0.8401 0.8701 19.50 cm 3 / mol
x1 0.8009 0.6986

Therefore,

V2 53.51 cm 3 / mol 0.75 19.5 cm 3 / mol 38.89 cm 3 / mol

Now calculate the partial molar volume of ethanol:


V n1V1 n2V2

214.0 cm 3 1 mol 38.89 cm 3 /mol


V1
58.37 cm 3 / mol
3 mol

72

6.39
Using the definition of G, the Gibbs energy of mixing of an ideal gas can be rewritten in terms of
the enthalpy of mixing and the entropy of mixing:
ideal gas
ideal gas
ideal gas
g mix
hmix
Tsmix

Since an ideal gas exerts no intermolecular interactions,


ideal gas
hmix
0

From Equation 6.48:


ideal gas
smix
R xa ln xa xb ln xb

so
ideal gas
g mix
RT xa ln xa xb ln xb

To find the partial molar Gibbs energy of mixing of species a, we apply Equation 6.29:

Gmix a nng mix

T , P , nb

Applying the expression above

nb
na
RT na ln na nb ln nb na nb lnna nb
ng mix RT na ln
nb ln
na nb
na nb

where the mathematical relation of logarithms was used. Thus,

n nb RT ln x
n
RT ln na a 0 ln na nb a
a
na nb
na
a

T , P , nb

at infinite dilution xa goes to zero, and the ln term blows up,

Gmix a nng mix

Gmix a a g a
As chemical engineers, we are often interested in the limiting case of infinite dilution. We see
that even for ideal gas mixtures the chemical potential in this limit is not mathematically well.behaved, In Chapter 7, we will develop a different function, the fugacity, which behaves better.

73

6.40
At equilibrium
l
H

v
H
2O

or
G Hl O G Hv O
2
2

Since the water in the liquid phase is pure


GHl 2O g Hl 2O hHl 2O Ts Hl 2O

The enthalpy and entropy of liquids are not sensitive to pressure changes. We can use data from
the saturated steam tables at 25 C to determine the Gibbs energy.
l
g H
104.87 kJ/kg 298.15 K 0.3673 kJ/kg K
2O
l
g H
4.640 kJ/kg
2O

l
l
gH
MWH 2 O g H
0.0180148 kg/mol 4.640 kJ/kg 0.0836 kJ/mol
2O
2O
Therefore,
v
H

2O

l
gH
83.6 J/mol
2O

74

You might also like